Anda di halaman 1dari 107

Universidad La Salle.

Facultad Mexicana de Medicina.


Curso de Extensin Universitaria para la Preparacin del Examen Nacional para
Aspirantes a Residencias Mdicas.
Examen Mdulo de Gineco-obstetricia.
Modalidad a Distancia.

1.- Paciente de 50 aos, con mioma uterino de tamao equivalente a una gestacin de 12
semanas, que presenta hipermenorreas y hemoglobinemia de 9 gr%. No se demuestra
patologa asociada. Se encuentra en espera para la prctica de una histerectoma
programada a realizar en 4 meses. En esta paciente est indicado el tratamiento
preoperatorio con:
a) Estrgenos.
b) Inhibidores de la fibrinlisis.
c) Anlogos de la GnRH.
d) Derivados del cornezuelo del centeno.

Anlogos de la GnRH: Son derivados de la hormona GnRH en donde se ha realizado una


sustitucin peptdica en posicin 6 y en algunos casos en la 10, obteniendo compuestos
hasta unas 200 veces ms potentes debido a mayor afinidad por los receptores y a su
resistencia a la degradacin por peptidasas. Aunque su accin inicial produce un incremento
en la produccin de FSH-LH (efecto flure-up o llamarada) tras 56 das de exposicin contnua, los receptores son internalizados producindose un estado
de hipogonadismo hipogonadotropo y niveles de estradiol similares a los de la
postmenopausia. Los anlogos de GnRH estn disponibles en distintas frmulas:
administracin nasal (varias aplicaciones al da), subcutnea (aplicacin diaria) o
intramuscular (preparados depot mensuales o trimestrales) (Shaw RW 1999).
Marco Filicori y sus colaboradores de la Universidad de Bolonia fueron los primeros en
utilizar en 1983 los aGnRH en un estudio que confirm su eficacia para reducir el tamao
de los miomas uterinos y secundariamente sntomas como alteraciones menstruales, dolor
plvico y sntomas de presin local. Otros autores como Minaguchi H y colaboradores
continan comprobando la efectividad de los anlogos de la GnRH en el tratamiento del
mioma uterino tras evaluar en el ao 2000 seis estudios con un total de 602 pacientes
tratadas con nafarelina. La disminucin del tamao se calcula entre un 30-70%, y se ha
observado como el mayor porcentaje de reduccin ocurre tras el primer mes de
tratamiento, no existiendo reducciones o siendo stas mnimas despus del tercer mes
(Healy et al 1986; Friedman et al 1989; Matta et al 1989; Williams y Shaw 1990). En
miomas pediculados o con gran proporcin de calcio o colgeno (hialinizacin) la repuesta es
tambin menor. Debe tenerse en cuenta que si despus de dos meses de tratamiento no se
ha producido un significativo descenso del tamao del mioma, ste ya no debe ser esperado
y debe pensarse en la posibilidad de la existencia de un tumor muscular maligno no
diagnosticado (Messia AF et al 1998). Tras finalizar el tratamiento y recuperarse el estado
de hipogonadismo, el mioma retorna rpidamente a su tamao inicial (Friedman AJ et al
1987; Matta WH et al 1989).

En casos prximos a la menopausia, la reduccin del tamao del mioma y su sintomatologa,


permitira hablar de una solucin mdica del problema, pero en todo caso la utilizacin de
aGnRH facilitara la intervencin quirrgica al acortar el tiempo de intervencin, la
hemorragia y el acceso a localizaciones complicadas como el caso de miomas
interligamentarios o situados en istmo o crvix. En el caso de la ciruga histeroscpica la
reduccin del tiempo de ciruga permitira reducir el volumen de fluidos aportado a cavidad
uterina y los riegos de absorcin e hiponatremia.
Los mecanismos de accin por los que los aGnRH actan son: o Hipoestrogenemia: es
necesario mantener la hipoestrogenemia, pues la elevacin de sus niveles lleva a un rpido
incremento del tamao del mioma. El crecimiento del mioma es dependiente de los niveles
de estrgenos (aumentan de tamao con el embarazo y se reducen durante la menopausia o
el tratamiento con aGnRH, pudiendo volver a crecer durante la THS), pero aunque los
estrgenos parecen ser importantes en el crecimiento del mioma, su relacin debe ser algo
ms compleja pues no se han descrito incrementos significativos del tamao de miomas
durante el tratamiento con gonadotrofinas en RA (situaciones con elevados niveles de E2),
algunos de ellos no se modifican durante el embarazo o incluso decrecen y se han
encontrado crecimientos despus del tratamiento con citrato de clomifeno (antiestrgeno).
En relacin con la hipoestrogenemia podran estar los cambios inducidos en el flujo vascular
uterino (incrementos en el ndice de resistencia de las arterias uterinas) que suponen una
reduccin de la vascularizacin o las modificaciones de distintos factores de crecimiento. o
Cambios histolgicos: el tratamiento con aGnRH puede producir degeneracin roja,
infiltracin linfocitaria, y necrosis, as como reduccin de la proliferacin celular e
incremento de la apoptosis. Pero en otras circunstancias no es posible encontrar 7
diferencias. No se ha encontrado una relacin entre los cambios histolgicos y el
porcentaje de reduccin del tamao del tero, y existe una gran variabilidad entre
distintas pacientes o entre distintos miomas de una misma paciente, no existiendo pues un
patrn histolgico caracterstico de respuesta ante el tratamiento con aGnRH.
BIBLIOGRAFA
1. Abad L, Abad de Velasco L, Parilla JJ. Etiopatogenia. Papel de las hormonas
esteroideas, factores de crecimiento y otras sustancias. Cuad Med Reprod
1999;5(1):15-29.
2. Albano C, Platteau P, Devroey P. Gonadotropin-releasing hormone antagonist: how good is
the new hope? Curr Opin Obstet Gynecol
2001;13(3):257-62.
3. Coutinho EM.Treatment of large fibroids with high doses of gestrinone.
Gynecol Obstet Invest 1990;30(1):44-47.
4. Chavez NF, Stewart EA. Medical treatment of uterine fibroids. Clin Obstet
Gynecol 2001;44(2):327-84.
5. De Leo V, la Marca A, Morgante G. Shortterm treatment of uterine
fibromyomas with danazol. Gynecol Obstet Invest 1999;47(4):258-262.
6. Eldar-Geva T, Healy DL. Other medical management of uterine fibroids.
Baillieres Clin Obstet Gynaecol 1998;12(2):269-88.
7. Felberbaum RE, Germer U, Ludwig M, Riethmuller-Winzen H, Heise S,
Buttge I, Bauer O, Reissmann T, Engel J, Diedrich K. Treatment of uterine fibroids with a
slow-release formulation of the gonadotrophin releasing hormone
antagonist Cetrorelix.HumReprod 1998;13(6):1660-8.

2.- Al realizar el diagnstico de hirsutismo asociado a ovario poliqustico, usted elige el


siguiente frmaco para su tratamiento por ser el ms adecuado:

a) Clomifeno.
b) Estrgenos.
c) Corticoide.
d) Acetato de ciproterona.

El acetato de ciproterona parece ser ms efectivo que otros frmacos para el hirsutismo
en mujeres causado por la produccin ovrica excesiva de andrgenos.
Una de las causas de hirsutismo (crecimiento piloso excesivo) en mujeres es la
hiperproduccin de andrgenos a partir del ovario. Varios frmacos pueden utilizarse para
contrarrestar los efectos del andrgeno. El acetato de ciproterona es un frmaco
antiandrognico. Los efectos adversos informados con su uso fueron aumento de peso,
depresin, fatiga, sntomas mamarios y disfuncin sexual. La revisin de los ensayos
encontr que el acetato de ciproterona parece ejercer un efecto en el hirsutismo similar a
otros frmacos utilizados para el tratamiento del hirsutismo por exceso de andrgenos.
No existen pruebas suficientes para comparar los efectos adversos de las opciones de
tratamiento.
Van der Spuy ZM, le Roux PA. Acetato de ciproterona para el hirsutismo (Revisin
Cochrane traducida). En: La Biblioteca Cochrane Plus, nmero 4, 2007. Oxford, Update
Software Ltd. Disponible en: http://www.update-software.com. (Traducida de The
Cochrane Library, 2007 Issue 4. Chichester, UK: John Wiley & Sons, Ltd.).

3.- Femenino de 36 aos nulpara tras 2 aos de relaciones sexuales sin contracepcin, que
desde hace 1 ao presenta dismenorrea, dispareunia y sangrado vaginal intermenstrual.
Cul sera la primera orientacin diagnstica?:
a) Insuficiencia lutenica.
b) Enfermedad inflamatoria plvica.
c) Dismenorrea funcional.
d) Endometriosis.
La endometriosis consiste en la aparicin y crecimiento de tejido endometrial fuera del
tero, sobre todo en la cavidad plvica como en los ovarios, detrs del tero, en los
ligamentos uterinos, en la vejiga urinaria o en el intestino. Es menos frecuente que la
endometriosis aparezca fuera del abdomen como en los pulmones o en otras partes del
cuerpo.

La endometriosis es una enfermedad relativamente frecuente, que puede afectar a


cualquier mujer en edad frtil, desde la menarquia hasta la menopausia, aunque algunas
veces, la endometriosis puede durar hasta despus de la menopausia. La endometriosis
altera la calidad de vida de las mujeres que la padecen, afectando a sus relaciones de
pareja, familiares, laborales y de reproduccin.
Sntomas
Los sntomas clsicos son la dismenorrea, dolor plvico, dispareunia, sangrados
intermestruales y en muchos casos, esterilidad.
El dolor no tiene que ver con el tamao y la severidad de la lesin; generalmente cuanto
menor es la lesin mayor dolor produce. El dolor se agrava con las menstruaciones y en los
casos en que la lesin ocupa el fondo de saco de Douglas, puede dar dispareunia. Existe un
aumento de la PGF2 alfa y PGE2 y un aumento de las contracciones uterinas que podra
deberse a un depsito de endometrio en la cavidad peritoneal.
La esterilidad debido a la endometriosis podra deberse a distintas causas de acuerdo a la
severidad de la patologa. En los casos de endometriosis severa puede haber un factor
tuboperitoneal con adherencias y alteracin en la anatoma de la pelvis que interfiera con el
transporte del esperma y el vulo. En los casos de endometriosis leve hay varios
mecanismos propuestos que justifican su relacin con la infertilidad: foliculognesis
alterada, fase ltea inadecuada, fagocitosis espermtica, mala calidad ovocitaria,
embriotoxicidad y alteracin a nivel de la implantacin.. La produccin de prostaglandinas
por el endometrio ectpico puede afectar la motilidad tubaria, la foliculognesis y la
funcin del cuerpo lteo. Puede haber un aumento de la activacin de los macrfagos
peritoneales en la endometriosis que cause la fagocitosis de los espermas o la secrecin de
citoquinas que pueden ser txicas para el embrin. Segn algunos investigadores habra un
60% de las mujeres con endometriosis que presentan un sndrome de Folculo Luteinizado
no roto (LUF) en el cual el folculo no se rompe en la ovulacin y el vulo queda atrapado.

Referencias bibliogrficas:

1.RuizV.Endometriosisyfertilidad.Ed.AcostayWarman,pp.99.
2.Lpes,VH.PalomoE.Incidenciasdeendometriosisenunapoblacininfrtil.XXICongreso
nacionaldeGinecologayObtetricia.Guatemala,1993.
3.ElEoley,etal.Danazolbutnotginadotropinreleasinghormoneagonistssuppresses
autoantibodiesinendomeriosis.FertilSteril1990;54:725.

4.AcostaAA.ButtramVCJr.BeschPK,MalinakLR,VanDerHeydenJ.A.proposedclassfication
ofpelvicendometriosis.ObstetGynecol1973;42:19.
5.ButtranVCJr.EvolutionoftherevisedAmericanFertilityclassificationofendometriosis.
Fert.Steril1985;43:347.
6.LpezVH.Tratamientomdicoquirrgicodelaendometriosis.SimposioElrostrocambiante
delaendometriosispanam3.12.1993.
7.SteinleitnerA.Heteroloustransplationofactivatedmurineperitonelmacrophagesinhibitis
gameteinteractioninvivo;Aparadigmfoendometriosisassocitedsubfertility.FertilSteril
1990;54:725.
8.DamewoodM.Effectofserumfrompatientswithminimaltomildendometriosisonmouse
embryogrowth.FertilSteril1990;54:917.
9.ProugS.Peritonealfluidfracctionsfrompatientswithendometriosisdonotpromotetwo
cellmouseembryogrowth.FertilSteril1990;54:927.

4.- Se presenta paciente de 40 aos de edad a su consultorio refiriendo mastalgia que es


ms severa antes de la menstruacin. A la palpacin hay nodularidad excesiva, hiperestesia
y reas qusticas que la paciente refiere disminuyen en tamao despus de la menstruacin.
El diagnstico ms probable es:
a) Mastopata fibroqustica.
b) Fibroadenomas.
c) Papiloma intraductal.
d) Cncer de mama.

Es raro encontrar una mujer mayor de 35 aos a quien no le hayan dicho, en un examen
fsico mamario, ecogrfico o mamogrfico, que tiene quistes en la mama o que su mama es
mastoptica.
Es el trastorno benigno de la mama ms frecuente y consiste en un aumento del tejido
mamario, especialmente en las zonas superiores y externas de las mamas, hacia las axilas,
que las hace ms densas.
La mastopata fibroqustica suele presentarse en ambas mamas, aunque puede ser de
diferente intensidad en una que en otra.
Puede presentarse a cualquier edad despus del inicio de la menstruacin, pero es ms
probable que aparezca entre los 30 aos y la menopausia. Raramente se presenta ms tarde
de esa edad.
El origen de este trastorno es funcional y responde a desequilibrios de las hormonas
sexuales femeninas y puede condicionar la aparicin de quistes mamarios.

Los sntomas pueden fluctuar de leves a severos en una mastopata fibroquistica mamaria,
se acentan tpicamente antes de cada perodo menstrual y desaparecen inmediatamente
despus.
Los sntomas abarcan:

Consistencia de protuberancias (como de "guijarros"), irregular y densa del tejido


mamario.
o Generalmente ms notoria en la parte superior externa de la mama.
Molestia en las mamas.
o Generalmente en ambas mamas.
o Puede ser persistente o puede aparecer y desaparecer.
Sensacin de llenura en las mamas.
Sensibilidad y dolor sordo e intenso.
Sensibilidad y edema premenstrual.

Secrecin ocasional del pezn.

Bibliografa:
1.
2.
3.
4.
5.

6.
7.

Jones III HW, Wentz AC. Tratado de Ginecologa de Novak. Editorial


Interamericana-McGraw Hill. Undcima Edicin 1994.
DiSaia-Creasman. Oncologa Ginecolgica Clnica. Editorial Mosby. Cuarta Edicin
1994.
Van Dinh T. Sumario de Patologa Ginecolgica. Editorial La Prensa Mdica
Mexicana. 1992.
Pernoll ML. Diagnstico y Tratamiento Ginecoobsttricos. El Manual Moderno.
Mxico. Sexta Edicin 1991.
Alvarez-Bravo A. Diagnstico de los trastornos menstruales y hemorrgicos. En:
Alfonso Alvarez Bravo y su obra. Editorial Marketing y Publicidad SA. Tomo I.
1993.
Vzquez E. Aspectos histoqumicos del endometrio humano despus del tratamiento
con progestgenos sintticos. Gac Md Mx 1966; 96: 1277-93.
Huerta MR, Malacara JM, Rivera-Cisneros A, Daz Cisneros FJ. Sntomas en
adolescentes de dos ciudades de Mxico y su asociacin con el ciclo menstual. Ginec
Obstet Mx 1994; 62: 146-50.

5.- Femenino de 26 aos G-3, P-1, A-1 con 39 SDG por FUR. Reporta contracciones
uterinas que han sido regulares las ltimas tres horas. Al examen encuentras que las
contracciones son cada tres minutos y duran 50 segundos y son firmes a la palpacin. Tuvo
ruptura de membranas hace una hora y lo demuestras con papel de nitrazina. El examen
digital cervical demuestra una dilatacin de 5 cm, con borramiento del 100% y presentacin
en vrtex en estacin 0. Cual de los siguientes criterios es el ms preciso para decir que
se encuentra en la fase activa del trabajo de parto?
a)
b)
c)
d)

Borramiento cervical ms de 90%


Duracin de las contracciones de ms de 30 seg.
Dilatacin cervical mayor de tres centmetros.
Ruptura de membranas.

FASES DEL TRABAJO DE PARTO


El trabajo de parto se divide en tres fases:

Fase 1 latente
Es llamado as al periodo que sirve para la preparacin uterina del parto, ocurre al final del
embarazo y va hasta el inicio de las contracciones del trabajo de parto. Los aspectos a
destacar en este lapso es el reblandecimiento cervical, el aumento importante en el nmero
de receptores para oxitocina a nivel de las clulas endometriales, un aumento sustancial en
los puentes de unin y el nmero de conexinas a nivel miometrial y por consiguiente una
mayor sensibilidad a los agentes uterotnicos.

Fase 2 activa
Es el lapso que representa el trabajo de parto activo, y se acepta que se inicie cuando
existen 3 cm de dilatacin y las contracciones uterinas son aptas para producir un avance
en el trabajo de parto; se divide en tres periodos:

Primer periodo. Se inicia cuando las contracciones uterinas alcanzan la frecuencia,


intensidad y duracin suficientes para causar borramiento y dilatacin del cuello uterino, y
finaliza cuando ste se encuentra en completa dilatacin.
El lapso de tiempo que dura es variable, pero se acepta como normal hasta diez horas en
primigrvidas y ocho horas en multigrvidas; pero independientemente de esto, se debe
considerar como adecuado si el borramiento y la dilatacin cervical son progresivos e
ininterrumpidos.

Segundo periodo. Se inicia con una dilatacin cervical completa y termina con la expulsin

del feto; tiene una duracin variable, pero se acepta como normal una hora en pacientes
primparas y 30 minutos en multparas; y tiene como caracterstica que debe de ser
progresivo e ininterrumpido.

Tercer periodo. Este comienza inmediatamente finalizada la expulsin fetal y termina con

la expulsin total de la placenta y las membranas corioamniticas; a este periodo se le


conoce tambin como de alumbramiento y es el ms corto de los periodos del parto; como
norma general se acepta que no debe de extenderse ms all de 10 minutos.
Existen algunos autores que incluyen un cuarto periodo dentro del trabajo de parto, el
cual abarca aproximadamente la hora posterior al alumbramiento, y comprende el lapso de
tiempo cuando ocurre la contraccin y retraccin de las fibras miometriales, as como la
trombosis de los vasos adyacentes, lo cual es un efectivo control de la hemorragia del sitio
de implantacin de la placenta.

Fase 3
Este periodo es el que representa el regreso de la mujer a su estado previo al embarazo, y
se caracteriza por la involucin uterina, la eyeccin lctea y por ltimo la restauracin de la
fertilidad; existen estudios que involucran en esta fase a la endotelina-1 y a la oxitocina
como substancias responsables de estos cambios postparto.
PROGRAMA DE ACTUALIZACION CONTINUA PARA GINECOLOGA Y OBSTETRICIA
PAC GO-1 Libro 3 Obstetricia 2005

6.- Femenino de 28 aos G5 P2 A3 tiene una historia de abuso de sustancias prenatal. A las
37 sdg tiene una parto vaginal obtenindose una neonato pequeo para la edad gestacional,
de gnero masculino con pqueas aperturas palpebrales, pliegues epicnticos, con cara
aplanada, filtrum hipoplsico y borde del vermillon delgado. Estos hallazgos son
caractersticos en neonatos cuyas madres tuvieron abuso prenatal de cual de las siguientes
sustancias:
a)
b)
c)
d)

Tabaquismo.
Alcohol.
Marihuana.
Narcticos.

EFECTOS DEL ALCOHOL SOBRE EL RECIEN NACIDO


1 - Efectos somticos.
En 1967 Lemoine y colaboradores describieron las anomalas observadas en hijos de madres
alcohlicas; posteriormente, en 1973, Jones, Smith y colaboradores las denominaron
sndrome alcohlico fetal (SAF). Las principales caractersticas observadas en los nios con
SAF son las siguientes: En el 50 a 80% hubo retraso del crecimiento intrauterino,
microcefalia, apertura palpebral estrecha, nariz corta y respingona, mandbula hipoplsica y
labio superior fino. Tambin se ha asociado a cardiopatas congnitas, anomalas en
extremidades y en la columna vertebral.
BIBLIOGRAFIA:
1.

Bell GL, Lau K. Problemas perinatales y neonatales por abuso de sustancias. Clin
Pediatr Nort America 1995, 2:247-266.
2. Byrd RS, Howard CR. Childrens passive and prenatal exposure to cigarrette
smoke. Pediatr Annals 1995: 24(12): 644-645.
3. Cruz M, Bosch J. Sndromes Pediatricos. Barcelona, Espaxs Publicaciones
Mdicas1998, 534-535.
4. Eyler FD, Behnke M. Desarrollo temprano en lactantes con exposicin a drogas. Clin
Perinatol 1999; 1: 105-149.

7.- Hace 40 horas una mujer de 19 aos, gesta 1, tuvo un parto con un producto femenino
de 3,600 g. la calificacin de Apgar en el RN fue de 9 al primer minuto y 9 a los 5 minutos.
La revisin de sus registros de trabajo de parto mostr que tuvo ruptura de membranas 7
horas antes del parto. Sus signos vitales al momento son: temperatura 38.2 C, FC 105 x,
TA 110/70, FR 16x; en la exploracin fsica el dolor leve a la palpacin del tero, las
mamas se encontraban sin eritema, ni dolor a la palpacin, no haba dolor a la compresin de
las pantorrillas. Cul de los siguientes es el mejor tratamiento antes de iniciar los
antibiticos?

a) Hemocultivo.
b) Cultivo de secrecin vaginal.
c) Examen general de orina y cultivo.
d) Espirometra por incentivo.

El vaciamiento incompleto ocasiona orina residual, distensin vesical excesiva y estasis,


adems del cateterismo intermitente con sonda vesical durante el trabajo de parto. Por lo
tanto, la vejiga en el puerperio est predispuesta a infecciones. El dolor leve a la palpacin
del tero puede ser normal en el puerperio y no se debe suponer de inmediato endometritis
puerperal. Cuando se sospecha endometritis, los cultivos de secrecin vaginal tienen poca
utilidad porque se encuentran los mimos microorganismos que en mujeres purperas sanas.
Los hemocultivos son apropiados para la valoracin diagnstica de la fiebre puerperal, pero
no son el paso inicial. La espirometra por incentivo se utiliza en el posoperatorio de
inmediato para fomentar la expansin pulmonar y disminuir las atelectasias. El legrado
uterino se utiliza para tratar la hemorragia.
Morgan M, Siddighi S. Ginecologa y obstetricia, National Medical Series. 5 edicin. Mc
Graw Hill. Pp. 29.

8.- Femenino de 34 aos que inicia tratamiento con sulfato de magnesio por presentar
eclampsia, se presentan signos de sobre dosificacin qu antdoto se debe emplear?

a) Gluconato clcico.
b) Nitroprusiato.
c) Simpaticomimticos.
d) Carbonato sdico.

NIVEL DE PRIMER CONTACTO (ATENCION PRIMARIA)


Se debe instruir a todas las embarazadas que deben acudir inmediatamente a un centro de
salud en cualquiera de los siguientes casos:

Edema que se desarrolla rpidamente (en pocos das)


Cefalea severa y persistente.
Dolor en la regin abdominal superior.
Visin borrosa.

Se debe realizar la medicin de la presin arterial y un anlisis de orina para la deteccin


de proteinuria a las mujeres que acudan a centros de salud presentando estos sntomas.

Convulsiones
Si se asiste a una mujer con eclampsia en un centro de atencin primaria:
1. Deben mantenerse las vas respiratorias permeables.
2. Se debe colocar a la mujer de costado (posicin decbito lateral izquierda) para evitar la
aspiracin del vmito u otras secreciones.
3. Si es posible, se debe establecer una va intravenosa.
4. Se debe administrar sulfato de magnesio.
Monitoreo de la administracin de sulfato de magnesio: Durante el tratamiento con sulfato
de magnesio, se recomienda realizar un control cada 4 horas, como mnimo, para detectar la
presencia de: Reflejo rotuliano, frecuencia respiratoria superior a 16 por minuto, volumen
de orina >100 ml en las 4 horas previas.
- Sobredosis de sulfato de magnesio: Todo centro de salud que utilice sulfato de magnesio
debe disponer de ampollas de gluconato de calcio (1 g) como antdoto para la sobredosis de
dicho frmaco.

Se sugiere medir la presin arterial y administrar antihipertensivos segn


corresponda.
Convulsiones recurrentes: En caso de convulsiones recurrentes, se administran
otros 2 a 4 g. de sulfato de magnesio por va IV en el lapso de 5 minutos, tanto para
el rgimen IM como el IV; la dosis se determina en funcin del peso de la paciente.

El sulfato de magnesio es un frmaco usado en el control de las convulsiones eclmpticas,


para suprimir o controlar las contracciones uterinas sean estas espontneas o inducidas, y
como broncodilatador luego del uso de beta agonistas y agentes anticolinergicos. Tambin
tiene indicacin como terapia de reemplazo en la deficiencia de magnesio, como laxante
para reducir la absorcin de txicos del tracto gastrointestinal. El sulfato de magnesio
esta ganando popularidad como tratamiento de inicio en el manejo de algunas arritmias,
particularmente en Torsades de Pointes, y en arritmias secundarias a sobredosis de

antidepresivos tricclicos o toxicidad digitlica. Esta tambin considerado clase Ila


(probable beneficio) para la fibrilacin ventricular refractaria y la taquicardia ventricular,
luego de la administracin de dosis de lidocaina y bretilio.

FARMACODINAMIA
El sulfato de magnesio tiene la capacidad de alterar la excitabilidad de la fibra miometrial,
afecta el acoplamiento excitacin contraccin y el proceso mismo de contraccin, inhibe
la entrada de calcio al sarcoplasma y reduce la frecuencia de los potenciales de accin.
Inhibe tambin la liberacin de acetilcolina. Por ser estas acciones comunes en las fibras
musculares se pueden ver afectadas tambin la musculatura voluntaria e incluso las fibras
miocrdicas.(1)

Bibliografa.
Graves C. Frmacos que contraen o relajan el tero. En: Hardman J, Limbird L, Molinoff P,
Ruddon R, Goodman A, eds. Goodman & Gilman. Las Bases Farmacolgicas de la Teraputica.
9 ed. Mxico DF: McGraw-Hill Interamericana; 1996. pp. 1012-3.

9.- Mujer de 32 aos que cursa en ste momento con diagnstico de preclampsia leve, el
frmaco de eleccin que se administra en esta patologa es:

a)
b)
c)
d)

Nifedipina.
Inhibidores de la enzima convertidora de angiotensina.
Clonidinas.
Alfametildopa.

Prevenir complicaciones a corto plazo de las mujeres con PA elevada que


comprometa el bienestar fetal.
Cuando la PAS es mayor o igual a 150 mmHg y la PAD mayor o igual a 100 mmHg.
El propsito es alcanzar cifras de TA alrededor de 140/90.
La medicacin antihipertensiva se reserva para los casos en que la PAD 100 mmHg.
Se recomienda continuar el tratamiento antihipertensivo previo al embarazo,
exceptuando el uso de IECA.

La alfametildopa y la hidralazina va oral son los frmacos de eleccin dado su uso


extensivo con seguridad y eficacia y sin efectos colaterales para el feto (excepto
hidralazina en lupus).

ALFA METILDOPA
500-2000 MG/DA

HIDRALAZINA
50-200 MG/DA
LABETALOL
100-400 MG/DIA
ATENOLOL
50-200 MG/DA
NIFEDIPINA
10-30 MG/DA

1. Aagard K, Belfort M. Eclampsia: Morbility, mortality, and management. Clin


Obstet Gynecolol. 2005; 48: 12-23.
2. Oyarzn E. Sndrome hipertensivo del embarazo en Oyarzn E. Ed. Embarazo de alto
riesgo. Ediciones Universidad Catlica de Chile. Santiago. 1997: 157175.
3. Roberts J, Redman C. Pre-eclamsia: More than pregnancy induced hypertens.

10.- Una mujer embarazada, puede afectar al feto y hacerlo contraer lesiones
importantes durante el embarazo o al salir al exterior (atravesando el canal de parto), s la
gestante se encuentra afectada de la siguiente patologa:

a) Tricomonas.
b) Gardenerella.
c) Herpes genital.
d) Gonococos.

Herpes genital
La prevalencia de herpes simplex genital o tipo 2 (VHS-2) en mujeres embarazadas vara
entre 7 y 33% en distintas series. La prevalencia ha experimentado un sostenido aumento
durante los ltimos aos. Se estima que aproximadamente 1 a 3% de las mujeres adquiere
cada ao la infeccin. En el caso de parejas discordantes, la tasa de adquisicin aumenta
hasta 10 a 30% anual. La adquisicin durante el embarazo es ~2%.

La transmisin al hijo ocurre principalmente cuando la mujer embarazada adquiere una


infeccin primaria. La transmisin es de 30 a 50% cuando la infeccin primaria ocurre
cerca del momento del parto. La mayor transmisin (85%) ocurre durante el parto. Sin
embargo, tambin puede ocurrir transmisin intrauterina (5-8%) y post-natal (8-10%). Los
factores que inciden en la transmisin son: infeccin primaria mucho mayor eficiencia que
infeccin recurrente, parejas discordantes, ttulos de anticuerpos maternos y
procedimientos obsttricos invasores, (los que estn absolutamente contraindicados).
Las manifestaciones en la mujer embarazada son principalmente bajo la forma de herpes
genital localizado, muy raramente ocurre diseminacin cutnea y visceral, situacin de
elevada mortalidad (50%). La infeccin en el nio, si ocurre en las primeras 20 semanas del
embarazo, puede provocar aborto en 25%, malformaciones cerebrales, cicatrices,
corioretinitis, RCIU. Si ocurre despus de las 20 semanas, puede causar parto prematuro,
RCIU, o herpes neonatal. Esta condicin clnica tiene tres formas de presentacin, las dos
primeras de elevada mortalidad y secuelas: herpes diseminado y encefalitis herptica o
infeccin localizada en piel, ojo y boca.
Referencia Bibliogrfica:
1.

Pass R, Weber T, Whitley RJ. Herpesvirus infections in pregnancy.


Recommendations from the International Herpes Management Forum.
Management Strategies Workshop and 7th Annual Meeting.
2. Whitley R J. Varicella - Zoster virus. Mandell, Douglas and Bennett's
Principles and Practice of Infectious Diseases. Mandell G, Bennett J,
Dolin R, editors. Fifth edition, 2000 Churchill Livingstone, Philadelphia,
pp: 1586-98.
3. Abarca K, Cohen B J, Vial P. Seroprevalence of parvovirus B19 in urban
Chilean children and young adults, 1990 and 1996. Epidemiol Infect 2002;
128: 59-62.

11.- Femenino de 64 aos de edad con la siguiente sintomatologa: plenitud, estreimiento,


imagen qustica en el ultrasonido plvico en ovario derecho de 15 por 15 cms. El diagnstico
ms probable es:
a) Teratoma qustico.
b) Disgerminoma.
c) Endometrioma.
d) Cistadenoma seroso.

Los Tumores de Ovario son una patologa frecuente dentro del contexto de la patologa
femenina. Por esta causa consultan un grupo elevado de mujeres, tanto las consultas de
ginecologa como las de Ciruga propiamente dicha. Las edades oscilan desde las tempranas
hasta las ya avanzadas, siendo el riesgo de degeneracin maligna muy variable y relacionado
con le edad. La experiencia de la clnica revela la alta incidencia de tumores de ovario en la
etapa del climaterio, comprendida entre los 35 y 65 aos de edad 1.
El cistoadenoma seroso de ovario (CSO) es un tipo de tumor derivado del epitelio
superficial (celmico), formado por reas qusticas. El cistoadenoma seroso de ovario es el
tumor ms frecuente de aquellos que provienen del epitelio celmico superficial. Hay
tumores pequeos macroscpicamente y tumores masivos que ocupan toda la pelvis e incluso
la cavidad abdominal. Estas frecuentes neoplasias qusticas uniloculares estn tapizadas
por clulas epiteliales altas, cilndricas y ciliadas, llenas de un lquido seroso claro y de
superficie lisa con abundantes vasos. Las variedades benigna, limtrofe y maligna
representan, en conjunto, 30% aproximadamente de todos los tumores del ovario. El riesgo
de presentar tumores epiteliales se incrementa con el paso de la edad, ya que pese a que la
declinacin de la funcin ovrica marca el envejecimiento gonadal progresivo, el ovario
humano nunca pierde su capacidad para generar tumores. Por lo general, cuando es
detectado, su tamao es grande, en donde la imagenologa puede ayudarnos a considerar su
diagnstico

1.
Captulo 22 Tumores Benignos de Ovario. En: Novak ER, Jones G., Jokes HW.
Tratado de Ginecologa. 9 ed. Ciudad de la Habana. Editorial Cientfico Tcnica;
1977.p.432 66.
2.
MedlinePlus Enciclopedia Mdica en Espaol: Quistes Ovricos. Disponible en:
http://vsearch.nlm.nih.gov/vivisimo/cgibin/querymeta?v%3Aproject=medlineplusspanish&s
pell=spell&query=Quistes+Ov%C3%A1ricos Acceso: Actualizado 20/6/06.
Captulo XL Tumores Ovricos En: Llusi Botella J, Nez Clavero JA. Tratado de
Ginecologa. Ciudad de la Habana. Editorial Cientfico Tcnica. 1983; T 3.1; p. 751 803.

12.- Femenino que cursa con 36. 5 semanas de gestacin acude al servicio por referir
malestar general, fosfenos, nausea y vmito, aprecia moderada ictericia, usted sospecha
de un sndrome de HELLP Que alteraciones de laboratorio espera encontrar al confirmar
el diagnstico?
a)
b)
c)
d)

Anemia Hemoltica, trombocitopenia, enzimas hepticas elevadas.


Trombocitosis, Enzimas heptica elevadas, Anemia hemoltica.
Anemia hemolitica, Trombocitosis, Fosfatasa Alcalina elevada.
Trombocitopenia, Leucopenia, Hipertensin Arterial.

DEFINICIN:
Es una complicacin de la preeclampsia en la cual adems de la Hipertensin Arterial
y proteinuria hay presencia de anemia hemoltica, enzimas hepticas elevadas y
recuento bajo de plaquetas.

EPIDEMIOLOGIA:
Se presenta en un 4 a 10% de las preeclmpticas, diagnosticndose anteparto en un
70% de los casos preferentemente antes de las 37 semanas, mientras que el 30%
de los casos restantes enferma en los primeros 7 das del puerperio, sobre todo en
las 48 h iniciales.
La proteinuria e hipertensin pueden estar ausentes en un 15 al 20% de los casos.
Incidencia mayor en multigestantes y en edades avanzadas.
Ocurre ms frecuentemente cuando se demora la salida del feto y cuando se presenta
desprendimiento de la placenta.
Mortalidad materna del 24% y mortalidad perinatal del 30-40%.

CLASIFICACION:
Sndrome de HELLP. Clasificacin de Mississipi.
CLASE
Plaquetopenia
LDH
1
Severa <50000 >600 IU/L
2
Moderada
>600 IU/L
>50000
<100000
3
Ligera >100000 >600 IU/L
<150000
PE severa
>150000
<400 IU/L
Eclampsia (sin
HELLP)

AST-ALT
>70 IU/L
>70 IU/L

>40 IU/L
<70 IU/L
<40IU/L

MANIFESTACIONES CLINICAS:
Malestar general, fatiga y molestias inespecficas 90%
Cefalea 70%
Epigastralgia 64%
Vmito 22%
Fosfenos 15%
Visin Borrosa 11%
Acfenos 3%
Ictericia.
Anemia no explicada.
Oliguria.

Si se aade una HEMORRAGIA HEPTICA, el paciente puede quejarse de dolor


en el HOMBRO DERECHO y EL CUELLO, adems de las molestias abdominales.

Equimosis en los sitios de punciones venosas, petequias en los sitios de presin del
brazo, pero pueden tener pruebas de Rumpel Leed negativas.
En casos severos se pude presentar ascitis como causa de hipertensin portal.

DIAGNOSTICO:
El diagnstico clnico del sndrome de HELLP se plantea en gestantes o purperas con
preeclampsia severa-eclampsia, excepto en el 15-20%, en las cuales esta asociacin no
puede ser demostrada, en tanto se cumplan los criterios de Sibai:

MANIFESTACIONES CLINICAS:
Malestar general, fatiga y molestias inespecficas 90%
Cefalea 70%
Epigastralgia 64%
Vmito 22%
Fosfenos 15%
Visin Borrosa 11%
Acfenos 3%
Ictericia.
Anemia no explicada.
Oliguria.

Si se aade una HEMORRAGIA HEPTICA, el paciente puede quejarse de dolor en el


HOMBRO DERECHO y EL CUELLO, adems de las molestias abdominales.
Equimosis en los sitios de punciones venosas, petequias en los sitios de presin del
brazo, pero pueden tener pruebas de Rumpel Leed negativas.
En casos severos se pude presentar ascitis como causa de hipertensin portal.

DIAGNOSTICO:
El diagnstico clnico del sndrome de HELLP se plantea en gestantes o purperas con
preeclampsia severa-eclampsia, excepto en el 15-20%, en las cuales esta asociacin no
puede ser demostrada, en tanto se cumplan los criterios de Sibai:

HEMOLISIS
Frotis perifrico anormal (eritrocitos fragmentados)
Hematocrito (>24%)
Bilirrubina indirecta (>1.2mg/dL)
Deshidrogenasa lctica (>218 UI/L)

ENZIMAS HEPTICAS ELEVADAS

LDH >218UI/L .
AST >30UI/L .
ALT >37UI/L .

PLAQUETAS BAJAS
<100.000/mm3

BIBLIOGRAFIA
1.

Sibai baha, El sndrome HELLP. Universidad de Valencia , revista quincenal de


Obstetricia clnica y ginecologa, Octubre 2003.
2. V. Cararach, Sndrome de HELLP y Repercusiones maternas. X curso intensivo de
formacin continuada materno fetal. Enero de 2003.
3. Toirac, Abelardo. Sndrome de Weistein HELLP Hospital Ginecoobstetrico Tamara
Bunke. Junio 2002
4. De la Fuente, David. Sndrome HELLP. Medicina Universitria 2003; 5 (19): 101 -9
5. Andrea G. Witlin, DO, Baha M. Sibai, MD. Diagnosis and Management of women
with Hemolysis Elevate Liver Enzymes, and Pletelet Count (HELLP) syndrome.
Hospital Physician. Febrero 1999.
6. CIFUENTES B, Rodrigo. Ginecologa y obstetricia basadas en las evidencias.
Bogot: Distribuna, 2006. Sexta edicin. 447 - 283 p.

13.- Femenino de 23 aos acude al servicio de ginecologa, por referir ciclos opsomenorreicos, desde el inicio de su menarquia, en los ltimos 7 das ha incrementado 15 Kg.
de lo que pesaba habitualmente. Exploracin Fsica: Acn facial importante, as como
bigote.
El diagnostico ms probable en esta paciente es:
a)
b)
c)
d)

Sx. De Asherman.
Sx. Stein Leventhall.
Sx. Amenorrea Galactorrea.
Sx. Karman.

Sndrome de Ovario Poliqustico (SOP) es uno de los ms comunes trastornos endocrinos


que afectan a las mujeres alrededor del 5% al 10% de las mujeres en edad reproductiva
(12-45 aos) y se piensa que es una de las principales causas de la infertilidad femenina.
Las caractersticas principales son la obesidad, anovulacin (dando lugar a la menstruacin
irregular) o amenorrea, acn, y las cantidades excesivas o los efectos de andrognicos
(masculinizantes) hormonas. Los sntomas y la severidad del sndrome varan mucho entre
las mujeres. Si bien las causas son desconocidas, resistencia a la insulina, la diabetes y la
obesidad estn fuertemente correlacionadas con el SOP.
Bibliografa:
1.

Bulun SE, Adashi EY. The physiology and pathology of the female reporductive axis.
In: Kronenberg HM, Melmed S, Polonsky KS, Larsen PR, eds. Williams Textbook of
Endocrinology. 11th ed. Philadelphia, Pa: Saunders Elsevier; 2008: chap 16.

14.- Femenino de 19 aos, atendida en sala de urgencias gineco-obstetricas, Antecedente:


cursa embarazo de 38 SDG. Exploracin Fsica: En trabajo de parto. Repentinamente
presenta sangrado profuso transvaginal y dolor abdominal.
La causa ms probable de la sintomatologa de esta paciente es:
a) Laceracin vaginal por coito.
b) Cervicitis.
c) Placenta previa.
d) Abruptio placentae.

DESPRENDIMIENTO PREMATURO DE PLACENTA NORMOINSERTA (DPPNI):


Constituye la separacin de la placenta de su rea de insercin antes del 3 perodo del
parto. La mortalidad fetal es muy alta (superior al 15%) y la materna es tres veces superior
a la esperada.
La aparicin frecuente de SFA, prematuridad, anemia, etc., hace que el nmero de secuelas
tanto sensitivas como motoras sea alto. Desde el punto de vista materno, complicaciones
secundarias a la hemorragia, a las alteraciones de la coagulacin o bien a la embolia
pulmonar tambin tienen una tasa muy alta.
Desde la antigedad se identific al cuadro clnico caracterizado con la trada sintomtica
de hipertona, metrorragia y muerte fetal, con pronstico materno comprometido. En 1775,
Ricci diferenci la separacin prematura de una placenta de insercin normal, de aquella de
insercin baja; a la primera llam hemorragia accidental, y a la segunda hemorragia
imprevisible. Couvelaire introdujo el trmino de desprendimiento prematuro de placenta
y describi la aparicin de la apopleja uterina
(tero de Couvelaire)
Bibliografa:
1.- Obstetricia. Scwarcz, Sala, Duverges. 7 edic. Edit. El Ateneo. (Biblioteca Fac. Med.
UNNE).

15.- Femenino de 28 aos con un ndice de masa corporal (IMC>30), baches amenorreicos,
acn, hirsutismo y esterilidad de 2 aos de evolucin, cabra pensar en:

a)
b)
c)
d)

Hipotiroidismo.
Fallo ovrico precoz.
Sndrome del ovario poliqustico.
Amenorrea de causa uterina.

El sndrome de ovarios poliqusticos (SOPQ) afecta aproximadamente a un 4% de mujeres


en edad reproductiva y se caracteriza por anovulacin crnica e hiperandrogenismo. Es la
causa ms comn de infertilidad en mujeres.
Se caracteriza clnicamente por acn, alopecia, hirsutismo, irregularidades menstruales
e infertilidad.
Los hallazgos de laboratorio ms frecuentes son: aumento de la hormona luteinizante
(LH), aumento de la relacin LH/FSH (hormona folculoestimulante), aumento de
andrgenos (tanto ovricos como adrenales) y de estrgenos circulantes. Otros hallazgos
de laboratorio habituales son una prueba tolerancia oral a la glucosa anormal y alteraciones
en el perfil lipdico.
Todo esto junto con las imgenes ecocardiogrficas caractersticas definen al sndrome.

La teraputica permite dos grandes enfoques que pueden superponerse: la correccin de


las manifestaciones de hiperandrogenismo y el tratamiento de las alteraciones del eje
reproductivo (anovulacin, esterilidad).
Los antiandrgenos estn fundamentalmente indicados para tratar los sntomas
virilizantes.
Las alternativas para inducir la ovulacin son numerosas: al citrato de clomifeno y a la
antigua reseccin en cua se agregan las gonadotrofinas humanas, pulsos de GnRH (hormona
liberadora de gonadotrofinas), medidas o frmacos para modificar los niveles de insulina, y
finalmente tcnicas quirrgicas endoscpicas para reducir la masa ovrica.
Revista de Posgrado de la VIa Ctedra de Medicina - N 125 Marzo 2003
Pg. 37-40
SINDROME DE OVARIOS POLIQUISTICOS
Dra. Sandra Beneyto, Dra. Mara Andrea Ferreyra, Dr. Andrs Galfrascoli,
Dr. Andrs Gonzlez, Dra. Susana Sosa
16.- Para el diagnstico radiolgico de sinusitis aguda, la proyeccin que mejor valora los
senos maxilares y las estructuras intranasales es?
a)
b)
c)
d)

Lateral.
Waters.
Submentoniana.
Anteroposterior.

La proyeccin Waters es la proyeccin que mejor permite valorar la neumatizacin,


opacificacin o engrosamiento de mucosa de los senos maxilares as como las estructuras
intranasales.

Proyeccin de Waters u occipito-mentoniana para senos maxilares


(Radiografa normal).
Gonzlez-Saldaa N, Infectologa Clnica Peditrica, 7 edicin, pginas 63-98.

17.- Las siguientes manifestaciones clnicas son las ms frecuentes en el lupus:


a)
b)
c)
d)

Las cutneas y las articulares.


Las cardacas.
Las pulmonares.
Las musculares.

Despus de la fatiga, la fiebre, la astenia, las alteraciones de la piel, el pelo y las mucosas
ocupan el segundo lugar entre las manifestaciones clnicas (85% de los casos).
El clsico eritema facial en alas de mariposa ocurre en 52% y es consecuencia frecuente de
fotosensibilidad; tambin se localiza en el trax, espalda y brazos como lesiones
eritematosas simtricas superficiales con zonas centrales atrficas anulares.
El lupus eritematoso discoide se localiza en la piel cabelluda, pabellones auriculares, cara y
cuello y se asocia con frecuencia a fotosensibilidad y fenmeno de Raynaud.
Bibliografa:
GUTIRREZ C, MOZO L. Diagnstico inmunolgico: enfermedades auto inmunes. En:
Inmunologa
Clnica. Bases moleculares y celulares. J. Pea, ed. Madrid. Arn. 107-118, 1996.

18.- En un paciente con diagnstico de Enfermedad de Hirschsprung, Cul de las


siguientes complicaciones es la ms grave?
a)
b)
c)
d)

Vlvulus.
Invaginacin Intestinal.
Sangrado de Tubo Digestivo.
Enterocolitis.

Los problemas con un nio que padece la enfermedad de Hirschsprung dependen de la


porcin de intestino que tiene clulas nerviosas normales. El setenta por ciento de los
bebs con la enfermedad de Hirschsprung carecen de clulas nerviosas solamente en una
porcin de los ltimos treinta o sesenta centmetros (uno o dos pies) del intestino grueso.
Siendo una de las complicaciones mas severas la enterocolitis.
Bibliografa:
1.- Operative Pediatric Surgery. Moritz M. Ziegler. International Edition, pg 445 463
Bibliografa: Urgencias en Pediatra, Interamericana.McGraw Hill. Captulo: Urgencias
Mdico Quirrgicas, Seccin XXIII, Pg. 182-194
Enciclopedia Mdico Quirrgica-Pediatra 4-014-L-10-2005.

19.- La siguiente maniobra es la que nos indica en un recin nacido, que la articulacin de
la cadera esta luxada:
a) Signo de Galeazzi.
b) Signo de Barlow.
c) Signo de Ortolani.
d) Signo de Pistn.

Prueba de Ortolani : Con esta prueba se detecta una cadera ya luxada, se coge con la
mano el miembro flexionado, la cadera se coloca en abduccin mientra se levanta el fmur
con cuidado y se sita los dedos a nivel del trocnter mayor. Si la prueba es positiva se
siente la reduccin de la cadera dentro del acetbulo.
Prueba de Barlow: Es una prueba inductora para identificar una cadera inestable pero
an localizada en su sitio; no es una prueba adecuada para diagnosticar luxacin de cadera.
La cadera en aduccin ligera y con la palma de la mano se empuja suave y cuidadosamente
hacia atrs, la presencia de un movimiento de pistn o al percepcin de una cabeza
femoral subluxada sobre el borde posterior del acetbulo.
Prueba de Galeazzi: Con el nio acostado se le flexionan las caderas y rodillas de modo
que los talones se apoyen sobre la mesa y reconocer el acortamiento relativo del muslo.

Bibliografa:
1.-Skinner, H. Diagnstico y tratamiento en Ortopedia. Ed. Manual Moderno. Mxico,
2004. pp. 625.

20.-Paciente diagnosticado con esclerosis mltiple (considerada como un proceso inflamatorio y desmielinizante del SNC). En el diagnstico de esta enfermedad es muy til la
presencia de?
a) Ms de 100 linocitos por microlitro en el lquido cefalorraqudeo.
b) Ms de 50 polimorfonucleares por microlitro en el lquido cefalorraqudeo.
c) Elevaciones del cido rico en plasma.
d) Bandas oligoclonales en el lquido cefalorraqudeo.

La esclerosis mltiple (EM) es una enfermedad desmielinizante del sistema tema nervioso
central, que habitualmente se caracteriza por ataques recurrentes de disfuncin
neurolgica focal y multifocal.

Los sntomas y signos clsicos de EM dependen de la localizacin del foco de


desmielinizacin y pueden incluir una variedad de disfunciones que no son especficas para
la EM, tales como alteracin de la visin, ataxia y temblor intencional, debilidad o parlisis
de una o ms extremidades, espasticidad y problemas vesicales. Hay,sin embargo, tres
anormalidades, altamente sugestivas de EM:

Neuritis ptica, la cual es el sntoma inicial en cerca del 25% de los pacientes
Oftalmoplejia internuclear que se asocia con nistagmus monoocular
Signo de Lhermitte, que es una sensacin "elctrica" por detrs del cuello y de la
espalda que llega hasta las piernas (al flexionar el cuello).

En fibras nerviosas normales mielinizadas, la conduccin ocurre de manera saltatoria; las


corrientes de accin se confinan a las secciones no mielinizadas del axn saltando de un
ndulo de Ranvier al siguiente. Este tipo de conduccin es mucho ms eficiente desde el
punto de vista energtico que la transmisin a travs de la superficie del axn entero,
aumentando as la velocidad de conduccin con una prdida mnima de energa.
La desmielinizacin de una porcin de la fibra nerviosa normalmente mielinizada, puede
conducir a: bloqueo de la conduccin en el sitio de la prdida de mielina, disminucin de la
velocidad de conduccin nerviosa a travs de la fibra alterada y fatiga subjetiva aumentada
o un mayor consumo de energa.
La disfuncin neurolgica observada en la EM es una reflexin de la alteracin de la
conduccin a travs de segmentos parcial o completamente desmielinizados o de una fibra
nerviosa mielnica. Adems, el hecho de que el tiempo de conduccin a travs de los
segmentos desmielinizados disminuye con aumento de la temperatura puede explicar por
qu los sntomas clnicos de EM se empeoran al aumentar la temperatura corporal.
Diagnstico
El diagnstico de EM se basa fundamentalmente en dos parmetros. Por un lado los
hallazgos en los estudios de imagen por Resonancia Magntica en los que se presentan (en
las imgenes dependientes de T2) reas hiperintensas de localizacin fundamentalmente
periventricular en la substancia blanca subcortical en mltiples localizaciones as como

tambin en la mdula espinal; este estudio permite visualizar no solamente la localizacin de


las reas desmielinizantes sino tambin en forma seriada pueden realizarse evaluaciones
para valorar la evolucin clnica y subclnica de la enfermedad. Los otros estudios bsicos
para el diagnstico son la determinacin de bandas oligoclonales, la determinacin de la
protena bsica de la mielina y la medicin de las inmunoglobulinas en el lquido
cefalorraqudeo. Estos dos factores, en conjunto con la informacin clnica, permiten
establecer con ciertas bases de seguridad el diagnstico de esta enfermedad.
Referencias bibliogrficas
1.
2.
3.
4.
5.
6.
7.

8.
9.
10.

Noseworthy JH, Lucchinetti C, Rodrguez M, Weinshenker BJ. Multiple sclerosis.


N. England J Med. 2000;343:938-52.
Omari KM, John GR, Sealfon SC, Raine CS. CXC chemokine receptors on human
oligodendrocytes: implications for multiple sclerosis. Brain 2005;128:1003-15.
Mi S, Miller RH, Lee X, et al. LINGO-1 negatively regulates myelination by
oligodendrocytes. Nat Neurosci. 2005;8:745-51.
John GR, Shankar SL, Shafit-Zagardo B, et al . Multiple sclerosis: re-expression of
a developmental pathway that restricts remyelination. Nat Med. 2002;8:1115-21.
Lucchinetti C. The spectrum of idiopathic inflammatory demyelinanting disease. In:
American Academy of Neurology. Syllabi on CD ROM. 2000.
Hartung HP, Grossman RI. ADEM: distinct disease or part of the MS spectrum?
Neurology 2001; 56:1257-60.
Capello E, Voskuhl RR, and McFarland HF, Raine CS. Multiple sclerosis: reexpression of a developmental gene in chronic lesions correlates with
remyelination. Ann Neurol. 1997; 41:797-805.
Atlanta. Georgia. AAN. 115 Anual Meeting. Natural Course of Multiple Sclerosis
Redefined: National Inst of Neurological Disorder and Stroke, 16, 1990.
Hemmer B, Archelos JJ, Hartung HP. New concepts in the inmunopathogenesis of
MS. Nat Rev Neurosci. 2002;3:291-301.
Kurtzke JF. Geography in multiple sclerosis. J Neurol. 1977;215:1-26.

21.- Paciente de 25 aos, Gesta 1, Para 1. Con dos citologas lesin de alto grado, prueba
de Schiller positiva y biopsia de crvix que demuestra carcinoma In Situ. La conducta es:
a)

Conizacin.

b)

Histerectoma total abdominal.

c)

Histerectoma y salpingooforectoma bilateral.

d)

Electrocauterizacin del crvix.

La conizacin cervical es el tratamiento de eleccin en pacientes con cncer cervicouterino


microinvasor y ms si existedeseo de fertilidad. Asimismo, la histerectoma extrafasciales
un mtodo adecuado en lesiones de 0.5 a 3 mm de invasin.Adems se propone que, para
pacientes con lesiones de 3.1 a 5 mm de invasin, a partir de la membrana inicial sinfactores
de mal pronstico como invasin vascular y linftica, sean tratadas con histerectoma
extrafascial, ya que en aquellasa las que se realiz linfadenectoma plvica, con este tipode
lesin, no se encontr metstasis a ganglios linfticos.

Resultados del tratamiento en cncer cervicouterino microinvasor en el Instituto Nacional


de Cancerologa de Mxico (1980-1999)

Bibliografa:
1.- Mestwerdt G. Fruhdiagnose des Kollumkarzinoms. Zentralb Gynaekol, 1947 ;69 :326.
2. - Morrow CP, Curtin JP. Surgery for cervical neoplasia. In Gynecologic Cancer Surgery.
New York, Churchill Livingstone, 1996, p 472.3.
3. - Burghardt E, Holzer E. Diagnosis and treatment of microinvasive carcinoma of the
cervix uteri. J Obstet and Gynecol 1977; 49:641-653.
4.- Sedlis A, Sall S, Tsukada Y, et al. Microinvasive carcinoma of the uterine cervix: a
clinical-pathologic study. Am J. Obstet Gynecol. 1979;133:64.

22.- Posterior a un trabajo de parto con expulsin normal, y tras una hora aproximada en
periodo de alumbramiento en el que se practic masaje uterino y se increment
moderadamente la dosis de oxitocina, no aprecian signos de desprendimiento placentario,
se indica una extraccin manual de placenta, que resulta imposible por no existir plano de
separacin entre la placenta y la pared uterina. Cul es el diagnstico ms probable?

a) Engatillamiento placentario.
b) Placenta succenturiata con cotiledn aberrante.
c) Placenta circunvalata.
d) Placenta adherente por acretismo placentario.

Se denomina a la placenta como acreta cuando sta se implanta en zonas donde la decidua
es deficiente o anormal y por tanto hay una infiltracin del miometrio por vellosidades
coriales; esta infiltracin puede ser focal, parcial o total. A su vez esta condicin se
subdivide en acreta, increta y percreta. La placenta increta y percreta infiltran todo el
espesor de la pared miometrial, en la percreta adems las vellosidades, perforan la serosa
y llegan en algunas ocasiones a infiltrar rganos vecinos, especialmente la vejiga. La PA
est limitada a la superficie miometrial. La frecuencia de presentacin del AP vara entre
10 y 48 por 10.000 partos. (Oishi A 1999, Hung TH 1999, Zaki ZM, 1998).

En las mujeres con acretismo placentario se han visto factores de riesgo, dentro de los
cuales se encuentran:
1.
2.
3.

4.
5.
6.

Edad y multiparidad: La presentacin AP, aumenta con la paridad de la paciente y


la edad, siendo muy rara en primparas.
Placenta previa: esta se ha encontrado en el 30% de los casos de PA. Igualmente
se ha visto PA en el 9.3% de las pacientes con placenta previa (Miller DA, 1997).
Cesrea anterior, o cirugas uterinas previas: Se ha visto este antecedente en el
25% de los casos. En el 29% de los casos la PA estaba implantada en la cicatriz
uterina y solo en el 5% la placenta estaba implantada en otro sitio. (Miller Da, 1997).
Dilatacin y legrado, en el 25% de los casos.
Infeccin uterina previa, remocin manual de la placenta, leiomiomas y otras
anomalas uterinas: La asociacin con estas entidades es inconstante.
Niveles anormalmente elevados de feto-protena y de b-HCG, en el segundo
trimestrre. (Hung TH, 1999).

Una placenta adherente o penetrante no es fcil de diagnosticar antes del


alumbramiento. Despus de ste, se manifiesta como retencin placentaria y sangrado
uterino. El diagnstico generalmente se realiza, despus de intentar la extraccin manual
de la placenta.

Las manifestaciones clnicas propias de la placenta adherente, de la placenta acreta y de la


placenta increta, consisten en una manifiesta dificultad o imposibilidad para la expulsin o
extraccin de la placenta. Como consecuencia de la atona parcial y de la hemostasis
insuficiente en las zonas de despegamiento placentario, se producir una hemorragia ms o
menos grave que en nada se diferenciar de la hemorragia de la atena uterina. Y no ser
solamente al intentar el alumbramiento artificial que se pondran de manifiesto las razones
ntimas de la retencin placentaria; alumbramiento que ser engorroso en. la placenta
ahderente e imposible en las variedades acreta e increta.
En varias ocasiones puede no existir hemorragia y en estos casos la nica manifestacin de
este estado morboso ser la prolongacin del perodo del alumbramiento. La placenta
adherente, como toda placenta retenida, es pronto presa de un proceso infeccioso sin
embargo se han sealado casos de placentas retenidas aspticamente durante muchos
mese3. al cabo de los cuales han sido expulsadas sin causar trastorno alguno; pero hay que
hacer observar que en estas enfermas se ha tratado de retensin de mebranas por abortes
ovulares.

Bibliografa:
1.

Arredondo-Soberon F, Sabella V, Garza-Leal J, Valente PT. Placenta increta en


primer trimestre de embarazo. Ginecol Obstet Mex 1995; 63: 279-81.
2. Cantanzarite V, Stanco L, Schrimmer S et al. Managing placenta
previa/accreta. Contemp Obstet Gynecol 1996; 41: 66-95.

3. Ecker JL, Sorem KA, Soodak L, et al. Placenta Increta Complicating a FirsTrimester Abortion A case report. Journal Reproductive Medicine. 1992; 37-10.
4. Finberg G, William J. Placenta accreta: prospective sonographic diagnosis in
patients with placenta previa / accreta. Contemp Obstet Gynecol 1996: 41: 66-95.
5. Gist RS, Voung V, Brody S, Rees P, Landry AD. Placenta increta occurring in a
bligter ovum. South Med J. 1996; 89(5): 545-7.
6. Harden,MA, Walters MD, Valente PT Postabortal hemorahage due to placenta
increta: A case report. Obstet Gynecol. 1990; 75: 523.
7. Hudon L, Belfort MA, Broome DR. Dosis and management of placenta percreta: A
review. Obster Gynecol survey 1998; 53: 509-517.
8. Hung TH, Shau WY, Hsieh CC, et al. Risk factors for placenta accreta. Obstec
Gynecol 1999; 93: 545-50.
9. Kinoshita T, Ogawa K, Yusumizu T, Kato J. Spontaneous rupture of the uterus due
to placenta percreta at 25-weeks gestation: a case report J Obster Gynaecol Res
1996; 22: 125-8.
10. Kirkinen P, Helin-Martikainen HL, Vanninen R, Patanen K. Placenta accreta: imaging
by gray-scaleand contrast enhanced color Doppler somography and magnetic
resonance imaging. J Clin Ultrasound 1998; 26: 90-4.

23.-Femenino de 22 aos, que presenta una tumoracin de 2 cm. de dimetro en el


cuadrante nfero-externo de la mama izquierda, indolora, de consistencia firme,
superficie lisa, forma ovoidea, mvil y bien delimitada del parnquima vecino, sin antecedentes de derrame por el pezn, sin piel de naranja ni retraccin del pezn, cul
sera su diagnstico de presuncin?

a)
b)
c)
d)

Fibroadenoma.
Carcinoma.
Ectasia de los conductos mamarios.
Quiste solitario.

FIBROADENOMA MAMARIO
Tumor benigno ms frecuente en las mujeres entre los 20 y 35 aos.
ETIOLOGIA
Existen mltiples teoras siendo la ms aceptada la hormonal, generalmente son nicos, solo
el
20% son mltiples o bilaterales. De tamao variable hasta de 10 cm. Ocupa el 13.6% de la
patologa mamaria benigna.
CUADRO CLNICO
Lesin nodular de consistencia dura, de larga evolucin y no dolorosa. Normalmente llegan a
los 3 cm. De dimetro. Durante la fase tarda del ciclo menstrual el tumor suele presentar

un leve aumento de tamao. Durante la menopausia presentan regresin hasta la


calcificacin (signo de palomitas de maz).
DIAGNOSTICO
Es clnico, se presenta como un tumor bien delimitado, desplazable, no adherido a piel ni a
planos profundos, liso o multilobulado en ocasiones. Se localiza frecuentemente en
cuadrantes externos.
EXAMENES DIAGNOSTICOS
ULTRASONIDO MAMARIO.- Identifica un ndulo slido, bien delimitado de bordes
regulares .
TRATAMIENTO.Conservador con vigilancia estrecha dependiendo del tamao y en caso de ser necesario
exresis del ndulo para estudio histopatolgico.
hospitalgeneral.salud.gob.mx/
BIBLIOGRAFIA
1. Snchez BC. Tratado de Enfermedades de la glndula mamaria. Ed. Manual Moderno. Cap.
13- 15.
2.- De Vita V. Cancer of the Breast. In Cancer: Principles and Practice of Oncology: Fifth
Ed. Philadelphia: Lippincott-Raven, Chapter 36; pp: 1521-1616.
3.-Consenso Nacional Acerca del Tratamiento de Cncer de Mama. En Tumores de mama:
Diagnstico y Tratamiento. 2 Ed. McGraw-Hill Interamericana; pp: 119-126.
4.-Eberlein T. Current management of carcinoma of the breast. Ann Surgery 1994; 220:
121-136.
5. Encyclopedie Medico. Chirurgicale Praxis Medica, Editions Techiques de Mexico, tomo 5,
ao 2005.

24.- Femenino de 43 aos, gesta- 4, partos-3, abortos-1, con diagnstico de anemia


ferropnica, de 9.5 g/dl, refiere ciclos menstruales de 31,32 x 8,9 das de duracin,
acompaados de cogulos, los cuales aparecieron despus del nacimiento de su segundo hijo
hace 13 aos. E.F.: Buen estado general, TA 130/80, genitales con evidencia de sangrado
activo, al tacto vaginal se detecta tero de consistencia firme voluminoso, irregular,
aproximadamente de 12 cm. anexos libres.
El diagnstico ms probable es:
a) Adenomiosis uterina.
b) Cncer cervicouterino.
c) Miomatosis uterina.
d) Hiperpalsia adenomatosa de endometrio.

MIOMATOSIS UTERINA
Definicin:
Tumor benigno que se origina en el miometrio, por lo que su componente histolgico
predominante es el tejido muscular y, en menor medida, el conectivo y fibroso. El nico

tratamiento efectivo es el quirrgico; sin embargo, slo requieren ser tratados aquellos que
producen sntomas.
Evaluacin y Diagnstico:
Historia:
1.
2.
3.
4.

El sntoma ms frecuente suele ser la hemorragia uterina.


Los sntomas principales estn relacionados con el crecimiento del tumor.
La paciente puede notar una masa en hipogastrio o abdomen inferior.
La masa se puede asociar a dolor plvico, o manifestaciones por compresin de
rganos o estructuras vecinas.
5. Puede haber alteracin de la fertilidad.
Examen Fsico:
1. Se debe realizar con la vejiga y el recto vacuo.
2. El hallazgo primordial es el aumento de volumen y consistencia del tero, el cual
puede ser simtrico (ndulos submucosos) o irregular (ndulos intramurales o
subserosos).
Exmenes Auxiliares:
1.
2.
3.
4.

5.
6.
7.

Papanicolau crvicovaginal: Indicado siempre; permite descartar neoplasia epitelial


cervical o cncer infiltrante de crvix.
Ultrasonido transabdominal y transvaginal: Indicado siempre; permite evaluar la
localizacin, tamao y nmero aproximado de miomas.
Hemoglobina, hematocrito: Indicado cuando hay historia de sangrado; orienta en la
severidad del sangrado y anemia.
Hemograma y VSG: Indicado cuando hay historia de fiebre; si es anormal sugiere
infeccin o necrosis del mioma (puede ser apropiado descartar infeccin de otro
rgano o sistema).
Grupo sanguneo y factor Rh: Si hay anemia severa o en el preoperatorio.
Perfil de coagulacin (tiempo de protrombina y de tromboplastina parcial, recuento
de plaquetas): Si hay historia de sangrado exagerado.
Gonadotrofina corinica (subunidad srica): Permite descartar posible embarazo
en casos de ciclos irregulares, retraso menstrual o tero de consistencia blanda.

Diagnstico Diferencial:
1.
2.
3.
4.
5.
6.

Embarazo.
Tumor de ovario.
Enfermedad inflamatoria plvica, complejo inflamatorio anexial plvico.
Endometriosis.
Adenomiosis.
Tumor extragenital: colon, retroperitoneo.

Referencias Bibliogrficas:
1.

2.
3.
4.
5.

Hillard PA. Benign Diseases of the Female Reproductive Tract: Symptoms and
Signs. En: Berek JS, Adashi EY, Hillard PA, eds. Novak's Gynecology. Baltimore:
Williams and Wilkins, 1996:331-97.
Hutchins FL, Greenber MD. Miomas Uterinos: Diagnstico e Indicaciones de
Tratamiento. Clinicas de Ginecologa y Obstetricia. Temas Actuales. 1995;5:609-14.
Davis KM, Sclass WD. Tratamiento Mdico para Miomatosis Uterina. Clinicas de
Ginecologa y Obstetricia. Temas Actuales. 1995;5:671-81.
Selwyn P, Oskowitz MB. Leiomyomata Uteri. En: Friedman EA, ed. Gynecological
Decision Making. St. Louis: Mosby, 1983:148-9.
Diaz Huamn V. Tumores Benignos del Aparato Reproductor Femenino. En: Ludmir
A, Cervantes R, Castellano C, eds. Ginecologa y Obstetricia, Prevencin Diagnstico - Tratamiento. Lima: Concytec, 1996:907-25.

25.- Femenino de 23 aos de edad, refiere que desde hace un par de meses ha presentado
hemorragia irregular o postcoital, actualmente presenta disuria y dolor abdominal usted
debe sospechar en cervicitis por:
a) Chlamydia.
b) Micoplasma.
c) Gardnerella.
d) Candida.

Segn los datos de la Organizacin Mundial de la Salud, anualmente se detectan 89 000


000 de nuevas infecciones por Chlamydia trachomatis en el mundo. Esta infeccin provoca
uretritis y cervicitis, y las secuelas incluyen enfermedad inflamatoria plvica, embarazo
ectpico, infertilidad por dao tubrico, epididimitis, proctitis y artritis reactiva. Se
considera principalmente un problema de salud en la mujer, en ella las manifestaciones y
consecuencias son ms dainas para la salud reproductiva. Los individuos infectados con
Chlamydia trachomatis pueden portar el microorganismo por meses o aos y transmitir la
enfermedad a sus parejas sexuales. Su diagnstico sigue siendo un reto, ya que quienes la
padecen presentan sntomas muy leves o son portadores asintomticos.
Manifestaciones clnicas.
La cervicitis es la manifestacin clnica ms frecuente de la infeccin por C. trachomatis en
la mujer. Sin embargo, el 70% de las mujeres infectadas no tienen sntomas, mientras que
en el tercio restante las evidencias clnicas son poco especficas de infeccin, como flujo
genital, dolor abdominal o pelviano, sangrado y/o disuria.

La presencia de disuria puede indicar una uretritis acompaante, lo que sucede en el 35%

de los casos. En otras oportunidades, solo la uretra est comprometida, y la infeccin


uretral se manifiesta como piuria o disuria con cultivo negativo (23% de los casos).
El diagnstico se realiza al examinar el hisopado endocervical, que muestra flujo
amarillento o verdoso con ms de 10 PMN por campo de inmersin en el examen de Gram.
Este resultado define la cervicitis mucopurulenta (CMP) la cual tambin puede ser
producida en casos de infeccin por gonococo o mixta (C. trachomatis y gonococo). Por lo
tanto, el diagnstico debe confirmarse mediante estudios de mayor especificidad, como las
tcnicas moleculares (test de ligasa, PCR), que tienen una sensibilidad del 96%
aproximadamente, o la deteccin del antgeno por tcnica de ELISA, con una sensibilidad
del 75%. Tambin se ha demostrado que las tcnicas moleculares en el primer chorro de
orina son especficas y altamente sensibles.
Cuatro de cada diez mujeres con cervicitis no tratada adquieren enfermedad inflamatoria
pelviana (EPI), con mayor riesgo de sufrir embarazo ectpico, infertilidad y dolor crnico
pelviano. El riesgo de infertilidad se eleva segn el nmero e intensidad de los episodios:
alrededor del 10% despus de un episodio, del 30% despus de dos, y mayor del 50% si ha
habido tres o ms episodios. Por otro lado, el embarazo ectpico es cinco a siete veces ms
frecuente cuando se trata de pacientes con antecedentes de EPI.

Tratamiento

CLAMIDIA

TRACHOMATIS

Azitromicina 1 g
VO dosis nica
Doxiciclina 100 mg VO cada 12 horas por 7 das
Eritromicina 500 mg VO cada 6 horas por 7 das
Ofloxacina 300 mg
VO cada 12 horas por 7das
Levofloxacina 500 mg VO cada 24 horas por 7 das

Bibliografa:
Basado en Guas Clnicas para el manejo de las ITS, OMS, 2003 / Norma Oficial Mexicana
NOM -039-SSA2-2002.

Berek J. (2002) Ginecologa de NOVAK. Mxico. Ed. Mc Graw Hill Interamericana. Pag 293.

26.- Femenino de 33 aos que cursa con 38.5 semanas de gestacin que ingresa al servicio
con trabajo de parto. En el transcurso de trabajo de parto durante la dilatacin presenta
dolor intenso y brusco. A la exploracin usted observa metrorragia escasa y aumento del
tono uterino a la palpacin abdominal que resulta muy doloroso. El diagnstico ms
probable es:
a) Placenta previa.
b) Rotura de vasos previos.
c) Crioamnionitis hemorrgica.
d) Desprendimiento de placenta.

Fisiopatologa de la hemorragia
La hemorragia es el signo fundamental que domina el cuadro clnico de la placenta previa.
Para explicar su mecanismo existen distintas teoras:
Mecanismo de Jacquemier: Se produce crecimiento armnico de la placenta y del tero
hasta la semana 26, 28. Despus el segmento inferior crece ms deprisa y favorece el
despegamiento lo que origina la hemorragia en el embarazo.
Mecanismo de Schroeder: Las contracciones uterinas en el parto traccionan del segmento
inferior hacia arriba y empujan al feto hacia abajo despegando la placenta.
Mecanismo de Pinard: Explica las hemorragias gestacionales y del parto. El estiramiento de
las membranas de la zona de menor radio (orificio interno cervical) como consecuencia de
las contracciones tira de la placenta y la desprenden.
Mecanismo de Bartholomew: Explica la hemorragia en los casos de placenta previa central.
La zona placentaria que reviste el orificio interno es un rea isqumica ya que no recibe
vasos deciduales. A este nivel disminuye la presin sangunea, por lo que la sangre tiende a
dirigirse hacia esta zona y escapa por la cara materna.
En el alumbramiento tambin puede haber una hemorragia importante producida por un
doble mecanismo:
-Desprendimiento parcial antes de la expulsin en los casos de placenta oclusiva.
-Atona uterina en la zona de insercin despus de expulsada la placenta y vascularizacin
anmala.

Manifestaciones clnicas de la placenta previa en el embarazo.


- Sntomas: Principalmente la hemorragia. Toda hemorragia vaginal acontecida en el tercer
trimestre debe hacer pensar en una placenta previa. Las hemorragias suelen ser
espontneas, no acompaadas de dolor, de sangre roja y se presentan de forma
intermitente, con intervalos variables entre las mismas. Progresivamente se van haciendo
ms frecuentes y ms graves. La primera hemorragia suele aparecer en forma inesperada
generalmente nocturna, cesando en menos de media hora. Las hemorragias ulteriores son
ms graves y ms precoces.
- Signos: La consecuencia fundamental es la anemia materna que depende de la cuanta de la
hemorragia (la sangre es de origen materna ya que procede de espacios intervellosos).
- Exploracin:
Exploracin general para valorar la existencia de signos de anemia.
Exploracin obsttrica: valorar el tamao del tero (adecuado para la edad gestacional), es
blando e indoloro. A menudo la esttica fetal est alterada (transverso, oblicuo, nalgas). No
debe efectuarse tacto vaginal cuando haya existido hemorragia en embarazo avanzado por
el riesgo de infeccin y de despegamiento y aumentar as la hemorragia.
Auscultacin fetal normal.
Exploracin ecogrfica: es una tcnica fundamental en el diagnstico de la placenta previa.
Permite determinar la localizacin placentaria y la variedad de la placenta previa. En
general, la placenta puede identificarse a partir de la 9 semana. No obstante, a lo largo de

la gestacin por crecimiento uterino se produce un cambio en sus relaciones con el tero
("emigracin placentaria", imagen de desplazamiento). As el diagnstico de certeza de
lmites placentarios solo puede establecerse hacia la semana 34. Siempre ser necesario
hacer una adecuada identificacin del orificio cervical interno (ms fcil con sonda
transvaginal). Aadiendo al estudio Doppler color se observa la vascularizacin y las zonas
que sangran.

27.- Gestante de 9 semanas la cual tiene contacto con un menor que, 6 das ms tarde,
desarrolla un cuadro de exantema y sndrome general infeccioso sugerente de infeccin por
virus de rubola. En el primer control serolgico gestacional se detect la negatividad de la
IgG especfica. Cul de las siguientes afirmaciones es la correcta?

a) La aparicin de IgM materna positiva constituye indicacin para la determinacin de


la IgM fetal.
b) No existe posibilidad de contagio dado que el nio ya no se hallaba en fase de eliminacin
viral.
c) En este perodo de la gestacin, el riesgo de la infeccin y de afectacin embrionaria es
mnimo.
d) La existencia de IgM fetal negativa excluye en este caso la posibilidad de transmisin
transplacentaria.

INFECCIONES EN EMBARAZADA
VACUNACIONES:
CONTRAINDICADAS:
- Parotiditis
- Rubola
- Sarampin
- Fiebre amarilla
- NO se RECOMIENDA: gripe, poliomielitis, hepatitis B.
S SE PUEDEN DAR (cuando estn indicadas):
- Fiebre tifoidea
- Rabia
- Ttanos
- Tos ferina (ver pregunta mir, que pone que no)
TOXOPLASMOSIS:
50% de transmisin. Si grave (en 1er T, pero es menos frecuente): aborto, parto
pretrmino, muerte fetal intratero.
Dx: lo de siempre. Calcificaciones cerebrales.
TTO:
- En casos de seroconversin ESPIRAMICINA (depresin medular, ac.
folnico) ht final del embarazo.

Si infeccin fetal: PIRIMETAMINA Y SULFADIAZINA en ciclos de 3


semanas alternando con la espiramicina ht final del embarazo.

RUBEOLA:
Contagio >80% si la madre se contagia en 8 primeras semanas. SORDERA
CONGNITA.
Acs <1/16 susceptibilidad para infeccin. La aparicin de IgM materna positiva es
indicacin de determinacin de IgM fetal.
La madre no debe quedarse embarazada en los 3 meses siguientes a la vacunacin.
Profilaxis de EXPOSICIN: gammaglobulina, solo eficaz en primeros 7-8d.
CITOMEGALOVIRUS:
Insospechada. Inclusiones citomeglicas en OJO DE BHO en clas de tejidos
afectados.
SFILIS:
Prueba sexolgica a todas las gestantes. Si se infecta, en los 3 meses neonatales:
lesiones CUTNEOMUCOSAS, OSTEOCONDRITIS Y HEPATOESPLENOMEGALIA.
DX: screening VDRL o RPR (no treponmicas, pero el embarazo es la primera
causa de falso positivo)
certeza FTA-Abs o MHA-TP (treponmicas)
VARICELA:
Si en 1er T 2%; gravemente teratognica. En perodos ms avanzados o
periparto enfermedad sistmica generalizada o SNC.

HEPATITIS B:
Se infecta el hijo si:
- Madre PORTADORA CRNICA.
- INFECCIN ACTIVA durante la gestacin.
- Madre con HEPATITIS CRNICA ACTIVA.
HBsAg positivo (slo) riesgo bajo de transmisin placentaria.
HBsAg + HBeAg 90% de transmisin.
*El riesgo de cronificacin ser muy alto si se adquiere en el perodo perinatal.
*Especial vigilancia del crecimiento fetal.
*Se realizar profilaxis ACTIVA y PASIVA.
*La gestacin NO aumenta el riesgo de curso clnico grave.

RUBOLA Y EMBARAZO
Existen tres situaciones claramente diferenciadas que exigen planteamientos diagnsticos
distintos:
Determinacin de la inmunidad frente a rubola en la gestante, sin sospecha clnica ni
epidemiolgica de padecer la enfermedad: El objetivo de este estudio es conocer si la
gestante est protegida, de una posible infeccin por el virus de la rubola, durante el

embarazo. Se recomienda la determinacin cualitativa de anticuerpos totales o de IgG


especfica, en la primera consulta de control del embarazo. Se desaconseja expresamente
la evaluacin cuantitativa de los resultados, ya que no proporciona ninguna informacin til.
La presencia de anticuerpos refleja contacto previo con el virus, y por tanto inmunidad,
haciendo innecesaria la realizacin de nuevos controles en embarazos sucesivos.
A pesar de que se describe que la rubola puede cursar de manera asintomtica, F. de Ory
et al estudian 185 sueros de 101 mujeres embarazadas con presencia de IgM y slo
confirman la existencia de primoinfeccin en tres de las mismas, asocindose siempre a
datos clnicos o epidemiolgicos compatibles, por lo que es desaconsejable la realizacin
sistemtica de IgM a las embarazadas.
Si la mujer embarazada es seronegativa, deber adoptar las precauciones necesarias para
evitar la exposicin al virus y debe ser vacunada frente a la rubola en el post-parto
inmediato.
Sospecha clnica de infeccin aguda durante el embarazo: Este caso puede plantearse
ante la existencia de una clnica compatible en la embarazada, o por exposicin a un sujeto
con infeccin aguda por rubola.
La presencia de IgG en ausencia de IgM indica que la mujer est protegida, por vacunacin
o por infeccin antigua y por tanto no deben realizarse ms determinaciones.
La demostracin de seroconversin, con ausencia de anticuerpos en el primer suero y
presencia de stos en el segundo, obtenido 15-21 das despus, es la forma ms segura de
diagnosticar una primoinfeccin por este agente. Sin embargo, si el primer suero de la

enferma presenta anticuerpos, aunque se produzca un incremento del ttulo de estos en el


segundo suero, puede ser debido a una reinfeccin.
La presencia de IgG y de IgM especfica en una paciente, nos hace sospechar la presencia
de primoinfeccin, sin embargo, debemos tener en cuenta varios aspectos:
a.- La IgM puede tener reacciones heterlogas entre rubola y otros virus como EBV, CMV,
Parvovirus B 19 y virus del sarampin (por reacciones cruzadas o por estimulacin policlonal
de linfocitos de memoria); por tanto es necesario confirmar su presencia, siendo la tcnica
de ELISA de captura la que presenta mejor especificidad y sensibilidad.
b.- La IgM puede aparecer durante las reinfecciones, pero a ttulos bajos y durante poco
tiempo.
c.- En un pequeo porcentaje de personas, la IgM puede mantenerse positiva en suero hasta
6 meses. Thomas et al, detectan la presencia de IgM en el 9% de los casos a los 3 4
meses de la infeccin aguda.

El estudio de la avidez de la IgG diferencia si la IgG es de aparicin reciente (baja avidez


se asocia a infeccin primaria aguda) o si hay ausencia de infeccin primaria (IgG de alta
avidez), puede ser una tcnica que ayude a valorar la presencia de IgM y puede colaborar
en la diferenciacin entre primoinfeccin y reinfeccin. F. de Ory et al. estudian mltiples
patgenos y comunican que esta tcnica presenta una sensibilidad entre el 81 y el 100% y
una especificidad del 100%. Tambin se est valorando la utilidad en el diagnstico de la
IgA, aunque los datos no son an concluyentes.
Todos estos datos serolgicos deben ser interpretados junto con los datos clnicos de la
embarazada, en el caso de que los haya y junto con los datos que podamos obtener de la
posible fuente de infeccin.
Bibliografa:
1.-BOSMA TJ, CORBETT KM, OSHEA S, BANATVALA JE, BEST JM. PCR detection of
rubella virus in clinical samples. J Clin Microbiol 1995; 33:1075-1079.
2.-BOSMA TJ, CORBETT KM, OSHEA S et al. Use of PCR for prenatal and postnatal
diagnosis of congenital rubella. J Clin Microbiol 1995; 33:2881-2887.
3.-DE ORY F, CASAS I, DOMINGO CJ, ECHEVARRA JM. Application of
fluoroimmunoassay to the identification of low avidity specific IgG against pathogenic
human viruses and Toxoplasma gondii. Clin Diagn Virol 1995; 3:323-332.
4.-DE ORY F, DOMINGO CJ. Los anlisis de avidez de la IgG especfica en el diagnstico
de la infeccin por el virus de la rubola. Med Clin (Barc) 1996; 107:118.
5.-DE ORY F, ECHEVERRA JM, DOMINGO CJ. Cribado rutinario de IgM especfica
antirrubola en mujeres embarazadas: una prctica desaconsejable. Prog Obstr Ginec. Dic
1998; 41:574-578.
6.-ENGLUND J, GLEZEN WP, PIEDRA PA. Maternal immunization against viral disease.
Vaccine 1998; 16:1456-1463.
7.-FREY TK, ABERNATHY ES, BOSMA TJ et al. Molecular analysis of rubella virus
epidemiology across three continents: North America, Europe and Asia, 1961-1997. J
Infect Dis 1998; 178:642-650.

28.- Ante una paciente de 35 aos que acude a consulta con antecedentes de G3 C2 A1,
refiere que ha presentado durante el primero y segundo trimestres de su embarazo
manchado con frecuencia intermitente ,a las 34 SDG inicia con hemorragia abundante,
repentina e indolora, su principal sospecha es:
a) Coriocarcinoma.
b) Ruptura uterina.
c) Placenta previa.
d) Desprendimiento grave de placenta normoinserta.

PLACENTA PREVIA
DEFINICIN:
Es cuando la placenta se implanta sobre o muy cerca del orificio cervical interno y una
parte de la placenta precede a la parte fetal que se presenta.
INCIDENCIA:
Esta es difcil determinar ya que muchos casos pasan desapercibidos, sobre todo cuando
ocurren los abortos en embarazos tempranos.
La prevalencia vara de 1 en 100 a 1 en 850 nacidos vivos, pero solo el 20% total.
Etiologa:
Edad avanzada, multparas, paciente con cesreas previas, paciente con aborto de
repeticin, esto debido a las gestaciones previas.
TIPOS:
Insercin baja.- Es cuando el borde placentario se encuentra en el segmento inferior a
menos de 6 cm del orifico cervical interno.
Marginal.- Es cuando el borde placentario alcanza los mrgenes del orificio cervical interno.
Parcial.- Es esta la placenta cubre parcialmente el orificio cervical interno.
Total.- La placenta cubre la totalidad del orificio cervical interno an con dilatacin
cervical avanzada.
DIAGNOSTICO:
La caracterstica es el STV de aparicin brusca en forma indolora en el segundo o tercer
trimestre. Frecuentemente hay ausencia de dolor a actividad uterina que son parmetros
para hacer el diagnstico.
La mayor incidencia de sangrado aparece a las 33-34 sdg.

ESTUDIOS DE GABINETE:
El estudio mas utilizado es la ultrasonografa obsttrica.
TRATAMIENTO:
El manejo va a depender de factores como son:
Edad gestacional, magnitud del sangrado, si hay trabajo de parto, variedad de placenta
previa y complicaciones materna.

BIBLIOGRAFA:
1.- Waxler P, Gottesfeld KR. Early diagnosis of placenta previa. Obstet Gynecol
1979;54:231-32.
2.- Cabrero-Roura L. Riesgo elevado obsttrico. Ed. Masson 1996; pp; 109-118.
3.- Patrick J, Placenta Previa, Clinical Obst and Gynecology 1990;33(3): 414-421.
4.- Chapman M, Furtenes ET, Significance of ultrasound in location of placenta in early
pregnancy Br J Obst Gynecol 197;86: 846. 57.

29.- Femenino de 30 aos con sospecha diagnstica de endometriosis, el sntoma


caracterstico de esta patologa es:
a) Dispareunia.

b) Dismenorrea.
c) Metrorragia.
d) Leucorrea.

ENDOMETRIOSIS:
Es relativamente frecuente en personas jvenes, 25-30 aos. Se define como la
localizacin de la mucosa endometrial en un lugar ectpico.
La localizacin normal de endometrio es el tero. La mucosa endometrial sufre una serie de
cambios durante el ciclo. Cuando la mucosa endometrial penetra en el msculo uterino
(miometrio) se denomina: Adenomiosis.
Patogenia:
No se conoce realmente por qu se produce y tampoco existe un tratamiento definitivo
para ella.

Teoras: a) Teora Metaplsica celmica (T. De Meyer): El celoma es un epitelio que recubre
la cavidad abdominal (metaplasia).

b) Teora Transplantativa (Sampson): La mucosa endometrial a travs de las trompas,


cuando se produce una menstruacin si el orifico cervical est cerrado, la sangre refluye
(por mecanismo retrgrado), y va a cavidad abdominal donde se producen implantes de
clulas endometriales (en fondos de saco de Douglas, peritoneo...). Tambin por
intervenciones quirrgicas. Esta teora es la que parece ms veraz.
c) Teora embrionaria: Es poco importante. Dice que la endometriosis se produce por los
restos embrionarios de Wolf o Mller.
d) Teora inmunolgica: Suele influir sobretodo en las personas estriles, con disminucin
de la respuesta inmunitaria por: - Disminucin de la citotoxicidad de las clulas NK// Aumento de resistencia de las clulas endometriales a la destruccin// - Aumento de la
actividad de los Macrfagos.
Clnica:

Disfuncin menstrual: Aparicin de dismenorrea progresiva, que no aparece desde


el principio (es tarda), es intermenstrual y aumenta al final ms intensa. Va
aumentando con los aos (al contrario que la Dismenorrea normal).

Exploracin: Retraccin de los ligamentos uterosacros, nodulaciones, rugosidades y


dolorosos al tacto. Dispareunia (dolor en las relaciones sexuales). Dolor en la
defecacin (porque la reaccin inflamatoria produce adherencias y dolor en la
movilizacin del tero). Dismenorrea.

Disfuncin ovrica: Se hace quistectoma, son ms conservadores en el tratamiento


quirrgico. Insuficiencia ltea. Hiperprolactinemia. Amenorrea// No son muy
frecuentes ni especficas puede que no se den las tres.

Disfuncin reproductiva: Es causa de esterilidad (estadios III y IV de la


clasificacin americana: Reaccin inflamatoria que afecta a anejos, recto, sigma,
apndice, ureter...)// Se producen abortos de repeticin e insuficiencia ltea.

Bibliografa
1.
2.
3.

4.

5.

6.
7.

Stenchever A. Comprehensive Gynecology. 4th ed. St. Louis, Mo: Mosby;


2001:1065-1070.
Noble J. Textbook of Primary Care Medicine. 3rd ed. St. Louis, Mo: Mosby;
2001:325.
Chen C, Cho S, Damoskosh AI. Prospective study of exposure to environmental
tobacco smoke and dysmenorrhea. Environ Health Perspect 2000; 108(11): 10191022.
Wilson ML, Farquhar CM, Sinclair OJ. Surgical interruption of pelvic nerve
Pathways for primary and secondary dysmenorrhea. Cochrane Database Syst Rev
2000; (2): CD001896.
Morrison B, Daniel S, Kotey P, et al. Rofecoxib, a specific cyclooxygenase-2
inhibitor, in primary dysmenorrhea: A Randomized controlled trial. Obstet Gynecol
1999; 94(4): 504-508.
Schroeder B, Sanfilippo J. Dysmenorrhea and pelvic pain in adolescents. Pediatrics
Clinics of North America. 1999; 46 (3): 555-571.
Dawood MY. Dismenorrea. Clnicas Obsttricas y Ginecolgicas. 1990; 1: 167-176.

30.- Se trata de femenino de 33 aos con antecedentes patolgicos de hipertensin


arterial crnica bien controlada tratada con IECAS , actualmente cursa con 7 semanas de
gestacin , signos vitales dentro del parmetro normal y exmenes de laboratorio sin
alteraciones, se refiere asintomtica , Cul sera la conducta a seguir?

a) Suspender los IECAs dado el riesgo que presentan para el feto.


b) Mantener el tratamiento y asociar alfametildopa para disminuir los riesgos fetales de
los IECAs.
c) Mantener el tratamiento y asociar hidralacina para disminuir los riesgos maternos de los
IECAs.
d) Mantener el tratamiento dado el buen control tensional.

El uso de IECA y ARAII durante el segundo y tercer trimestre de embarazo est


contraindicado, debido a que estos medicamentos inducen toxicidad fetal (descenso de la
funcin renal, oligohidramnios, retraso en la osificacin del crneo) y toxicidad neonatal
(insufi ciencia renal, hipotensin, hiperpotasemia).

En cuanto a su uso durante el primer trimestre de embarazo, un estudio publicado en el


ao 20061 mostraba un incremento de la incidencia de malformaciones congnitas, en
particular malformaciones cardiacas, en nios nacidos de madres expuestas a IECA
durante el primer trimestre de embarazo en comparacin con las mujeres que no
recibieron tratamiento antihipertensivo o que recibieron tratamiento con otros
medicamentos antihipertensivos. Estudios posteriores realizados no han confirmado a da
de hoy los resultados de este estudio. En lo referente a los ARAII, no se dispone de

estudios epidemiolgicos analticos apropiados, por lo que no se puede descartar que exista
el mismo riesgo que para los IECA.
A pesar de estas incertidumbres, el Comit de Medicamentos de Uso Humano (CHMP) de la
Agencia
Europea de Medicamentos (EMEA) ha recomendado prudencialmente evitar el uso de IECA

y ARAII durante el primer trimestre del embarazo.

Bibliografa:
Cooper WO et al. Major congenital malformations after fi rst-trimester exposure to ACE
inhibitors. N Engl J Med 2006; 354 (23): 243- 51.
(ref.: 2008/10, junio

31.- Femenino de 23 aos, G1 en trabajo de parto prematuro con embarazo de 30 semanas


de gestacin. A pesar del uso de agentes tocolticos, estos no han dado resultado. Se puede
inducir la maduracin pulmonar del producto por medio de:
a) Betametasona.
b) Sulfato de magnesio.
c) Hidroxiprogesterona.
d) Clorprocana.

La utilizacin de betametasona como inductor de madurez pulmonar fetal (IMPF) disminuye


la morbilidad neonatal relacionada con prematurez pero su efecto diabetgeno materno ha
sido poco estudiado.
La revisin Cochrane de un ciclo nico de corticosteroides se actualiz en 2006. En esta
actualizacin se incluyeron 21 estudios con un total de 3885 mujeres y 4269 lactantes.
En la revisin se descubri que la administracin de determinados corticosteroides a
mujeres con riesgo de tener un parto prematuro reduce considerable los riesgos de
complicaciones relacionadas con la prematurez como muerte fetal y neonatal combinada,

sndrome de dificultad respiratoria, hemorragia cerebroventricular, enterocolitis


necrotizante, infecciones sistmicas y retraso en el desarrollo durante la niez. Los
beneficios estaban presentes cuando el tratamiento se iniciaba entre las 26 y las 35
semanas de gestacin y en los nios que nacan entre 1 y 7 das despus de haber
comenzado el tratamiento; tambin se observaron beneficios en los subgrupos de mujeres
con rotura prematura de membranas y trastornos hipertensivos. La muerte fetal y neonatal

combinada se redujo incluso en neonatos que nacieron a menos de las 24 horas de haber
administrado la primera dosis.
No se demostraron beneficios cuando el tratamiento comenz antes de las 26 semanas de
gestacin, tampoco se observaron beneficios en los recin nacidos antes de las 26 semanas
de gestacin ni en los que nacieron despus de 7 das o ms de la administracin del
tratamiento.
En el caso de los neonatos que nacieron despus de las 36 semanas hubo una tendencia a
aumentar la muerte fetal y neonatal combinada.
Se observ una reduccin en el peso al nacer en los neonatos que nacieron entre los das 1 y
7, al igual que en los que nacieron ms de 7 das despus del primer tratamiento.
Un estudio que reclut mujeres con preeclampsia severa sugiri que las mujeres tratadas
tenan un mayor riesgo de sufrir diabetes gestacional.
La evidencia epidemiolgica y en animales sugiere que pueden haber efectos adversos a
largo plazo por la exposicin prenatal a los corticosteroides, entre ellos la alteracin de la
tolerancia a la glucosa y la hipertensin. Los estudios en animales tambin han sugerido que
afecta el crecimiento del cerebro.
1.
2.

3.

4.

5.
6.

7.

National Institute of Health (NIH). Consensus Conference; Effect of corticosteroide for fetal maturation on perinatal outcomes. JAMA 1994;(12):1-19.
White A, Marcucci G, Andrews E, Edwards K. Antenatal steroids and neonatal
outcomes in controlled clinical trials of surfactant replacement. Am J Obstet
Gynecol 1995; (173):286-90.
Klauss MH, Fanaroff AA, Martin RJ. Problemas respiratorios. En: Asistencia del
recin nacido de alto riesgo. 2 ed. La Habana: Editorial Cientfico-Tcnica,
1981:194.
Avery M, Frank N, Gribetz I. The inflationary force produced by pulmonary
vascular distention in excised lungs. The possible relation of this force to that
needed to inflatc the lungs at birth. J Clin Invest 1959;38:456.
Chu J, Clements J, Cotton E. Neonatal pulmonary ischemia. Pediatrics 1965;40:733.
Liggins GC, Howle RN. A controlled trial of antepartum glucocorticoid treatment
for prevention of respiratory distress syndrome in premature infants. Pediatrics
1972;50: 515-25.
Wright LL, Verter J, Younes N. Antenatal corticosteroids administration and
neonatal outcome in infants 501 to 1500 g. Am J Obstet Gynecol 1995; (173):263.

32.- Femenino de 31 aos se envia de alta con diagnstico de enfermedad inflamatoria


plvica, regresa a los 15 das con temperatura de 38.5 c, mal estado general y datos de
irritacin peritoneal, El diagnstico ms probable es?

a) Endometritis.
b) Hidrosalpinx.
c) Absceso tubo-ovrico.
d) Ooforitis aguda.

La enfermedad inflamatoria plvica (EIPA) es un sndrome clnico caracterizado por la


infeccin del tracto genital superior que se produce casi siempre por va ascendente desde
el cuello uterino. El impacto que la infeccin plvica ejerce sobre la condicin fsica de la
mujer va desde la infeccin asintomtica o silente a una mayor morbilidad que en algunos
casos puede llegar hasta la muerte. Incluye una variedad de condiciones inflamatorias que
afectan el tracto genital superior. Los Centros de Control de Enfermedades (C .D. E.) la
definen como un sndrome agudo debido al ascenso de microorganismos de la vagina o el
cuello uterino al endometrio, trompas uterinas y en ocasiones a las estructuras vecinas
(ovarios, peritoneo y cavidad pelvianas).
En el momento actual se incluyen como principales agentes etiolgicos de la E.I.P.A la
Neisseria gonorrhedae, las clamydias y los anaerobios. Otros microorganismos como los
microplasmas y los actinomices se estn observando con frecuencia.
La presencia de anaerobios as como de bacterias aerobias puede deberse a un fenmeno
de sobre infeccin secundaria. Hay autores que sealan que excepto para el gonococo y la
Clamydia trachormatis, no existen datos suficientes que permitan afirmar que otras
bacterias tengan un papel primario en la infeccin de unas trompas sanas. Una vez alterada
la integridad anatmica de la trompa, se producira la infeccin mixta o poli microbiana.

El absceso tubo ovrico es una formacin inflamatoria que compromete el ovario y la


trompa y puede ser uni
o bilateral. En este
absceso
las
estructuras
comprometidas estn infectadas y contienen pus. Este proceso inflamatorio es
secundario a un proceso infeccioso de la pelvis, habitualmente producido por
grmenes muy patgenos, que llegan al tracto genital a travs de una relacin sexual, es
decir corresponde a una complicacin severa de una enfermedad de transmisin sexual.
Se caracteriza por aumento de volumen del ovario y trompa, los que se encuentran
adheridos entre s producto de esta infeccin, adems el proceso infeccioso se extiende
habitualmente a otras estructuras y rganos pelvianos, los que estn muy inflamados y
adheridos formando lo que se denomina plastrn.
El tratamiento se inicia mdicamente con antibiticos de amplio espectro para cubrir
tanto grmenes aerbicos como anaerbicos, generalmente requiere de hospitalizacin

para iniciar una terapia agresiva endovenosa con los antibiticos y para monitorizar
adecuadamente a la paciente, pues la infeccin produce compromiso del estado
general pudiendo llegar hasta la sepsis generalizada.
Diagnostico
El cuadro clnico se sospecha cunado una paciente consulta por dolor abdominal intenso,
progresivo, fiebre y compromiso de su estado general, habitualmente en el examen se
encuentra un distensin abdominal y a la palpacin del abdomen hay dolor, y signo de
blumberg positivo o irritacin peritoneal. El Tacto vaginal demuestra fondos de saco
vaginales abombados y dolorosos y habitualmente el cuello del tero lateralizado y
doloroso a la movilizacin si el compromiso es unilateral, adems de palpar una masa para
uterina irregular y sensible.

BIBLIOGRAFA.
1.
2.
3.
4.
5.
6.
7.
8.

Botella Llusi,J.Clavero Nez,J.A:Tratado de Ginecologa.14 edicin.Ed.Diaz de


Santos.pg 833-844.Madrid,1993.
Brunham,R,C:Infectionin woman and ectopic pregnancy. Am J Obstet
Gynecol.67:722,1999.
Cates,W,Wasserheit,J,N:Genital Infection Epidemiology and sequeale.Am J Obstet
Gynecol 164-1771,1998.
Keit,L,G; Berger,G,S:On the causation of pelvic inflammatory disease. Am J Obstet
Gynecol 149-215,2002.
Muller,B,R;Allen,J,et al.Pelvic Inflamatory disease after histerosalpingography.Brit
J Obstet Gynecol,91-1181,1999.
Toth,A,O Leary,W,M: Evidence of microbial transfer by espermatozoo.Am J O
bstet Gynecol 59-556,2003.
Varela,R,et col:Abceso Tuboovrico,Acta mdica Portuguesa ,p:537-542,Vol.
8,2001.
Sopper,D,E:Pelvic Inflamatory disease.Infections disease.Clin of North America
.831-840,vol 8;n 4.Dec 2003.

33.- Se trata de femenino de 19 aos con menarca a los 12 aos ritmo menstrual 45x4.
Refiere vida sexual activa desde los 17 aos con frecuencia de 4 veces por semana. A la
exploracin se encuentra acn intenso en la frente, mejillas y mentn. Acude a consulta
por que desea adoptar un tratamiento anticonceptivo por va oral, lo mas adecuado es:
a) Norgestimato.
b) Ciproterona.
c) Gestodeno.
d) Levonorgestrel.
El efecto antiandrognico especfico del acetato de ciproterona acta por inhibicin
competitiva de la unin de la 5 - alfa - dihidrotestosterona con el receptor citoslico de las
clulas blanco, que disminuye la produccin y la excrecin de sebo y el aumento y el
desarrollo del vello.
Es un derivado de la 17 - alfa - hidroxiprogesterona que posee accin progestgena. Su
accin antigonadotrfica se suma a la del etinilestradiol. El acetato de ciproterona no
posee accin estrognica sino un efecto antiestrognico, y tampoco posee accin nociva
sobre la funcin de la corteza suprarrenal.

Indicaciones en la mujer: Manifestaciones de androgenizacin de grado severo, por


ejemplo, hirsutismo grave, alopecia androgentica de tipo grave, a menudo acompaados por
manifestaciones graves de acn y/o seborrea.
Indicaciones en el hombre: Atenuacin del impulso en las desviaciones sexuales.
Tratamiento antiandrgeno del carcinoma de prstata inoperable.
BIBLIOGRAFA
1. Swift S. Current opinion on the classification and definition of genital tract prolapse.
Curr Opin Obstet
Gynecol 2002; 14: 503-7.
2. De Caro R, Aragona F, Herms A, Guidolin D, Bizzi E, Pagano F. Morphometric analysis of
the fibroadipose tissue of the female pelvis. J Urol 1998; 160: 707-13.
3. Gill E, Hurt W. Pathophysiology of pelvic organ prolapse. Clin Obstet Gynecol 1998;
25(4): 757-69.
4. DeLancey, J. Anatomic aspects of vaginal eversion after Hysterectomy. Am J Obstet
Gynecol. 1992;
166(6 pt 1): 1717-24.

34.- Paciente femenino de 27 aos de edad con deseo de un embarazo, antecedentes de


G3 A2 - P1 se le realiza una histerosalpingografa, se constata que existe un sndrome de
Asherman. Ello significa que se trata de:

a)
b)
c)
d)

Sinequias uterinas.
tero bicorne.
Endometriosis en la trompa.
Insuficiencia istmico cervical.

El sndrome de Asherman es una enfermedad ginecolgica rara que se caracteriza por la


presencia de sinequias (adherencias) intrauterinas que pueden ocasionar amenorrea
(ausencia de perodos menstruales regulares) e infertilidad.
En 1894 Heinrich Fritsch describe por primera vez la presencia de sinequias intrauterinas
de tipo postraumtico, en una paciente que desarroll una amenorrea secundaria a un
curetaje. Posteriormente en 1927 Bass inform de veinte casos de atresia (oclusin de una
abertura natural) cervical tras abortos inducidos, pero no fue hasta 1948, cuando Joseph G.
Asherman recopil la informacin hasta entonces existente y acu el nombre con el que se
conoce actualmente a la enfermedad.
Asherman describi originalmente dos tipos diferentes de amenorrea secundaria, en
funcin de su etiologa (estudio de las causas de las enfermedades): la amenorrea
traumtica atrtica, debida a estenosis del orificio cervical interno y la amenorrea debida
a adherencias intrauterinas. Posteriormente ambas entidades se agruparon en una nica
entidad bajo el nombre de sndrome de Asherman.
Suele presentarse en mayor proporcin tras dilataciones y curetajes uterinos de repeticin
y sobre todo si se realizan durante el embarazo o si existe infeccin uterina en el momento
en el que se realizan estas intervenciones.
Las adherencias intrauterinas pueden producirse debido a cualquier factor que lleve a una
destruccin de las paredes del miometrio (capa muscular de la pared del tero). Sin
embargo, hay que distinguir entre factores predisponentes, siendo el principal de ellos el
embarazo y factores causales, entre los que se encuentran: traumatismos uterinos,
intervenciones quirrgicas que afecten al tero, agentes fsicos o qumicos e infecciones
uterinas por tuberculosis o esquistosomiasis. En cualquier caso, el factor ms importante es
el trauma uterino en el momento del parto o el puerperio.

El cuadro clnico es muy variable y las manifestaciones clnicas varan con el grado de
oclusin de la cavidad uterina y la severidad de las adherencias, pudiendo presentarse:
esterilidad cuando la oclusin de la cavidad uterina incluye porciones proximales (ms cerca
de un centro, tronco o lnea media) de las trompas de Falopio o cuando las adherencias
impiden la nidacin del huevo; las pacientes presentan con frecuencia amenorrea,
oligomenorrea (disminucin de la frecuencia de las menstruaciones), dismenorrea
(menstruacin dolorosa) y abortos repetidos.
Bibliografa:
1.

Hysteroscopic treatment of severe Asherman's syndrome and subsequent fertility.


Capella-Allouc S; Hum Reprod, 1999 May.

35.- En una consulta prenatal de rutina, una mujer de 28 aos de edad, G5 P4, con 28 SDG,
refiere que no ha sentido movimiento fetal durante los ltimos 2 das. Su embarazo ha sido
complicado debido a que padece hipertensin crnica, para lo cual se le recetaron tabletas
de alfa-metildopa 2 veces al da. Al examen, su FU es de 30cm, y las maniobras de Leopold
demuestran que el feto se encuentra en situacin transversa. Su TA es 145/85mmHg. No
se encuentra latido cardiaco con el Doppler. Cul de los siguientes es el paso ms
apropiado a seguir en el manejo?

a)
b)
c)
d)

Realizar un test sin estrs.


Amniocentesis.
USG.
Beta-HCG (cuantitativa)

Probable bito:
Sintomatologa y diagnstico Signos funcionales: No se perciben movimientos fetales por
12-24 horas. Disminucin o ausencia de sntomas y/o signos como nauseas vmito,
hipertensin, albuminuria) Paraclnicos: *ecografa: diagnstico precoz y exacto: Doppler.
*Radiologa: hay 3 signos: +deformacin del crneo +curvatura y torsin de la columna
+presencia de gas en el feto *lquido amnitico: puede estar meconiado, o sanguinolento
Signos locales: en los senos hay secrecin calostral, sangrado leve y oscuro por vagina, el
feto se vuelve blando a la palpacin, fetocardia (-), puede haber detencin y/o disminucin
de la altura uterina, bajo peso corporal, entre otros

Bibliografa:
1. OBSTETRICIA, Schwarcz R, editorial El ateneo, 2003. 2. Sociedad espaola de
ginecologa y obstetricia, junio 2002. 3. OBSTETRICIA CLNICA, Llaca V, edicin 2000,
captulo 24; Pg, 315-316.

36.- Cual de los marcadores sricos para diagnosticar Infarto Agudo al Miocardio
traduce lesin celular (necrosis)?
a)
b)
c)
d)

Creatinfosfokinasa.
CPK-MB.
De4shidrogenasa lctica.
Troponina I.

Las troponinas cardiacas son protenas que forman parte de los mecanismos de regulacin
de la contraccin del msculo cardiaco, estn presentes en las fibras miocrdicas.
La troponina es una protena globular de gran tamao, contiene tres subunidades
polipeptdicas: troponina C (fijadora de calcio), troponina I (inhibidora de la interaccin
actina-miosina) y troponina T (fijadora de tropomiosina).
Cuando se necrozan las clulas del tejido miocrdico pierden la integridad de la membrana
celular y las molculas intracelulares difunden hacia la microcirculacin y a los linfticos.
Estas macromolculas se detectan en la circulacin perifrica y constituyen los marcadores
bioqumicos especficos de dao al miocardio.

Bibliografa:
-Owen Avril. Cardiac troponins: improved diagnosis and cost benefits. 2001. Clinical
Laboratory International. Volume 25. No. 8. P.14-15.

37.- Cul de las siguientes vacunas son conjugadas?

a) Pneumococo de 7 serotipos, meningococo de cuatro serotipos y Haemophilus


influenzae tipo B.
b) Sabn, triple viral y varicela.
c) Influenza, hepatitis A y hepatitis B.
d) BCG, pneumococo polivalente y DPT.

Conjugadas de Haemophilus influenzae tipo b (Hib)


Las vacunas conjugadas de Hib unen covalentemente el polisacrido de Hib con una protena
acarreadora, que induce una respuesta inmunolgica dependiente de clulas T y memoria
inmunolgica que no se induce con el polisacrido solo en la vacuna. Las protenas
acarreadoras autorizadas para infantes incluyen el toxoide tetnico (vacuna PRP-T); un
mutante, toxina diftrica no txica (HbOC); y la protena de membrana externa de
Neisseria meningitidis (PRP-OMP). Una cuarta vacuna (PRP-D), conjugada al toxoide
diftrico, est autorizada slo para uso en nios mayores (15 meses de edad o mayores)
debido a su inmunogenicidad ms baja en infantes.

Bibliografa:
1.

Ammann AJ, Ashman RF, Buckley RH et al. Use of intravenous gammaglobulin in


antibody immunodeficiency: Results of a multicenter controlled trial. Clin Immunol
Immunopathol 1982; 22:60-7.

2. Roifman CM, Lederman HM, Lavis S, Stein LD, Levison H, Gelfand EW. Benefit of
intravenous IGG replacement in hypogammaglobulinemic patients with chronic
sinopulmonary disease. Am J Med 1985; 79:171-4.
3. American Academy of Pediatrics, 1994, Red Book: Report of the Committee on
infectious Diseases, 23rd ed. Elk Grove Village, IL, 1994.

38.- Masculino de 47 aos con datos positivos de alcoholismo crnico, al cual se le


diagnstica pancreatitis aguda Cul es la primera medida teraputica a adoptar?
a) Iniciar antibiticos de amplio espectro.
b) Administracin de inhibidores de la bomba de protones.
c) Dieta absoluta.
d) Aspiracin nasogstrica.

El 80 % de los pacientes con pancreatitis aguda se tratan mediante medidas de


sostn, suspensin de la va oral, hidratacin intravenosa, analgsicos y alimentacin
parenteral cuando la va oral se restringe ms all de la semana.

La indicacin de la suspensin de la va oral se basa en el dolor y la intolerancia digestiva.


No debe ser prolongada y se debe restablecer secuencialmente luego de 48 horas sin dolor.
La alimentacin parenteral no tiene ningn sentido si el restablecimiento de la va oral se
realiza dentro de los primeros 7 das. De no ser as se sabe que esta patologa grave
provoca una agresin severa que determina un estado hpercatablico por lo tanto debe
implementarse soporte nutricional para evitar la desnutricin y las complicaciones que ella
trae consigo (alteracin de la modulacin de la respuesta inflamatoria, translocacin
bacteriana, inmuno supresin, etc.)

El 60 % de esta enfermedad presenta hipermetabolismo (1,5 veces el metabolismo basal),


el 40 % son normo o hipometablicos. Segn la Conferencia de Consenso de Nutricin de la
Agresin (Francia 1998), los triglicridos no son contraindicados a menos que el paciente
presente una hipertrigliceridemia importante. La necesidad de aporte debe ser de 0,25 a
0,30 g/Kg. La suplementacin de micronutrientes fundamentalmente antioxidantes
(vitaminas A, C, E y selenio) y zinc estn indicadas.
La suplementacin en base a glutamina, inmunomoduladores, nuevas emulsiones lipdicas en
base a aceite de oliva necesitan aun ser confirmadas para la pancreatitis severa.

La va enteral se debe privilegiar ya que no solo es mas fisiolgica sino que tambin
presenta menor costo y complicaciones habiendo actualmente estudios que muestran un
menor ndice de morbimortalidad en estos pacientes ya que la integridad de la barrera
intestinal limitara la sobreinfeccin bacteriana pero fundamentalmente la fngica
( Kalfarentzos 97, Windsor 98, Pupelis 2000)

Referencias:
1.

Banks PA, Freeman ML; Practice Parameters Committee of the American College of
Gastroenterology. Practice guidelines in acute pancreatitis. Am J Gastroenterol.
2006 Oct;101(10):2379-400.
2. Frossard JL, Steer ML, Pastor CM. Acute pancreatitis. Lancet. 2008;371:143-152.

39. - Is the most common genetic feature in ankylosing spoondylitis:


a)
b)
c)
d)

BRCA-1.
HLA B27.
HNF 4alfa.
K-ras de novo mutation.

La espondilitis anquilosante es un padecimiento reumtico inflamatorio,


generalizado y crnico, que afecta primordialmente el esqueleto axial, con la
presencia de dao de las articulaciones S-I (sacroiliitis) como su hallazgo
fundamental.

Races griegas: ankilos (inclinado-fusin), espondilos (vrtebra)


PREVALENCIA: Africanos y esquimales: 0.1 %. Blancos: 0.5 1 %. Indios Haida
(Norte de Canad): 6 %
PREVALENCIA DEL HLA-B27: 6 8 %
PREVALENCIA EN FAMILIARES DE ENFERMOS, HLA-B27 +: 10 20 %
INCIDENCIA: 6.3 6.9/100 habitantes por ao
CONCORDANCIA en gemelos: Monocigotos, 63 %; dicigotos, 12.5 %
EDAD: Se inicia en la adolescencia o la edad adulta temprana. Rara despus de los
40 aos
GNERO: 3 5:1 a favor del hombre

FACTORES GENTICOS:

Antgenos del CPH:


- HLA-B27
- HLA-B60

- HLA-DRB1
- TNF
- Otros de clase III: MICA, TAP, LMP2, LMP7, HSP70,

complotipos

Antgenos fuera del CPH:


- Regiones en los cromosomas 1, 2, 6, 9, 10, 16 y 19; 16q

FACTORES AMBIENTALES
ALTERACIONES INMUNOLGICAS

Brown MA, Crane A, Wordswoth BP.


Curr Opin Rheumatol 2003;14:354-60.

40. - A 60-year-old woman presents to a physician complaining of a swelling in her neck.


Her past medical history is significant for rheumatoid arthritis and Sjgren syndrome.
Physical examination reveals a mildly nodular, firm, rubbery goiter. Total serum thyroxine
(T4) is 10 mg/dL, and third-generation thyroid-stimulating hormone (TSH) testing shows a
level of 1.2 mIU/mL. Antithyroid peroxidase antibody titers are high. Which of the
following is the most likely diagnosis?

a)
b)
c)
d)

Euthyroid sick syndrome.


Graves disease.
Hashimoto thyroiditis.
Silent lymphocytic thyroiditis.

Tiroiditis de Hashimoto

Es la causa ms frecuente de hipotiroidismo en las zonas donde no existe dficit dietario


de yodo y los sntomas de hipotiroidismo suelen ser el motivo de su diagnstico. Es
frecuente su asociacin con otras enfermedades autoinmunes.
Es caracterstica la presencia de un pequeo bocio, firme, irregular y no doloroso al tacto
con sensacin de plenitud en la garganta. Los Ac anti-TPO (anticuerpos antiperoxidasa) son
positivos en el 90% de pacientes y los antitiroglobulina en un 20 a 50%. La VSG es normal o
casi normal. La ecografa muestra una glndula hipoecognica sin ndulos.
Una vez el hipotiroidismo est establecido debe tratarse con hormona tiroidea a dosis
sustitutiva: 50 microgramos diarios inicialmente (25 si edad avanzada, enfermedades
cardiovasculares o consuntivas o niveles previos muy bajos de T4 libre) aumentando 25
mcrgr/ da cada 15 a 30 das segn los mismos criterios, hasta 75-100 micrgr/da,
ajustndose posteriormente segn controles que deben dilatarse al menos 2-3 meses
desde el inicio del tratamiento ya que las respuestas son lentas. Los pacientes con
hipotiroidismo subclnico con niveles altos de Ac anti-TPO debe ser tratados tambin por
su frecuente evolucin a hipotiroidismo clnico y mayor riesgo arteriosclertico por
elevacin lipdica. Si el bocio es grande debe administrase hormona tiroidea en dosis

suficiente para frenar la TSH, lo que puede hacer disminuir en 6 meses hasta un 30% el
tamao de la glndula.
Si se observa un ndulo tiroideo debe ser estudiado mediante PAAF a fin de descartar
carcinoma tiroideo o linfoma (raro, pero con un riesgo relativo de 67 en este tipo de
tiroiditis)

Lectura recomendada.
Tiroiditis de Hashimoto y sndrome de Sjgren.
Asociacin o sndrome mltiple autoinmune?
L. SIERRA SANTOS, A. SENDINO REVUELTA, R. PACHECO CUADROS,
G. APARICIO JABALQUINTO, F. J. BARBADO HERNNDE

Servicio de Medicina Interna. Hospital la Paz. Universidad Autnoma. Madrid


Sierra Santos L, Sendino Revuelta A, Pacheco Cuadros R, Aparicio Jabalquinto G, Barbado
Hernndez FJ. Tiroiditis de Hashimoto y sndrome de Sjgren. Asociacin o sndrome
mltiple autoinmune? An Med Interna (Madrid) 2001; 18: 86-87
HASHIMOTOS THYROIDITIS AND SJGRENS SYNDROME.
ASSOCIATION OR MULTIPLE AUTOIMMUNE SYNDROME?

Bibliografa:
Beers MH, Berkow R. Editores. El manual Merck. 10 ed. Madrid: Harcourt; 1999. p. 96-7.
Fauci AS, Braunwald E, Isselbacher KJ, Wilson JD, Martn JB, Kasper DL et al. Editores.
Harrison Principios de Medicina Interna. 14 ed. Madrid: McGraw Hill; 1998. p. 2310-2.
Larsen PR, Davies TF. Hipotiroidismo y tiroiditis. En: Larsen PR, Kronemberg HM, Melmed
S, Polonsky KS. Williams Tratado de Endocrinologa. 10 ed. Madrid: Elsevier; 2004. p. 465500.
Pearce EN, Farwell AP, Braverman LE. Thyroiditis. N Engl J Med 2003; 348:2646-55.

41.- Al encontrase usted en la sala de expulsin atendiendo un parto, observa que se


enfrenta ante una atona uterina Cul es el manejo inmediato a realizar?

a) Masaje y compresin del fondo uterino.


b) 20 unidades IV de oxcitocina.
c) 0.2mg Im de metilergonovina.
d) Empaquetar con gasas.

Atona Uterina
a) Compresin bimanual externa.

b) Masaje uterino combinado (endouterino y abdominal).


c) Medicamentos: uso de oxitcicos.
d) Evacuacin uterina de cogulos.
e) Transfusin de sangre o hemoderivados.
f) Evaluar factores de coagulacin.
g) De persistir el cuadro clnico, proceder a:
1. Ligadura de arteria uterovrica.
2. Sutura de las paredes uterinas (puntos de colchonero).
3. Sutura de Lynch.
4. Ligadura bilateral de arterias hipogstricas.
5. Histerectoma abdominal subtotal.
6. Histerectoma abdominal total.
h) Hemoglobina - hematocrito control seriado.
Bibliografa:
(1) Gua Clnica Basada en las Evidencias: Manejo de la Hemorrasia Postparto. Centro
Latinoamericano de
Perinatologia y Desarrollo Humano (CLAP). Novedades del CLAP no 16, abril 2002.
(2) OMS/FNUAP/UNICEF/BM. Integrated Management Of Pregnancy And Chidbirth IMPAC, 2000.
(Traduccin al Espaol: Manejo de las complicaciones del embarazo y el parto: Gua para
obstetrices y mdicos 2002.
(3) OutLook. Prevencin de la Hemorragia Postparto: Manejo del Tercer Perodo del Parto.

42.- Mujer de 24 aos la cual presenta un ndulo mamario palpable de aparicin brusca. La
ecografa revela un ndulo anecognico, de limites muy precisos, morfologa regular y
refuerzo posterior, nico de 3.5 cms. de dimetro. El diagnstico ms probable es?
a) Cncer.
b) Displasia fibrosa.
c) Fibroadenoma.
d) Quiste.

Quistes. Los quistes mamarios son fciles de detectar con la ecosonografa. Pueden ser
lesiones nicas o mltiples que se observan como imgenes redondeadas, anecognicas, de
paredes delgadas, contornos bien definidos, con importante reforzamiento acstico
posterior y sombras laterales delgadas. Pueden presentar septos intraqusticos y, en
ocasiones, se pueden observar ecos internos que sugieren detritus celulares o proceso
inflamatorio. Se debe descartar la presencia de lesiones intraqusticas o la coexistencia de
otras alteraciones benignas o malignas. En caso de ser sintomticos, el tratamiento
adecuado es la puncin y aspiracin de la lesin con aguja guiada por palpacin o
ecosonografa de acuerdo con ell tamao, profundidad y caractersticas del contenido. El
uso del ultrasonido garantiza el vaciamiento completo.

REFERENCIAS
Barth V, Prechtel K. Mama normal. En: Barth V, Prechtel K, editores. Atlas de patologa de
la glndula mamaria.
2da ed. Madrid: Editorial Mdica Panamericana, 1991.
Bush H, McCredie A. Carcinoma of the breast during pregnancy and lactation. In: Allen HH,
Nisker JA. Cancer in pregnancy. New York: Futura Publishing Co. Inc., 1986.
Byrd BF, Bayer DS, Robertson JC, Stephenson JE Jr.
Treatment of breast tumor associated with pregnancy and lactation. Ann Surg. 1962;
155:940-7.

43.- Se trata de femenino de 29 aos de edad, la cual inicia con hiperemesis gravdica de
difcil control, as como sangrado trasvaginal. Se realiza el diagnstico de mola hidatiforme.
El tratamiento inicial indicado en esta patologa es?
a)
b)
c)
d)

Histerectoma total abdominal.


Legrado por aspiracin.
Metotrexate y seguimientos radiogrficos.
Legrado uterino instrumental.

La enfermedad trofoblstica gestacional agrupa a diferentes entidades interrelacionadas:


mola completa, generalmente diploide con origen cromosmico paterno, mola parcial
generalmente triploide, tumor trofoblstico del lecho placentario y coriocarcinoma, con
tendencias variables a la invasin local y a las metstasis, cuyo denominador comn es la
hipersecrecin de hCG. El coriocarcinoma es diploide y proviene de ambos progenitores,
excluyendo probablemente su origen directo en la mola completa. El tumor trofoblstico
del lecho placentario est constituido por trofoblasto mononuclear intermedio no
conteniendo vellosidades corinicas e inmunohistoquimicamente caracterizado por expresar
muchas de sus clulas hPL y unas pocas hCG
Tratamiento
Hay que tratar las complicaciones como la hipermesis, anemia, hipertensin, y alteraciones
electrolticas, coagulopatas, alteraciones cardio-respiratorias y preeclampsia, procediendo
a evacuar la mola lo antes posible, con lo que se producirn menos malignizaciones.

La evacuacin del contenido uterino se realiza mediante dilatacin, y legrado por aspiracin.
Adems se pauta profilaxis antibitica y oxitcicos.3
La histerectoma, con la mola en su interior, est indicada en pacientes de edad superior a
40 aos o en mujeres con ms de tres hijos, ya que en ambos grupos se ha demostrado una
mayor incidencia de malignizacin.

. Tras la ciruga, se mide la concentracin de gonadotropina corinica humana para


determinar si la extirpacin ha sido completa. Si es as, el valor de esta hormona vuelve a la
normalidad, en unas 8 semanas, y se mantiene en esos valores. Si una mujer a la que se le ha
extirpado una mola queda embarazada, es difcil interpretar un valor alto de gonadotropina
corinica humana, porque podra estar causado tanto por el embarazo como por una parte
de la mola que no se ha extirpado. En consecuencia, a las mujeres a las que se les ha
extirpado una mola se les recomienda no quedar embarazadas durante un ao. Las molas
hidatiformes benignas no necesitan quimioterapia, pero las malignas s. Los frmacos que se
usan para este tratamiento son el metotrexato, la dactinomicina o una combinacin de
ambos.
Bibliografa:
1.

Silverman L,Romero Zambrano F, Saldao S. Enfermedad molar. Diagnstico,


tratamiento y seguimiento, 1987.
2. Puertas A, Lpez Fernandez J et al. Enfermedad trofoblstica. Casustica del
Hospital Virgen de las Nieves de
Granada. Cln Invest Gin Obs 1993; 20: 98-103.

3.

Enfermedad trofoblstica gestacional. Propuesta Normativa Perinatolgica y


Ginecolgica de Alto Riesgo.
Ministerio de Asuntos Sociales. Tucumn. 1996-1997; 19:171-183.
4. Jones. Enfermedad Trofoblstica Gestacional: qu hemos aprendido en la ltima
dcada. Am J Gynecol Obstet 1990;162: 1286-1292.
5. Resmen del simposio Enfermedad Troblstica Gestacional del Segundo Congreso
Nacional de AGORA, 1990.
6. Gonzalez Merlo et al. Protocolos de diagnstico y tratamiento en Obstetricia y
Ginecologa, Barcelona: editorial Salvat, 7:35-45.

44.- Una mujer de 23 aos de edad gesta 1, tuvo un parto vaginal espontneo con un
producto con peso de 4,350 g. despus de 5 minutos de traccin suave del cordn umbilical
se expuls la placenta, que parece estar intacta. Se inici el masaje del fondo uterino y se
pidi a la enfermera que administrara 20 unidades de oxitocina en 100 ml de solucin
Ringer lactato. Despus de una inspeccin cuidadosa del canal del parto se observa una
laceracin de segundo grado y una laceracin de 2 cm en la pared vaginal izquierda que se
intent reparar. En la exploracin fsica se encuentra un fonso uterino blando y atnico. Los
signos vitales son: temperatura 37.1C, TA 164/92, FC 130x, FR 18 X. Cul de los
siguientes es el mejor tratamiento?
a) Oxitocina 10 unidades directas en goteo intravenoso.
b) Metilergonovina 0.2 mg IM.
c) Prostaglandina F 0.25 mg IM.
d) Legrado.

Morgan M, Siddighi S. Ginecologa y obstetricia, National Medical Series. 5 edicin. Mc


Graw Hill. Pp. 28. La atona uterina es la causa ms comn de hemorragia puerperal. El
masaje energtico y la oxitocina diluida no han sido tiles para interrumpir la hemorragia y
por tanto el siguiente paso es agregar un frmaco uterotnico. La metilergonovina est
contraindicada porque la paciente se encuentra hipertensa a pesar de la hemorragia
intensa, el siguiente frmaco es la prostaglandina. La administracin de oxitocina no diluida,
10 UI por va IV podra causar hipotensin grave. La exploracin manual podra ser
apropiada si se sospecha laceracin como causa de hemorragia. El legrado es apropiado

para la hemorragia puerperal tarda, cuando se sospecha retencin de los productos de la


concepcin.

45.- Femenino de 36 aos, es atendida en consulta externa con reporte de papanicolaou


que reporta un NIC I, la especuloscopa se observa crvix con ectropin periorificiario.
El mtodo ms sensible para corroborar el diagnstico en esta paciente es:
a)
b)
c)
d)

Papanicolaou.
Colposcopa.
Exudado vaginal.
Prueba de koh.

9.5.2 Las pacientes a quienes se les realiz citologa cervical, cuyo resultado es LEIBG
(infeccin por VPH, displasia leve o NIC 1); LEIAG (displasia moderada y grave o NIC 2 y 3)
o cncer deben enviarse a una clnica de colposcopa, para realizar estudio colposcpico.
9.5.3 Si el resultado de la citologa es LEIBG, la colposcopa es satisfactoria y sin
evidencia de LEIBG, se realizar control citolgico en un ao (Apndice Normativo A)
9.5.4 Si la citologa es de LEIBG, la colposcopa es satisfactoria y existe evidencia de
lesin, se debe tomar una biopsia dirigida.
9.5.4.1 Si la biopsia dirigida es negativa, se realizar nueva colposcopa para verificar el
diagnstico y en caso necesario, tomar nueva biopsia dirigida y revalorar.
9.5.4.2 Si la biopsia dirigida es reportada como LEIBG se podr dar tratamiento
conservador: criociruga, electrociruga o laserterapia (slo si cumple con las condiciones
referidas en el Apndice 1) o se podr mantener a la paciente en vigilancia en la clnica de
colposcopa, con colposcopa y estudio citolgico cada seis meses, durante 24 meses.
Jueves 31 de mayo de 2007 DIARIO OFICIAL (Primera Seccin)
9.5.4.3 Si la biopsia dirigida es reportada como LEIAG (Lesin Intraepitelial Escamosa de
Alto Grado) se realizar tratamiento conservador (electrociruga o laserterapia). En las
mujeres posmenopusicas, dependiendo de las condiciones anatmicas del crvix, se
realizar tratamiento conservador en la clnica de colposcopa o tratamiento quirrgico
(histerectoma extrafascial) en el servicio que corresponda.
9.5.4.4 Si la biopsia dirigida reporta cncer microinvasor o invasor, la paciente se
transferir a un Servicio o Centro Oncolgico para su tratamiento correspondiente.
9.5.4.5 Si la citologa reporta LEIBG y la colposcopa es no satisfactoria, se tomar
cepillado endocervical (Apndice Normativo A)
9.6 En caso de colposcopa no satisfactoria, negativa a LEIBG y con cepillado endocervical
negativo, se continuar su control en la clnica de colposcopa en seis meses, con colposcopa
y citologa.

9.6.1.1 Si el cepillado endocervical reporta LEIBG se tratar a la paciente como LEIAG,


con mtodos conservadores escisionales.
Referencias:
Jueves 31 de mayo de 2007 DIARIO OFICIAL (Primera Seccin)
Modificacin a la Norma Oficial Mexicana NOM-014-SSA2-1994, Para la prevencin,
deteccin, diagnstico, tratamiento, control y vigilancia epidemiolgica del cncer
crvico uterino.
Al margen un sello con el Escudo Nacional, que dice: Estados Unidos Mexicanos.- Secretara
de Salud.
MODIFICACION A LA NORMA OFICIAL MEXICANA NOM-014-SSA2-1994, PARA LA
PREVENCION,
DETECCION,
DIAGNOSTICO,
TRATAMIENTO,
CONTROL
Y
VIGILANCIA
EPIDEMIOLOGICA DEL CANCER CERVICO UTERINO.

46.- Se trata de paciente femenino de 37 aos de edad, G-3, C-2. Es ingresada a hospital
presentando cefalea, acfenos, fosfenos y epigastralgia en barra con embarazo de 34
semanas. E.F T/A 160/110, FC 84 x, FR 18 x, no presenta fiebre, somnolienta, sin
agregados cardioventilatorios, hepatalgia. F.U. de 25 cm. Producto nico vivo. FCF 110 lpm,
genitales sin prdidas ni modificaciones cervicales. Laboratorio: hb 9.8 g/dl, plaquetas de
54 mil, TP 11 seg TPT 27, TGO 160 ng/dl TGP 160 ng/dl, hiperbilirrubinemia indirecta,
albuminuria 300 mg/dl, Acido rico de 8.1 mg/dl, creatinina de 1.5 mg/dl.
El diagnstico ms probable es:

a)
b)
c)
d)

Sx anticuerpos antifisfolpidos.
Prpura trombocitopnica trombtica.
Sindrome de hellp.
Hgado graso.

DEFINICIN:
Es una complicacin de la preeclampsia en la cual adems de la Hipertensin Arterial y
proteinuria hay presencia de anemia hemoltica, enzimas hepticas elevadas y recuento
bajo de plaquetas

MANIFESTACIONES CLINICAS:

Malestar general, fatiga y molestias inespecficas 90%


Cefalea 70%
Epigastralgia 64%
Vmito 22%
Fosfenos 15%
Visin Borrosa 11%
Acfenos 3%
Ictericia
Anemia no explicada
Oliguria

BIBLIOGRAFIA:

Sibai baha, El sndrome HELLP. Universidad de Valencia , revista quincenal de Obstetricia


clnica y ginecologa, Octubre 2003.
V. Cararach, Sndrome de HELLP y Repercusiones maternas. X curso intensivo de
formacin continuada materno fetal. Enero de 2003.
Toirac, Abelardo. Sndrome de Weistein HELLP Hospital Ginecoobstetrico Tamara Bunke.
Junio 2002
De la Fuente, David. Sndrome HELLP. Medicina Universitria 2003; 5 (19): 101 -9
Andrea G. Witlin, DO, Baha M. Sibai, MD. Diagnosis and Management of women with
Hemolysis Elevate Liver Enzymes, and Pletelet Count (HELLP) syndrome. Hospital Physician.
Febrero 1999.
CIFUENTES B, Rodrigo. Ginecologa y obstetricia.

47.- Se trata de paciente femenino de 19 aos que acude a consulta refiriendo presentar 6
meses sin regla. Se manifiesta preocupada por su sobrepeso, ha estado a dieta y ha perdido
6 kg en 8 meses. Actualmente pesa 46 Kg. con talla de 1,65 mts. Signos vitales dentro de
sus parmetros normales. Se realiza prueba de embarazo con resultado negativa. Cul es
la causa ms probable de su amenorrea?:

a) Disgenesia gonadal.
b) Hipogonadismo hipogonadotropo.
c) Sndrome de ovario poliqustico.
d) Adenoma hipofisario.

El hipogonadismo femenino se caracteriza por un fallo gonadal debido a la alteracin del


propio ovario o secundario a un fallo hipotlamo-hipofisario.
Este fallo de la gnada puede ocurrir en distintos momentos de la vida y por causas
diversas, lo que va a condicionar una presentacin clnica diferente.
El ovario posee dos funciones relevantes: producir gametos femeninos (oognesis) y
secretar hormonas esenciales en la regulacin de la funcin reproductora y que influyen en
la diferenciacin y el desarrollo de los rganos sexuales (hormonognesis).
Los mecanismos celulares y moleculares responsables de la produccin de oocitos y
hormonas por la gnada femenina son parcialmente independientes entre s. Sin embargo,
ambas funciones son llevadas a cabo de modo concertado gracias a la accin de un complejo
sistema de control que implica interacciones mltiples entre el hipotlamo, la hipfisis y el
ovario.
Para la adquisicin de la capacidad reproductora a partir de la pubertad se requiere una
adecuada formacin e integracin funcional durante las etapas tempranas del desarrollo.
Los elementos que componen este eje reproductor son: 1) el sistema neuronal hipotalmico
responsable de la produccin del neuropptido GnRH; 2) las clulas gonadotropas de la

hipfisis anterior que secretan LH (hormona luteinizante) y FSH (hormona folculo


estimulante), y 3) el ovario.
Hay que destacar que la formacin de la gnada y de los elementos hipotlamohipofisarios del eje gonadotropo se llevan a cabo de modo independiente en etapas
tempranas del desarrollo embrionario, producindose posteriormente su integracin
funcional en circuitos de retroalimentacin positivos y negativos, esenciales para una
correcta funcin reproductora.

FALLO GONADAL PRIMARIO O HIPOGONADISMO


HIPERGONADOTRFICO
Incluye a aquellos pacientes en los que la produccin y accin de los esteroides estn
reducidas. En los primeros 4 aos de vida y a partir de los 9-10 aos, la hipfisis, al faltarle
la retroalimentacin negativa de estos esteroides, aumenta la produccin de
gonadotropinas. Sin embargo, la elevacin de las gonadotropinas no puede normalizar la
funcin gonadal.
Las causas pueden ser congnitas y adquiridas.

Etiologa comn a ambos sexos:

Alteraciones congnitas del receptor de las gonadotropinas

Recientemente se han clonado y mapeado los genes de los receptores de


as gonadotropinas (LH-R y FSH-R) (20), estando ambos en 2p21, y se han identificado
diversas mutaciones y deleciones que conducen a una disminucin de la funcin de estos
receptores, independiente de las gonadotropinas (hipogonadismo hipergonadotropo); en
otros casos se produce un aumento de la funcin de estos receptores.

Mutaciones que ocasionan prdida de la funcin del receptor de la LH

Hasta la fecha se han encontrado seis sustituciones de una base del gen del LH-R que
conducen a dos mutaciones nonsense o sin sentido que ocasiona un codn de parada, y a
cuatro mutaciones missense o mutaciones con error de sentido en los que cambia un
aminocido en la protena. La mayora son homocigotos y se considera que el trastorno es
A-R. Clnicamente, existe una forma grave y una menos grave, lo que tiene relacin con la
actividad del receptor. En las mujeres hay amenorrea, falta de respuesta del ovario a la LH
y respuesta normal a la FSH con desarrollo folicular normal.

Mutaciones que ocasionan prdida de la funcin del receptor de la FSH

Son ms raras. Clnicamente las mujeres homocigotas presentan disgenesia ovrica e


hipogonadismo hipergonadotropo, mientras que las mujeres heterocigotas son
fenotpicamente normales.
BIBLIOGRAFA:
1. Caete R, Jimnez L. Ontogenia y diferenciacin gonadal. Gnadas. Actualizaciones en
Endocrinologa. McGraw-Hill Interamericana. Captulo 1, 2000.
2. Barrio R, Ezquieta B. Mecanismos genticos de la diferenciacin sexual: sus alteraciones.
Gnadas. Actualizaciones en Endocrinologa. McGraw-Hill Interamericana. Captulo13,
2000.
3. De Roux N, Morel Y, Hardelin JP. Genetic anomalies of the gonadotropic axis. Rev Prat

1999; 49 (12): 1277-1282.


4. Tena-Sempere M, Aguilar E, Pinilla L. Fisiologa del eje hipotlamo-hipofiso-ovrico.
Gnadas.
Actualizaciones en Endocrinologa. McGraw-Hill Interamericana. Captulo 2, 2000.
5. Labarta JI y cols. Hipogonadismo hipergonadotrpico. En Pubertad normal y patolgica,
2. curso de postgrado. Valladolid, 1996.
6. Veldhuis JD. Neuroendocrine mechanisms mediating awakening of the human
gonadotropic axis in puberty. Pediatr Nephrol 1996; 10 (3): 304-317.
7. Hopwood NJ. Pathogenesis and management of abnormal puberty. Spec Top Endocrinol
Metab 1985; 7: 175-236.
8. Minagawa M, Yasuda T, Niimi H J. Spinal and femoral bone mass accumulation during
normal adolescence: comparison with female patients with sexual precocity and with
hypogonadism. Clin Endocrinol Metab 1996; 81 (3): 1248-1253.
9. Park KH, Lee SJ, Kim JY, Kim JY, Bai SW, Kim JW. A concomitant decrease in cortical
and trabecular bone mass in isolated hypogonadotropic hypogonadism and gonadal
dysgenesis. Yonsei Med J 1999; 40 (5): 444-9.

48.- Es el caso de paciente femenino de 17 aos la cual presenta amenorrea acompaada


de profundas alteraciones del olfato. Cul de los siguientes diagnsticos es el ms
probable?
a) Sndrome de amenorrea-galactorrea.
b) Amenorrea de causa uterina.
c) Sndrome de ovario poliqustico.
d) Amenorrea por alteracin hipotalmica.

AMENORREA HIPOTALAMICA. (Hipogonadotrpica)


Psicgena. (Stress emocional)
Anorexia nerviosa. (Deficiencia nutricional)
Ejercicio excesivo. (Carrera de fondo, natacin, gimnasia, ballet)
Frmacos. (Fenotiazina, reserpina, bloqueadores ganglionares,
anticonceptivos)
Pseudocisis.
Sndrome de Kallman (Deficiencia selectiva de gonadotropinas y anosmia).
El ejemplo clsico de la alteracin hipotalmica que lleva a desrdenes del ciclo menstrual
es el Sndrome de Kallman.

Lectura Recomendada:
Etiopatogenia de la amenorrea hipotalmica funcional Interaccin de las respuestas
hormonales del Sistema Nervioso Central y Neuropptidos Perifricos
Revista Argentina de Endocrinologa y Metabolismo

Copyright 2008 por la Sociedad Argentina de Endocrinologa y Metabolismo


Vol 45 No. 2.

49.- Femenino de 40 aos de edad G.3 P.1 C 2, la cual es diagnosticada por miomatosis
uterina de pequeos y medianos elementos sintomticos, sus antecedentes refieren
cirugas plvicas previas, el tratamiento de eleccin es:

a) Histerectoma total abdominal.


b) Progesterona.
c) Observacin.
d) Anlogos de GnRH.

CUADRO CLINICO
La miomatosis uterina muestra manifestaciones clnicas en menos del 50%, de estas las ms
frecuentes son:
1.
2.
3.
4.
5.
6.

Hemorragia uterina anormal.


Dolor.
Distensin abdominal.
Compresin genitouterina.
Compresin gastrointestinal.
Compresin plvica.

10. DIAGNOSTICO
El diagnstico se realiza a travs de imagenologa:
1-Ecografa.
2-TAC.
3-Rayos X.
4-Histeroscopia.

11. TRATAMIENTO
La miomatosis uterina debe ser tratada cuando produzca cualquiera de las manifestaciones
clnicas anotadas, toda paciente que se programe para histerectoma debe tener
previamente legrado biopsia.
Lo podemos dividir en Conservador o Radical.
1.

CONSERVADOR:

Este tratamiento se puede instaurar en pacientes con deseo de preservar el tero.


Igualmente se puede subdividir:
-Expectante
-Quirrgico: miomectoma
-Medico
Tratamiento expectante: Esta indicado en pacientes, cuyos sntomas son leves y no deseen
o tengan alguna contraindicacin medica para tratamiento quirrgico. En ellas se
recomienda controles clnicos y ecogrficos cada 6 meses a 1 ao.
Tratamiento mdico.
AINES
Anlogos GnRH:
Progestgenos:
Andrgenos.
Antiandrgenos

Tratamiento quirrgico:
MIOMECTOMIA:
1. Criterios del ACOG en pacientes infecundas.
Procedimientos:
Va endoscpica:
Laparoscopia: Miomas subserosos sesiles o pediculados < 5cm.
Histeroscopia: miomas submucosos
Va laparotoma
Aquellos miomas que se salgan de las caractersticas anteriores.

Indicaciones:

Hemorragia anormal.
Perdida reproductiva.
Infecundidad.
Dolor.

El tratamiento previo con anlogos esta indicado cuando se desee disminuir el tamao del
mioma para prevenir sangrado quirrgico.

Contraindicaciones:

Embarazo.
Cncer endometrial.
Infecciones.
Dificultad tcnica.

2. Criterios del ACOG para Miomectoma en pacientes que desean conservar el tero.
Procedimiento:

Va endoscpica.
Va abdominal.
Va vaginal.

Indicacin:

a.

Presencia de uno o dos Leiomiomas asintomticos de tamao tal que se pueden palpar
por va abdominal y constituyen una preocupacin para la paciente.

b.

Pacientes ovulatorias con miomas como posible causa de hemorragia uterina excesiva,
demostrada por cualquiera de las siguientes circunstancias:
Hemorragia profusa: de duracin mayor de 8 das.
Anemia por prdida sangunea aguda o crnica.

2. RADICAL:
HISTERECTOMIA
Para pacientes post menopusicas, con paridad satisfecha o sin deseo de preservar el
tero.
Criterios del ACOG para Histerectoma por miomas.

1.

Presencia de 1, 2, o 3 Miomas asintomticos de tamao tal que son palpables por va


abdominal y preocupan a la paciente.

2. Hemorragia uterina excesiva.

Duracin mayor de 8 das.


Anemia por prdida sangunea aguda o crnica.

1.

Molestias plvicas producidas por los miomas: Signos compresivos.

Contraindicaciones:
1. Deseo de conservar la fecundidad.
2. Miomas asintomticos.
3. Contraindicacin mdica o dificultades tcnicas para la ciruga.

Bibliografa:
Guarnaccia M. and Rein M. Traditional Surgical Approaches to Uterine Fibroids
Abdominal. Myomectomy and Hysterectomy. Clinical Obstetrics and Gynecology
2001. 44.2. 385-400.
- Milad. M and Sankpal R. Laparoscopic Approaches to Uterine Leiomyomas.
Clinical Obstetrics and Gynecology. 2001. 44-2. 401-411.
- Carlson K. et al. Indications for Hysterectomy. N. Engl. J. Med. 1993. 328(12) 56.

50.- Femenino de 35 aos de edad, a quien se realiza diagnstico de placenta percreta, el


tratamiento e eleccin en esta patologa es:

a)

Ergonovina a dosis altas.

b)

Hemostasia con puntos transfictivos.

c)

Histerectoma.

d)

Taponamiento uterino.

La placenta anormalmente adherida es poco comn y tiene importancia clnica por su


morbimortalidad, a consecuencia de hemorragia, perforacin, invasin y lesin de las vas
urinarias. Esta adherencia anormal est asociada con la implantacin placentaria sobre
cicatrices de cesrea previa, incisiones uterinas o legrados. La placenta percreta consiste
en la penetracin del tejido placetario a travs de toda la pared uterina, traspasando la
serosa de la misma. La identificacin de esta anormalidad antes del parto es posible
mediante mtodos de imagen (escala de grises por ultrasonido, ecografa Doppler color
pulsado o resonancia magntica nuclear). El tratamiento conservador se acompaa de
elevada morbilidad en muchos casos, por lo que el tratamiento quirrgico se convierte en
el definitivo. La literatura sugiere un aumento previsto en la incidencia de esta condicin
con base en el incremento del nmero de cesreas, por lo que la histerectoma postcesrea
ser una decisin que enfrentarn los especialistas con mayor frecuencia.

Bibliografa:
1.

Perucca E, Domnguez C, Yahng Ch, Garca R. Placenta previa percreta con invasin
vesical. Rev Chil Obstet Ginecol 1997; 62(3): 206-10.
2. Abbas F, Talati J, Wasti S et al. Placenta percreta with bladder invasion as a cause of
life threatening hemorrhage. J Urol 2000; 164: 1270-4.
3. Perucca E, Cazenave H, Barra A, Ochoa N, Villagrn G, Espinoza R, Estay R, Bustamante
R, Siebert A. Placenta previa percreta con invasin vesical. Rev Chil Obstet Ginecol
2002; 67(5): 364-7.
4. Price F, Resnik E, Heller K, Christopherson W. Placenta previa percreta involving de
urinary bladder. A report of two cases and review of the literature. Obstet Gynecol
1991; 78(3): 508-11.

51.- La principal causa de amenaza de parto pretrmino y de ruptura prematura de


membranas es:

a) Traumatismo.
b) Idioptico.
c)

Infeccin de vas urinarias.

d) Infeccin vaginal por cndida albicans.

La infeccin de vas urinarias es la complicacin infecciosa ms frecuente del embarazo. De


hecho, las mujeres son ms susceptibles a la infeccin de vas urinarias debido a los
siguientes factores:

Una uretra ms corta.


Fcil contaminacin de la uretra por bacterias de la vagina y el recto.
Posibilidad de la que la mujer no vace por completo la vejiga cada vez que
orina.
Movimiento de bacterias al interior de la vejiga con cada relacin sexual.

Se suman adems, los cambios que el propio embarazo produce en el aparato urinario como
son la relajacin del msculo liso de los urteres que impide que la orina llegue
adecuadamente a la vejiga para ser eliminada, y la compresin que ejerce la matriz sobre la
vejiga lastimando su cubierta interna y dejndola incapacitada para vaciarse por completo.
Bibliografa: 1. Velasco MV. Prevencin y tratamiento del parto pretrmino. Lo nuevo
acerca del viejo problema Rev Med IMSS 2001, 39 (%) 417-42.
2. Lastra ELG El parto pretrmino como problema de salud pblica .Perinatol Reprodud.
Human.
Vol 15 No 2.Abril-junio 2001 113-14.

52.- Una mujer de 27 aos G/4, P/0 con 6 SDG acude a su primera visita prenatal. Su
historia obsttrica pasada es importante porque tiene tres prdidas de producto en el
segundo trimestre. Refiere que en las tres ocasiones al presentarse al hospital presentaba
dilacin cervical completa. No recuerda haber tenido contracciones dolorosas. Niega
antecedentes mdicos y quirrgicos. El examen fsico es normal incluyendo un examen
plvico que muestra un cervix largo y cerrado. Despus de una larga discusin con la
paciente ella pide que se le practique un cerclaje durante este embarazo. Cual de los
siguientes es el momento ms apropiado para realizarlo?

a)
b)
c)
d)

Inmediatamente.
12 a 16 semanas.
24 a 28 semanas.
32 a 36 semanas.

El cerclaje cervical tiene sus indicaciones en la profilaxis y tratamiento de la incompetencia


cervical.

La incompetencia o insuficiencia cervical representa un 10% de las causas de parto


pretrmino y est asociada a una importante morbimortalidad neonatal.
Las modificaciones cervicales en el segundo trimestre de gestacin son causa de parto
prematuro y pueden deberse a:

1) Incompetencia cervical.
2) Prdida de tejido conectivo tras una ciruga cervical (conizacin).
3) Defectos congnitos como la hipoplasia cervical tras exposicin a dietilestilbestrol.
4) Infeccin intrauterina. Hasta un 51.5% de las pacientes con clnica compatible con
incompetencia cervical enmascaran un cuadro de infeccin intraamnitica subclnica.
Diferenciamos tres tipos de cerclaje:
1. El cerclaje se considera profilctico o electivo (o primario) cuando se realiza de forma
electiva por historia previa de incompetencia cervical antes de evidenciar cambios en el
cerviz y generalmente suele realizarse entre las 13 y 16 semanas de gestacin.
2. El cerclaje teraputico secundario que se realiza tras la deteccin, en el seguimiento
obsttrico, de modificaciones en el crvix antes de las 26 semanas de gestacin. Se realiza
en pacientes con un riesgo potencial de parto pretrmino.
3. El cerclaje teraputico terciario, en caliente, de rescate o emergent cerclage que
se realiza en pacientes que presentan la membrana amnitica visible a travs del orificio
cervical externo o en vagina.

GUIA CLNICA:
INDICACIONES DEL CERCLAJE
Unitat de Prematuritat. Servei de Medicina Maternofetal.
Institut Clnic de Ginecologia, Obstetrcia i Neonatologia, Hospital Clnic de Barcelona
Responsables del protocolo: T.Cobo, M. Lpez, M. Palacio.
Creacin: 24/01/07.
Modificaciones: 05/09/07.
ltima actualizacin: 17/01/10.

53.- Mujer de 26 aos, con tumor anexial de 6 cm lquido, dolor abdominal, fiebre,
leucorrea, con historia de cervicovaginitis de repeticin y dispareunia crnica, ltima
menstruacin hace una semana. El diagnstico ms probable ser:
a)
b)
c)
d)

Cistadenoma.
Embarazo ectpico.
Quiste de ovario.
Enfermedad plvica inflamatoria.

La EIP puede cursar con los siguientes sntomas:

Dolor abdominal bajo (incluyendo dolor anexial, dispareunia). Es el sntoma ms


frecuente (95%)
Aumento del flujo vaginal, flujo de caractersticas anormales (74%)
Sangrado anormal (intermestrual, poscoital) (45%)
Sntomas urinarios (35%)
Vmitos (14%)

Es posible la ausencia de sntomas.

Y en ella podemos encontrar estos signos:

Dolor a la movilizacin del cuello, dolor anexial en la exploracin vaginal bimanual


(99%)
En el examen con espculo observamos cervicitis y descarga endocervical purulenta
(74%)
Fiebre (> 38 C) (menos del 47%).
Masa plvica: sugiere abceso tuboovrico (ATO)
Peritonitis

CRITERIOS CLINICOS PARA EL DIAGNOSTICO DE SALPINGITIS


a. Dolor abdominal con o sin rebote.
b. Sensibilidad a la movilizacin del crvix.
c. Sensibilidad anexial.
Los tres criterios anteriores son necesarios para establecer el diagnstico, con uno o ms
de los siguientes:
a. Extendido de Gram de endocrvix positivo, para diplococos gram negativos
intracelulares .
b. Temperatura mayor de 38C .
c. Leucocitosis (mayor de 10.000 por c.c.)
d. Material purulento (positivo para leucocitos) en la cavidad peritoneal
obtenido por culdocentesis o laparoscopia.
Establecido el diagnstico clnico de EPI, se debe hacer la definicin del estado clnico y
anatmico de la patologa plvica:
a) No complicada (limitada a trompas u ovarios)
1) Sin peritonitis plvica.
2) Con peritonitis plvica.

b) Complicada (masa inflamatoria o absceso que compromete trompa (s) u ovario (s)
1) Sin peritonitis plvica.
2) Con peritonitis plvica.

Bibliografa:
1.- Beigi RH, Wiesenfeld HC. Pelvic inflammatory disease: new diagnostic criteria
and treatment. Obstet Gynecol Clin Norh Am. 2003; 30 (4): 777 93
2.- Center for Disease Control. Guidelines for treatment of sexually transmited
diseases. MMWR Recomm Rep. 2002 May 10;51(RR-6):1-78
3.- Center for Disease Control. Guidelines for prevention and management (MMWR.
40: 1 - 25 1991) Pelvic inflammatory disease: guidelines for prevention and
management.MMWR Recomm Rep. 1991 Apr 26;40(RR-5):1-25.
4.-Hager WD, Eschenbach DA, Spence MR, Sweet RL. Criteria for diagnosis and
grading of salpingitis. Obstet Gynecol. 1983 Jan;61(1):113-4.
5.-Prodigy Guidance. Pelvic inflammatory disease. [Internet]. UK : NHS,
Department of Health; 2003. [Acceso 18 de Junio de 2005]. Disponible en:
Ross J. Pelvic inflammatory disease. Clin Evid. 2004 Dec;(12):2259-65.
6.- Royal College of Obstetricians and Gynaecologists. Pelvic Inflammatory Disease.
Guideline n 32. [Internet]. RCOG; Mayo 2003. [Acceso 18 de Junio de 2005].

54.- Femenino de 25 aos con antecedentes de G/2, P/1, C/1 acude al servicio de consulta
externa, refiere que presenta una secrecin transvaginal bastante lquida, de baja
viscosidad, maloliente de color amarillo y gris, espumoso. Esta entidad es propia de
infeccin por:

a) Cndida albicans.
b) Gardenerella.
c) Tricomonas.
d) Gonococos.

El protozoario Thricomona Vaginalis es el responsable del 25% de las vaginitis.


20-50% de las mujeres cursan asintomticas.
La tricominiasis es predominantemente una infeccin transmitida sexualmente.
Debe sospecharse de abuso sexual en caso de encontrar Trichomona en pacientes
peditricos.

DIAGNOSTICO:
Flujo vaginal amarillo-verdoso con burbujas, muy ftido, irritacin vulvo-vaginal,
disuria.
El pH suele ser mayor de 4.5.

Estudio en fresco en donde se observa al microscopio el organismo flagelado


caracterstico.
75% se diagnostican con el estudio del Papanicolaou.

Referencias bibliogrficas:
1. Secretara de Salud. Norma Oficial Mexicana NOM -039-SSA2-2002, Para la prevencin
ycontrol de las infecciones de transmisin sexual. D.O.F. 19 de Septiembre 2003.
2. Kettler H, White K, Hawkes S. Mapping the landscape for sexually transmitted
infections:key findings and recommendations. Geneva, TDR (TDR/STI/ IDE/04.1).
3. CDC. Trends in Reportable Sexually Transmitted Diseases in the United States. CDC,
National Report. 2004.
4. Distribucin de los casos nuevos de enfermedades por mes Estados Unidos Mexicanos
2004. Sistema nico de Informacin para la Vigilancia Epidemiolgica/Direccin General
de Epidemiologa/SSA.
5. Aral S O. Sexual risk behaviour and infection: epidemiological considerations. Sex.
Transm. Inf. 2004;80:8-12.

55.- Una mujer de 33 aos de edad G3 P3, acude a consulta en busca de informacin acerca
de los mtodos anticonceptivos. No tiene APP de importancia, ni toma medicamentos. Ha
mantenido una relacin mongama con su pareja por los ltimos 9 aos. No quiere volver a
embarazarse, pero se niega a que se le practique una salpingoclasia. Su mdico recomienda
un DIU. Cul de los siguientes es ms probable que ocurra con este mtodo
anticonceptivo?

a)
b)
c)
d)

Amenorrea.
Embarazo ectpico.
Embarazo intrauterino.
Aumento de peso.

ETIOLOGIA

III.1 Factores que dificultan el camino del huevo. Este se implanta all donde se encuentra

en el 6-7 da postfecundacin.
A/ Procesos inflamatorios, causa ms frecuente de embarazo ectpico. Las salpingitis
deterioran la actividad ciliar y la motilidad tubrica. Hay otros procesos inflamatorios,
como, apendicitis que afectan secundariamente a las trompas, con produccin de
adherencias y acodaduras.
B/ Alteraciones de la motilidad tubrica, como ocurre con la administracin de gestgenos
a bajas dosis de forma continua (minipildora) o el empleo de contracepcin postcoital con
estrgenos.

C/ DIUs. 9-17% de embarazos en portadoras de DIU son ectpicos, lo que se explica


porque el DIU protege ms eficazmente frente a la gestacin intrauterina (995% de
seguridad) que frente al tubrico (95% de seguridad) con lo que aumenta el riesgo relativo
de E.E.

D/ Tratamientos de esterilidad, sobre todo los quirrgicos, cirugia conservadora de E.E


tubricos, ligadura tubrica o tcnicas de fecundacin asistida (in vitro y transferencia
embrionaria intrauterina)
III.2 Factores que favorecen la nidacin en la trompa: endometriosis tubrica, por un
cierto quimiotactismo del endometrio ectpico, as como, por la estenosis y adherencias que
se producen incluso con rganos vecinos.
III.3 Factores dependientes del propio huevo. Las anomalas genticas del huevo, mediante
alteracin en el momento de implantacin, o quiz por un mayor volumen, puede favorecer la
anidacin ectpica.
BIBLIOGRAFA EMBARAZO ECTOPICO
Cabero Roura, Ll y cols. Protocolos de Medicina Materno-fetal (Perinatologa), 2 edicin.
Ed. Ergon, S.A. Madrid. 2000. 120-122.
Bajo Arenas, J.M, Castellanos Bolado, P. Embarazo ectpico. En: Manual de asistencia a la
patologa obsttrica. Fabre E, Ed. INO: Zaragoza. 1997. 89 -107.
Carrera Macia, J.M y cols. En: Protocolos de obstetricia y Medicina perinatal del I. U.
Dexeus. 3 Edicin. Masson. Barcelona. 2000. 135-139.

56. - An 18-year-old woman complains of myalgias, a sore throat, and painful mouth sores
for 3 daysduration. Her temperature is 38.2 C (100.8 F), blood pressure is 110/80 mm Hg,
pulse is 84/min, respirations are 15/min. Her gingival are edematous and erythematous,
and there are vesicles on her right upper and lower lips. Her pharynx is mildly
erythematous but without exudates, and there is tender mobile cervical lymphadenopathy.
Her breath is not fetid, and the dentition is normal. Which of the following is the most
likely causal agent?

a) Actinomyces israelii.
b) Herpes simplex virus 1.
c) Nocardia asteroids.
d) Streptococcus pyogenes.

Las infecciones por herpes simplex son comunes en la prctica diaria, y con frecuencia
elpaciente acude a los servicios de urgencia. Estas infecciones son ocasionalmente
recurrentes, generalmente dolorosas y asociadas con sntomas sistmicos, por lo cual el
mdico de urgencias debe estar familiarizado con el cuadro clnico y su manejo.
Existen dos variedades de virus del Herpes simplex (VHS) capaces de causar infeccin en
el hombre: el tipo 1 (VHS-1) y el tipo 2 (VHS-2) que se distinguen entre s por varias

caractersticas, incluyendo sus comportamientos clnico y epidemiolgico, antigenicidad,


composicin del ADN y la sensibilidad a diferentes agentes fsicos y qumicos (Cuadro No.1)
Cuadro No. 1
DIFERENCIAS ENTRE LOS VIRUS HERPES SIMPLEX TIPOS 1 Y 2
Caractersticas clnicas
Va de transmisin
Sndromes
caractersticos

Reactividad a antgenos
monoclonales especficos

VHS-1
Oral
Oral-facial
Ocular
Encefalitis
Paroniquia
VHS-1 especfico

VHS-2
Genital
Genita
PerianaL
Neonata
Paroniquia
l VHS-2 especfico

El VHS-1 es de localizacin primordialmente extragenital, con predileccin por los tejidos


de origen ectodrmico, mientras que el VHS-2 corresponde al "Herpes progenitalis"
descrito por separado, dentro de las infecciones de transmisin sexual.
La regin oral es la localizacin habitual del Herpes simplex 1, el cual es causa frecuente de
lesiones orofaciales recurrentes y de otro tipo de enfermedades (encefalitis).
EPIDEMIOLOGIA
El ser humano es el nico reservorio natural conocido del virus herpes simplex, aunque
algunos animales de experimentacin pueden infectarse con facilidad.
La infeccin primaria del VHS-1 ocurre sobre todo durante la infancia, mientras que el tipo
2 se presenta en la adolescencia y adultos jvenes activos sexualmente. Las tasas de
infeccin son inversamente proporcionales al estrato socioeconmico.
El principal mecanismo de transmisin es el contacto directo con las secreciones infectadas.
El VHS-1 se trasmite por saliva y el VHS-2 por va genital. Aunque los ttulos virales son
ms altos cuando existen lesiones activas, tambin es frecuente la liberacin viral en
infectados asintomticos. Por lo tanto, la transmisin viral puede efectuarse an en
ausencia de lesiones activas.
La persistencia de la infeccin y la recurrencia de las lesiones son un fenmeno frecuente
tanto para el VHS-1 como para el VHS-2 y por lo comn se producen por reactivacin
endgena. Los factores que la precipitan van desde la luz solar, el viento, traumatismos
locales, fiebre, menstruaciones y hasta estrs emocional.
DIAGNOSTICO
Cuadro Clnico. Los cuadros clnicos causados por este virus se suelen dividir en dos grupos:
el debido a la infeccin primaria y el correspondiente a la infeccin recurrente. En el
primer grupo se incluyen la gingivoestomatitis aguda, la vulvovaginitis aguda y la infeccin

herptica del ojo, que puede llegar a queratitis. Las recurrentes se circunscriben al
"Herpes labialis", queratitis, blefaritis y queratoconjuntivitis. Todos los cuadros son
autolimitados, pero tanto las formas primarias como las recurrentes, se pueden complicar.
Una de estas complicaciones es la Encefalitis herptica y el Eczema herpeticum.
Infeccin primaria. El primer contacto clnico de infeccin por virus del herpes simple
suele ser el ms grave. Los enfermos aquejan fiebre, malestar general, artralgias y por
ltimo la presencia de un grupo de vesculas sobre una base eritematosa, dolorosa,
inflamada y sensible. La gingivoestomatitis es la manifestacin ms comn, cuya gravedad
vara desde la erosin de pequeas reas a la ulceracin extensa de la boca, lengua y encas.
La infeccin puede ser bastante grave como para dificultar la ingesta de alimentos y
lquidos (odinofagia). La curacin tiene lugar en 7 a 14 das, a menos que las lesiones se
sobreinfecten con estafilocos o estreptococos.
Infeccin recurrente. Generalmente existe prurito, dolor o molestias focales que
preceden la aparicin de las vesculas. Las vesculas se rompen espontneamente despus
de unos cuantos das y sanan en una semana sin dejar secuelas.
LECTURAS RECOMENDADAS
1.

Callen JP, Cooper Ma. Dermatologic emergences. Emerg Med. Clin North Am 3:641,
1985.
2. Guzmn M. Herpes simple, varicela zoster. En: Medicina Interna. Segunda Edicin.
Editado por F Chalem, JE Escandn, J Campos, R Esguerra. Fundacin Instituto de
Reumatologa e Inmunologa. Editorial Presencia Ltda. Santaf de Bogot, 1992.
3. Guerra Flecha J, Lizarraga Bonelli S. Enfermedades de transmisin sexual: herpes
genital Trib Med 79:29, 1989.
4. Jaramillo AC. Infecciones virales de la piel y sus anexos. En: Fundamentos de
Medicina. Enfermedades Infecciosas. Cuarta edicin. Corporacin para
Investigaciones Biolgicas. CIB. Medelln, 1989.

57. - A 17-year-old patient presents with severe pruritus that is worse at night. Upon
examination of the skin, areas of excoriated papules are observed in the interdigital area.
Family members report similar symptoms. Which of the following is the most likely
diagnosis?

a)
b)
c)
d)

Scabies.
Cutaneos larva migrans.
Contact dermatitis.
Dermatitis herpetiformis.

Es un padecimiento parasitario de la piel, sumamente pruriginoso, producido por el caro


Sarcoptes Scabiei Hominis. Suele afectar la piel, en forma generalizada con predominio en
pliegues. La incidencia es mundial, sin embargo es mas frecuente en pases con niveles
socioeconmicos bajos debido al hacinamiento y la falta de higiene. Se puede trasmitir por

contacto directo de piel a piel o por ropas o reas infestadas por el parsito, de ah que
con frecuencia sea un padecimiento familiar.
Caractersticas Clnicas
Los sitios de predileccin son los pliegues interdigitales en manos, caras laterales de dedos,
cara anterior de mueca, pliegues axilares anteriores, regin submamaria, areola, pezones,
brazos, antebrazos, caras internas de muslos, pene, escroto y regin nter gltea.
Las lesiones son ppulas con costras hematicas debidas al rascado intenso de predominio
nocturno. Rara vez se observan los tneles que produce el parsito. En adultos se respetan
las lneas de hebra, mientras que en lactantes o nios pequeos no sucede esto, siendo en la
edad peditrica un cuadro es generalizado.
Debido al prurito intenso y el rascado que este produce, una complicacin frecuente es la
impetginizacin, mientras que la automedicacin es la causa de la dermatitis por contacto
que en ocasiones tambin complica a esta enfermedad.

Bibliografa:
Executive Committee of Guideline for the Diagnosis, Ishii N. Guideline for the diagnosis
and treatment of scabies in Japan (second edition). J Dermatol. 2008 Jun;35(6):378-93.
- Walton SF and Currie BJ. Problems in Diagnosing Scabies, a Global Disease in Human and
Animal Populations. Clin. Microbiol. Rev. 2007 20: 268-279. doi:10.1128/CMR.00042-06
- Ulrich R Hengge, Bart J Currie, Gerold Jger, Omar Lupi, Robert A Schwartz. Scabies: a
ubiquitous neglected skin disease. The Lancet Infectious Diseases, Dec 2006;6(12):769779. doi:10.1016/S1473-3099(06)70654-5.

58.- Se trata de paciente inmunocomprometido que cursa con diarrea provocada por
Isospora Belli. El tratamiento de eleccin es:

a)
b)
c)
d)

Metronidazol.
Tinidazol.
Trimetroprima-Sulfametoxazol.
Albendazol.

Se han usado muchos agentes para tratar las infecciones por I. belli. Las combinaciones de
inhibidores de la dihidrofolato reductasa timidilato sintetasa, como el trimetropim (TMP) o
la pirimetamina, con sulfonamidas como el sulfametoxazol (SMX), sulfadiazina o
sulfadioxina son de probada eficacia, siendo el cotrimoxazol (TMP-SMX) el tratamiento de
eleccin. El uso de cotrimoxazol para el tratamiento o prevencin de la neumona por
Pneumocystis carinii previene la adquisicin de la primoinfeccin por I. belli o las

recrudescencias de la infeccin. La pirimetamina sola tambin es eficaz en pacientes con


alergia a las sulfonamidas.

Bibliografa:
CAMARENA JJ, BORRS R, GARCA DE LOMAS J. Coccidios intestinales.
Cryptosporidium e Isospora. En: Perea EJ. Enfermedades Infecciosas y Microbiologa
Clnica. Vol. II. Ediciones Doyma, Barcelona 1992, pp 1027-1033.

59.- Femenino de 45 aos que tras 20 min. Posteriores al inicio de una transfusin
presenta fiebre, ansiedad, dolor lumbar severo, nausea y enrojecimiento facial, por lo que
usted sospecha:
a) Reaccin hemoltica aguda.
b) Reaccin febril.
c) Toxicidad por citrato.
d) Hepatitis.

R e a c i n Hemoltica
Definicin:
Destruccin acelerada del eritrocito. De acuerdo a la causa puede ser inmune o no inmune,
por el sitio de destruccin puede ser intra o extravascular y por el tiempo de aparicin
puede ser aguda o retardada.
Incidencia
Reaccin hemoltica aguda: Las referencias internacionales reportan una incidencia de
reaccin hemoltica aguda de 1 en 6
000 en 30 000 unidades transfundidas, con una tasa de mortalidad de 1 en 500 000 a 1 en 1
000 000 de unidades.
Del total de las reacciones hemolticas agudas, el 6% resultan fatales. La FDA reporta que
alrededor del 41% de las muertes por transfusin son causadas por incompatibilidad ABO,
con una incidencia de mortalidad de 1 en 200 000 pacientes transfundidos.
Reaccin hemoltica retardada. La incidencia es de 1 en 2 500 a 1 en 4 000 unidades
transfundidas. La mortalidad es de 1 en
3.85 millones de unidades y de 1 en 1.15 millones de pacientes transfundidos.

Fisiopatogenia
La reaccin hemoltica transfusional ms grave se presenta cuando interactan los
eritrocitos transfundidos con los anticuerpos preformados en el receptor. La reaccin
antgenoanticuerpo puede no activar complemento de acuerdo a la inmunoglobulina implicada,

lo que conduce a hemlisis intra o extravascular. En la hemlisis extravascular algunas


citocinas con actividad inflamatoria y vasoactiva intervienen en la reaccin como: Factor de
necrosis tumoral alfa, Interleucina 1, 6, 8 y Protena quimioatractante de macrfago (MCP),
as como la liberacin de sustancias tromboplsticas que explican el cuadro clnico
caracterstico de la reaccin hemoltica transfusional. En la hemlisis extravascular el
eritrocito sensibilizado es destruido por el sistema fagoctico mononuclear.

REACCION HEMOLITICA AGUDA INMEDIATA DE TIPO INMUNE:


Es la reaccin ya descrita por incompatibilidad ABO y como consecuencia de ella al iniciar la
transfusin en pocos minutos pueden ser lisados la mitad ms de los de los hemates
transfundidos.
La reaccin hemoltica aguda se produce de forma brusca, tras la infusin de varios
mililitros de sangre aparece un cuadro de fiebre,escalofrios,dolor lumbar, dolor opresivo en
torax , cefalea nauseas con sin vomitos e incluso shock,dependiendo de la cantidad y
rapidez de la sangre transfundida.
Los sintomas dependen de los pacientes pero la fiebre aparece en todos los casos.
La reaccin es inmediata al inicio de la transfusin
En los enfermos anestesiados estas reacciones quedan enmascaradas y ser mnimas,por lo
que la hemlisis puede debutar con taquicardia e hipotensin brusca trs el inicio de la
transfusin,si persiste la transfusin puede aparecer oliguria,hemoglobinuria,CID fracaso
renal postoperatorio debido a algn efecto txico de la Hb a la situacin de shock.

Diagnstico Diferencial
Contaminacin Bacteriana del componente sanguneo
Hemlisis no inmune:
- Mecnica.
- Trmica.
- Osmtica.

Lecturas recomedadas:
Rev Mex Med Tran, Vol. 3, Nm. 1, pp 18-21 Enero - Abril, 2010
Hospital de Pediatra CMN Siglo XXI Boletn La Calidad Abr May Jun 2004.

60.- Femenio de
importante es:

28 aos con diagnstico de abceso mamario, la medida inicial ms

a) Antibiticos.
b) Drenaje quirrgico del absceso.
c) Instituir una terapia con antiinflamatorios.
d) Suspender la lactancia.

En el desarrollo del absceso puerperal hay inicialmente dolor, enrojecimiento, induracin;


en las fases tempranas de la celulitis los antibiticos son muy efectivos, pero una vez
formado el absceso, al igual que en el absceso no relacionado con la lactancia, se impone el
drenaje quirrgico.
El drenaje debe ser amplio, y en los abscesos no puerperales es necesario abrir los tabiques
que con frecuencia se forman en la cavidad del absceso.
Muchos autores preconizan el simple drenaje sin cubrimiento antibitico. Si se usan
antibiticos, stos deben ser debidamente seleccionados de acuerdo con el germen,
recordando que el ms frecuente es el Staphylococcus aureus.
Bibliografa:
1.

Beller FK, Galask RP. Infections of the breast. En: Infectious Diseases. Edited by
SL Gorbach, JG Bartlett, NR Blacklow.WB Saunders Co. Philadelphia, 1992
2. Benson EA. Management of breast abscesses.World J Surg 23:753, 1989

3. Bundred NJ, Dixon JMJ, Lumsden AB, et al. Are the lesions of duct ectasia sterile?
Br J Surg 72:844, 1985.
4. Ingham HR, Freeman R, Wilson RG. Anaerobic breast abscesses. Lancet 1:165, 1979
5. Scholefield JH, Duncan JL, Rogers K. Review of hospital experience of breast
abscesses. Br J Surg 74:469, 1987.
61.- Se trata de paciente femenino de 37 aos con diagnstico de endometriosis, en sta
patologa podemos encontrar elevacin srica de:
a)
b)
c)
d)

CA-125.
Tc 99.
AFP.
TGP.

Beck W. (1997) NMS. Obstetrics and Gynecology. USA. Williams & Wilkins. Pag 278. Se
encuentra elevacin srica de CA-125 en pacientes con endometriosis, se trata de un
antgeno de superficie encontrado en derivados del epitelio celmico, incluyendo el
endometrio, se correlacionan sus niveles con el grado de enfermedad y la respuesta a
tratamiento, pudiendo ser un marcador de recurrencia de la enfermedad.

62.- Una mujer de 67 aos refiere intenso prurito vulvar y sensacin quemante, al examen
el introito vaginal se encuentra estentico. Cul de los siguientes es el tratamiento
apropiado?
a)
b)
c)
d)

5-fluoracilo.
Testosterona tpica.
Corticoesteroides fluorados.
Estrgeno tpico.

Vulvovaginitis atrfica
El hipoestrogenismo conduce a atrofia de la vagina y el vestbulo vulvar, que los hace
fcilmente irritables y susceptibles a infecciones secundarias. Las pacientes refieren
sensacin de quemadura, prurito, disuria, hipersensibilidad y dispareunia. Puede
encontrarse al examen fsico atrofia, fisuras superficiales, y un flujo vaginal acuoso1. Hay
disminucin del tamao del introito2, prdida de la rugosidad y la vagina toma una apariencia
lisa y brillante.
Los hallazgos histolgicos revelan un epitelio vaginal delgado, disminucin de los lechos
capilares, y la citologa muestra, a medida que la atrofia progresa, aumento de las clulas
basales y disminucin o ausencia de las clulas superficiales2.

Se aconseja evitar el uso de jabones y dems irritantes de la piel. Se pueden utilizar


lubricantes simultneamente con los estrgenos o como terapia nica, si hay alguna
contraindicacin a las hormonas.
El tratamiento con estrgenos por va sistmica o transvaginal mejora y restaura los signos
y sntomas, y una a dos semanas despus de iniciar el tratamiento los cambios de atrofia
empiezan a mejorar rpidamente, se reduce el pH y se induce maduracin vaginal y de la
mucosa uretral, reduciendo la frecuencia de las infecciones urinarias3. La dosis y va de
administracin debe ser debidamente individualizada4. Contraindicaciones al tratamiento
con estrgenos, incluyen: la presencia de tumores estrgenosensibles, falla heptica
terminal y antecedentes de tromboembolizacin relacionada con ellos.
Menopausia y Piel. Parte II: Manifestaciones clnicas
dermatolgicas durante la menopausia.
MARA ISABEL BARONA C. Docente adjunto. Dermatloga Universidad del Valle-Cali.

63.- Femenino de 22 aos que cursa con amenorrea secundaria la cual presenta
menstruacin posterior a la administracin de progestgenos, lo ms probable es que tenga:

a)
b)
c)
d)

Estrgenos bajos.
Estrgenos normales.
Progesterona elevada.
Gonadotropinas altas.

La presencia de estrgenos se puede establecer de dos maneras: con la prueba de desafo


con acetato de medroxiprogesterona (AMP) y el ndice de maduracin vaginal. Si hay
estrgenos, luego de la administracin de 10 mg de AMP diarios por 10 das debera
presentarse un sangrado por vagina. Y de esta forma corroborar los niveles de estrgenos
serian normales.
La ausencia de esta metrorragia sugiere una obstruccin o un
hipogonadismo. La otra alternativa es realizar el ndice de madurez vaginal que es un
procedimiento sencillo que puede obtenerse en el consultorio. Usando una esptula como las
de Papanicolaou el gineclogo extrae una muestra de la vagina y la extiende en un
portaobjeto. Este portaobjeto se procesa y se estudia en el microscopio. Un extendido
maduro tendr clulas epiteliales superficiales grandes y numerosas con un ncleo rodeado
por una gran cantidad de citoplasma. Por el contrario, una muestra no estrognica tendr un
mayor nmero de clulas parabasales y basales con ncleo grande rodeado por escaso
citoplasma.

Speroff L, Glass RH, Kase NG, eds. Clinical Gynecologic Endocrinology and Infertility, 5th
ed. Baltimore: Williams & Wilkins; 1994:334-335.

64.-En el servicio de consulta externa recibe un frotis vaginal el cual reporta la presencia
al microscopio de clulas clave las cuales son propias de infeccin por:
a) Cndida albicans.
b) Gardnerella.
c) Tricomonas.
d) Gonococos.

La infeccin por gardnerella (bacteria) tiende a producir una secrecin blanca, gris o de
color amarillo turbio, con un olor ftido o a "pescado" que aumenta cuando la secrecin se
vuelve alcalina, como sucede despus del coito o de lavarse con jabn. Puede haber prurito
o irritacin vulvar, pero por lo general no son muy pronunciados. Al microscopio se aprecia
las famosas celulas clave. Realizando el diagnostico diferencial.

Clulas escamosas de capa intermedia alta, algunas de ellas con ncleos picnticos, y dos de
ellas con el citoplasma cubierto por formas cocceas que borran sus bordes, dndole el
aspecto de clula rebozada "clula clave". Gardnerella. .

Pernoll M. Enfermedades de trasmisin sexual. En: Manual de Obstetricia y Ginecologa.


Benson/Pernoll Editores.Editorial Interamericana S A. Mxico DF, 1994

65.- Femenino de 33 aos de edad, que cursa con 8 SDG, portadora de Diabetes MellitusInsulino Dependiente de 10 aos de evolucin, entre las siguientes, la afirmacin correcta
es:
a) El riesgo de aborto espontneo es elevado, independientemente del grado de control de
la glucemia.
b) Se debe hacer una determinacin de Hb glucosilada en la primera visita obsttrica
(Hb A1).
c) Se debe hacer amniocentesis en el segundo trimestre para determinar cariotipo y
alfafetoprotena.
d) En el segundo trimestre habr que evaluar conjuntamente la alfafetoprotena en suero
materno, beta-HCG y el estriol no conjugado.

En los ltimos decenios se ha visto un cambio notable en el pronstico del embarazo cuando
ste se asocia a diabetes.
Con la introduccin de la insulina en la prctica mdica, hace poco ms de 60 aos, la
mortalidad materna se redujo en forma rpida y espectacular, en cambio la
morbimortalidad neonatal ha tenido una disminucin lenta y gradual, logrando ser reducida
a cifras inferiores al 5%.
Gran parte de la mortalidad actual del hijo de madre diabtica, se relaciona con
malformaciones congnitas en cuya prevencin se concentra la investigacin actual.
El embarazo supone una pesada carga para las diabticas, debido a los efectos
diabetognicos que posee en s ste estado.
stas pacientes tienen una tendencia a la inestabilidad metablica y precisan
monitorizacin frecuente, tratamiento estricto y un ritmo de vida muy controlado. (3)
En las pacientes diabticas que ya tienen lesin orgnica, el embarazo puede acelerar el
proceso, por lo que precisan un control y un tratamiento intensivo.(3)
Debido a lo anterior es que lo ideal es que reciba asesoramiento antes de que quede
embarazada.
Importancia de controlar la Glicemia: El control inadecuado de la glicemia suele aumentar la
incidencia de abortos en el primer trimestre, anomalas congnitas fetales, macrosoma
fetal, polihidroamnios y fetos nacidos muertos. (1)

El perodo de mayor riesgo de malformaciones fetales es el perodo periconcepcional y el


de la organognesis, ste, riesgo se puede reducir mediante un adecuado control
metablico.(3)
A su vez, la administracin de cido Flico antes del embarazo y durante el primer
trimestre, en dosis de 400 ug/da, previene la aparicin de defectos del Tubo /Neural
Neur

Importancia del control fetal durante el embarazo : Debido a la posibilidad de Embriopata


Diabtica, es necesario efectuar una determinacin de Hemoglobina Glicosilada en etapas
precoces del embarazo, Ecografa Obsttrica a la semana 20 y un Ecocardiograma Fetal a
la semana 24 para descartar malformaciones fetales ms habituales.(3)

La HbA1c, tambin conocida como hemoglobina glicosilada o glicada, glucohemoglobina


o HbA1, es un trmino utilizado para describir una serie de componentes estables
minoritarios de la hemoglobina que se forman lentamente y sin intervencin enzimtica,
a partir de la hemoglobina y la glucosa. La velocidad de formacin de la HbA1c es
directamente proporcional a la concentracin ambiente de glucosa. Como los eritrocitos
son fcilmente permeables a la glucosa, el nivel de la HbA1c en una muestra de
sangre facilita la historia glucmica de los 120 das anteriores, duracin media de la
vida de estas clulas. En particular, la HbA1c refleja de una forma bastante exacta
la glucemia en los 2-3 meses anteriores al anlisis.
Bibliografa:
1. Cunningham F, MacDonald P, Gant tN. N. Diabetes. Williams Obstetricia, Editorial
Mdica Panamericana,20 0a ed ed.1998;52:1119 .1998;1119-1136.
2. Oyarzn E.Patologa Mdica del Embarazo. Alto Riesgo Obsttrico.Impresos
Universitarios S.A, 2 ed ed.1997;177 .190.
3. Arias F.Gua Prctica para el Embarazo y el Parto de Alto Riesgo. Mosby Doyma
Libros.Segunda Edicin.1995;15:284 -303.
4. Catalano P, Feudtner C, Rosenn B.Diabetes y Embarazo. Clinicas Obstetricas y
Ginecologicas , McGraw Hill Interamericana,Primera Edicin.Volumen 1 /2000;1 1-14.-Guas
Perinatales ao 2002.Diabetes y Embarazo.Ministerio de Salud Chile; Pginas:22 39.
5.Garner P. Type I diabetes mellitus and pregnancy .The Lancet 1995;346:157 157-161.
6. Barrett J., Salyer S., Boehm F. The nonsstress test: An evaluation of 1000 patients Am
J Obstet Gynecol 1981;141:153 -157.
7. Dicker D., Feldberg D., Yeshaya A., Peleg D., Karp M., Goldman J. Fetal surveillance in
insulin dependent diabetic pregnancy :Predictive value of the biophysical profile .Am
Obstet Gynecol 1988;159:800 804.

66.- Mujer de 26 aos, es atendida en consulta en la clnica de displasias por papanicolau


con lesin NIC I. Antecedentes: menarca 14 aos, ritmo 30x5 eumenorreica, inicio de vida
sexual a los 15 aos, 2 parejas sexuales, mtodo de planificacin familiar oclusin tubaria
bilateral, gestas 3 partos 3, crvix con lesin acetoblanca con extensin lineal de 2 cm.
para confirmar el diagnstico se debe realizar:

a)
b)
c)
d)

Crioterapia de lesin.
Biopsia de la lesin.
Captura de hbridos.
Repetir colposcopia.

9.5.2 Las pacientes a quienes se les realiz citologa cervical, cuyo resultado es LEIBG
(infeccin por VPH, displasia leve o NIC 1); LEIAG (displasia moderada y grave o NIC 2 y 3)
o cncer deben enviarse a una clnica de colposcopa, para realizar estudio colposcpico.
9.5.3 Si el resultado de la citologa es LEIBG, la colposcopa es satisfactoria y sin
evidencia de LEIBG, se realizar control citolgico en un ao (Apndice Normativo A)
9.5.4 Si la citologa es de LEIBG, la colposcopa es satisfactoria y existe evidencia de
lesin, se debe tomar una biopsia dirigida.
9.5.4.1 Si la biopsia dirigida es negativa, se realizar nueva colposcopa para verificar el
diagnstico y en caso necesario, tomar nueva biopsia dirigida y revalorar.
9.5.4.2 Si la biopsia dirigida es reportada como LEIBG se podr dar tratamiento
conservador: criociruga, electrociruga o laserterapia (slo si cumple con las condiciones
referidas en el Apndice 1) o se podr mantener a la paciente en vigilancia en la clnica de
colposcopa, con colposcopa y estudio citolgico cada seis meses, durante 24 meses.
Jueves 31 de mayo de 2007 DIARIO OFICIAL (Primera Seccin)
9.5.4.3 Si la biopsia dirigida es reportada como LEIAG (Lesin Intraepitelial Escamosa de
Alto Grado) se realizar tratamiento conservador (electrociruga o laserterapia). En las
mujeres posmenopusicas, dependiendo de las condiciones anatmicas del crvix, se
realizar tratamiento conservador en la clnica de colposcopa o tratamiento quirrgico
(histerectoma extrafascial) en el servicio que corresponda.
9.5.4.4 Si la biopsia dirigida reporta cncer microinvasor o invasor, la paciente se
transferir a un Servicio o Centro Oncolgico para su tratamiento correspondiente.
9.5.4.5 Si la citologa reporta LEIBG y la colposcopa es no satisfactoria, se tomar
cepillado endocervical (Apndice Normativo A)
9.6 En caso de colposcopa no satisfactoria, negativa a LEIBG y con cepillado endocervical
negativo, se continuar su control en la clnica de colposcopa en seis meses, con colposcopa
y citologa.
9.6.1.1 Si el cepillado endocervical reporta LEIBG se tratar a la paciente como LEIAG,
con mtodos conservadores escisionales.

Jueves 31 de mayo de 2007 DIARIO OFICIAL (Primera Seccin)


Modificacin a la Norma Oficial Mexicana NOM-014-SSA2-1994, Para la prevencin,
deteccin, diagnstico, tratamiento, control y vigilancia epidemiolgica del cncer
crvico uterino.
Al margen un sello con el Escudo Nacional, que dice: Estados Unidos Mexicanos.- Secretara
de Salud.
MODIFICACION A LA NORMA OFICIAL MEXICANA NOM-014-SSA2-1994, PARA LA
PREVENCION,
DETECCION, DIAGNOSTICO, TRATAMIENTO, CONTROL Y VIGILANCIA
EPIDEMIOLOGICA DEL CANCER CERVICO UTERINO.

7.3 El resultado del estudio citolgico es descriptivo y debe ser informado de la siguiente
manera:
a.- Negativo a cncer.

b.- Negativo con proceso inflamatorio.


c.- Displasia leve (NIC 1).
d.- Displasia moderada (NIC 2).
e.- Displasia grave (NIC 3).
f.- Cncer del cuello del tero in situ (NIC 3).
g.- Cncer microinvasor e invasor.
h.- Adenocarcinoma.
i.- Maligno no especificado.

67.- Mujer con diagnstico de amenorrea que acude con resultados de laboratorio los que
reportan los siguientes niveles hormonales: GnRH elevada, FSH y LH elevadas, hormonas
ovricas (estrgenos y progesterona) bajas. El defecto est a nivel de:

a)

Hipotlamo.

b)

Hipfisis.

c)

Ovario.

d)

Endometrio.

Evaluacin de la paciente con amenorrea secundaria


La mayora de las pacientes con AS que no estn embarazadas o no entraron en el
climaterio tienen una alteracin en algn nivel de la cascada reguladora del ciclo menstrual
femenino. A continuacin, esquematizamos la evaluacin de la AS en pasos. Cada mdico
deber adaptar los tiempos de la evaluacin a la situacin particular de cada paciente. No
obstante, recomendamos no saltear ninguno de estos pasos ya que un diagnstico preciso
permitir realizar un tratamiento racional y dar un pronstico respecto de la futura
funcin menstrual y de la fertilidad.

Primer paso (descartar el embarazo)

Su prueba se basa en el dosaje cuali o cuantitativo de la subunidad beta de la


gonadotrofina corinica humana. Si se certifica la ausencia de embarazo podr avanzarse al
siguiente paso, pero teniendo en cuenta siempre que si la probabilidad de embarazo es muy
alta se deber repetir la prueba nuevamente. Se recomienda no obviar esta prueba aunque
la paciente asegure que no ha mantenido relaciones sexuales.
Segundo paso (prueba de progesterona)

Debe realizarse slo si el test de embarazo es negativo y consiste en dar


medroxiprogesterona entre 30 a 50mg por va oral (un comprimido de 10mg durante 5 das)
o progesterona oleosa 100 a 200mg por va intramuscular en una sola dosis. El resultado de
esta prueba provee informacin acerca de si se produjeron estrgenos. La prueba se
considera positiva si se produce un sangrado luego de 2 a 14 das de la suspensin de la
progesterona. La respuesta positiva indica que existe integridad anatmica del aparato
genital femenino y que el ovario produce estrgenos. En estos casos, la AS se debe a que no
ha habido ovulacin. La causa ms frecuente de AS con prueba de progesterona positiva es
la disfuncin hipotalmica leve. En este caso, no se dispara el pico de LH necesario para
producir la ovulacin, no se ovula (ciclo anovulatorio), no hay cuerpo lteo y no hay
produccin de progesterona. El sistema reproductor queda en un estado folicular, con
grados variables de desarrollo folicular, acompaados de concentraciones cambiantes de
estradiol. Puede producirse hemorragia en ausencia de ovulacin dado que el endometrio
proliferativo sufrir ocasionalmente un desprendimiento parcial en respuesta a estos
niveles cambiantes de estradiol. Estos sangrados luego de un ciclo anovulatorio se
denominan sangrados uterinos disfuncionales (SUD). Como dijimos, ejemplos frecuentes de
situaciones de anovulacin son el estrs, la prdida brusca de peso y el ejercicio intenso.
Otras causas menos frecuentes son la hiperprolactinemia, el hipotiroidismo y el sindrome
del ovario poliqustico. Otras veces no hay una causa clara (idioptica) de este trastorno.
La prueba se considera negativa si NO se produce un sangrado luego de 2 a 14 das de la
suspensin de la progesterona, lo que ocurre cuando los niveles de estrgenos son bajos
(menores a 40 pg/ml). La causa ms frecuente de prueba negativa es la insuficiencia
gonadal de la menopausia.
Tercer paso

Para avanzar al tercer paso es preciso distinguir si la prueba de progesterona fue positiva
o negativa.

Pacientes con prueba de progesterona positiva

Una prueba de progesterona positiva hace diagnstico de ciclos anovulatorios. Segn los
antecedentes, el examen clnico y la probabilidad previa el mdico podr orientarse hacia
cul es el diagnstico. Hay situaciones de estrs, crisis vitales, cambios de peso brusco o
ejercicio extremo que ocasionan un trastorno del ciclo aislado. En estos casos, la prueba de
progesterona es diagnstica y teraputica. Es decir, la paciente no requiere estudios
posteriores. Es conveniente reasegurarla y explicarle que su periodo se normalizar una vez
solucionado el problema. En una paciente con trastornos anovulatorios frecuentes, que
presenta obesidad, acn e hirsutismo, existe alta sospecha de sndrome de ovario
poliqustico (SOP). En este caso, debemos solicitar una ecografa pelviana y un dosaje de
LH/FSH o de hormonas masculinas: testosterona libre (To), dehidroepiandrosterona
sulfato (DHEA-S) y 17OH progesterona.
Si la paciente tiene trastornos del ciclo y galactorrea, se debe solicitar un dosaje de
prolactina (PRL). El valor normal de PRL para la mujer no embarazada es de 20 a 25ng/ml.
Cuando el valor de PRL es menor de 100ng/ml, generalmente es de causa idioptica o
farmacolgica; si est entre 100 y 200ng/ml, puede ser por causa farmacolgica o por
enfermedad hipotalmica; si es mayor de 200ng/ml, la causa ms probable es el adenoma
hipofisario. Cuando el valor de la PRL es mayor de 100ng/ml, se debe solicitar un estudio
por imgenes que puede ser una tomografa computada de cerebro con contraste o una
resonancia magntica nuclear con gadolinio. Si se observa una imagen menor de 10mm, se
trata de un microadenoma de hipfisis y si es mayor de 10mm, se hace diagnstico de
macroadenoma. Si la paciente consulta por trastornos del ciclo frecuentes y no presenta
galactorrea o signos o sntomas de hipotiroidismo, igualmente deber solicitarse un dosaje
de tirotrofina (TSH) y de prolactina (PRL) sricas.

Pacientes con prueba de progesterona negativa

Deber dosarse el nivel srico de FSH cuyo valor refleja los niveles de estrgenos
circulantes (por el fenmeno de retroalimentacin negativa). Si la FSH es mayor de 40
UI/ml, los niveles de estrgenos son bajos. En este caso, se asume que existe una
insuficiencia ovrica. El valor de la FSH puede hacer el diagnstico diferencial entre la
falla ovrica (castracin temprana, menopausia precoz o menopausia normal) y la disfuncin
hipotlamo-hipofisaria severa. En la primera, la ausencia de retroalimentacin negativa
estimula la liberacin de las gonadotrofinas y, en consecuencia, stas se encuentran
elevadas en la sangre. En la segunda, los valores de gonadotrofinas son bajos debido a un
defecto en su produccin. El dosaje de FSH es ms sensible que el de LH para este
propsito, por lo tanto, si la prueba de progesterona es negativa y la FSH es baja, lo ms
probable es que la causa sea una disfuncin hipotlamo-hipofisaria severa. En este caso
debera solicitarse un estudio de diagnstico por imagen del cerebro (RMN con gadolinio o
TAC con contraste). En el cuadro 4 se muestran los valores normales de las hormonas
mencionadas.

Cuadro 4: valores hormonales normales

PRL (prolactina): 20 a 25ng/ml en mujeres no embarazadas.

TSH: de 0.5 a 5mUI/ml.

FSH: 5 a 30mUI/ml.

LH: 5 a 20mUI/ml (en el pico ovulatorio este valor se debe multiplicar por dos o tres).

Falla ovrica: FSH mayor de 40mUI/ml.

Disfuncin hipotlamo hipofisaria: FSH menor a 5mUI/ml y LH menor a 5mUI/ml.


En general, una paciente con disfuncin hipotalmica con gonadotrofinas bajas est
expresando una gravedad mayor que la disfuncin hipotalmica leve con prueba de
progesterona positiva.

En los casos en los que la prueba de progesterona es negativa, la FSH es normal, la paciente
no est en una edad cercana a la menopausia y existe alta sospecha de que exista una
enfermedad uterina, se recomienda realizar una prueba de estrgenos y progesterona.
Esta prueba sirve para conocer si el efector (el tero) responde a los estmulos hormonales.
Consiste en administrar estrgenos (1.25mg por da durante 20 das) ms progesterona
(10mg por da durante los ltimos 5 das en que se administran los estrgenos). Si no hay
sangrado, se debe repetir la prueba y, si nuevamente no hay sangrado, la prueba se
considera negativa. Esto indica que hay una falla en el efector, o sea, en el tero.
En este caso la paciente debe ser derivada al gineclogo. Las causas ms probables de falla
uterina son la endometritis por abortos o partos spticos y las sinequias por curetajes
vigorosos. Cuando falla el efector, la ecografa transvaginal puede ser normal. Por eso debe
realizarse una prueba de estrgenos y progesterona para evaluar directamente la
funcionalidad del tero. La prueba se considera positiva si hay sangrado (aunque sean slo
gotitas). Esto significa que el tero est indemne para responder a estmulos hormonales y
el problema es de origen hipotlamo- hipofisario.

Bibliografa:
1.

Jonathan R, Pletcher, Gail B. Slap. Menstrual Disorders Amenorrhea. Pediatric


Clinics of North America. June 1999; 46: Issue 3.
2. Bryan McIver, Susan A Romanski; Todd B Nippoldt. Evaluation and Management of
Amenorrhea. Mayo Clinic Proceedings Dec 1997; 72: 1161-1169.
3. Tarannun Master Hunter, Diana L.Heiman. Amenorrhea: Evaluation and Treatment.
Am Fam Physician 2006; 73: 1374-1382.
4. The Practice Committee of the American Society for Reproductive Medicine.
Current Evaluation of Amenorrhea. Fertility and Sterility Nov 2006: 86 Supl- 4:
148-155.

68.- Es el sndrome que ms comnmente es causa de amenorrea primaria:


a)
b)
c)
d)

Turner.
Feminizacin testicular.
Rokitansky.
Asherman.

Amenorreas primarias
Se considera amenorrea primaria a todos los casos en que la menarquia no haya tenido
lugar antes de los 16 aos (el 97% de las mujeres tienen la menarquia antes de los 15 aos y
medio) o antes de los 14 aos si no tiene caracteres sexuales secundarios .La causa ms
frecuente de amenorrea primaria es la disgenesia gonadal (defectuosa formacin de los
ovarios).
Las principales causas de amenorrea primaria incluyen el sndrome de Turner, el sndrome
de insensibilidad andrognica y la agenesia tero-vaginal. Para el diagnstico, es prctico
relacionar este sntoma con el grado de desarrollo puberal, la talla y con la presencia o no
de anormalidades en el tracto genital y es til la siguiente clasificacin, que ser revisada
desde el punto de vista gentico:
A)
Amenorrea con retraso puberal: Hipogonadismo hipergonadotrpico: sndrome de
Turner (ST), sndrome de Swyer (XYGD), sndrome de Perrault y otras formas de
disgenesia gonadal XX (XXGD). Hipogonadismo hipogonadotrpico: sndrome de Kallmann
(KS)
B)
Amenorrea con pubertad normal: Sndrome de ovario poliqustico (SOP) Hiperplasia
adrenal congnita no clsica
Sndrome de Mayer-Rokitansky-Kster-Hauser (MRKH) Sndrome de insensibilidad a los
andrgenos (SIA)

ReferenciaS bibliogrficas:
1. ASRM. Practice Committee of the American Society for Reproductive Medicine. Current
evaluation of amenorrhea. Fertil Steril. 2004;82 Suppl 1:S33-9.
2. Pletcher JR, Slap GB. Menstrual disorders. Amenorrhea. Pediatr Clin North Am.
1999;46(3):505-18.
3. Iglesias EA, Coupey SM. Menstrual cycle abnormalities: diagnosis and management.
Adolesc Med. 1999;10(2):255-73.
4. Aloi JA. Evaluation of amenorrhea. Compr Ther. 1995;21(10):575-8.
5. Mazzanti L, Cicognani A, Baldazzi L, Bergamaschi R, Scarano E, Strocci S, Nicoletti A, E.
Gonadoblastoma in Turner syndrome and Y-chromosome-derived material. Am J Med Genet.
2005;135(2):150-4.

69.-En la sala de urgencias recibe a una paciente que inicia con convulsiones por
preclampsia usted decide administrar el siguiente frmaco ya que es el de eleccin en
sta patologa:

a) Sulfato de magnesio.
b) Diacepam.
c) Fenitona.
d) Donadores de xido ntrico.

Manejo de la Preeclampsia
1. Manejo ambulatorio: HTA sin proteinuria significativa, se recomienda el reposo en cama.
Monitoreo de TA, peso, presencia de protenas en orina. Ecografas peridicas para ver el
feto y evaluar posibles retardo de crecimiento.
2. Manejo hospitalario: para mujeres con HTA inducida por el embarazo y 2+ o ms o
proteinuria significativa y en quienes fall el manejo ambulatorio.
3. Laboratorio y evaluacin del peso: debe realizarse diariamente. Evaluacin de la
dinmica fetal. Monitoreo de sntomas como cefalea, alteraciones visuales y dolor
epigstrico.
4. El parto es el tratamiento de eleccin: el cual debe realizarse cuando el feto est
maduro pero puede realizarse en forma temprana si la salud de la madre est en peligro o si
hay evidencia de distress fetal. El parto est indicado cuando la paciente cumple con los
criterios de preeclampsia severa. Betametasona 12.5 mg IM dos veces por da puede
estimular la maduracin de los pulmones fetales.
5. Terapia antihipertensiva: est indicada slo si la TA es persistentemente > 160/110 , es
importante disminuir la TA hasta una diastlica de 90 a 100 mmHg porque la presin normal
podra resultar en hipoperfusin de la placenta. Los diurticos nunca estn indicados, estas
pacientes ya son hipovolmicas. Los IECA no deben ser usados durante el embarazo. Las
medicaciones de largo plazo, incluyen alfa metildopa, atenolol y labetalol.
6. Terapia anticonvulsivante:

A- Profilaxis de las convulsiones: est indicada en todas las pacientes pre-eclmpticas


durante el trabajo de parto y el parto y por un mnimo de 24 hs luego del mismo. Algunos
mantienen la terapia con magnesio hasta que comienza la diuresis. El Sulfato de Magnesio
es la droga de eleccin. La dosis profilctica es de 4 a 6 g de sulfato de magnesio IV y
contina con 2 g c/ hora.
B- Tratamiento de las convulsiones: Sulfato de Magnesio 1 g/min IV hasta controlar
las convulsiones hasta un mximo de 4 a 6 g. El nivel teraputico es de 4 meq/l. Toxicidad
del magnesio: ausencia de reflejo patelar, debilidad muscular, parlisis respiratoria y
depresin cardaca, 10 ml al 10 % de gluconato de calcio puede ser administrada IV. La
terapia con sulfato de magnesio contina por lo menos 24 horas en el post parto, la terapia
puede detenerse si la excrecin urinaria es > 200 ml/h por cuatro horas consecutivas.
C- Prevencin: 81 mg de aspirina diarios pueden ser administrados luego del primer
trimestre en mujeres con hipertensin crnica o historia previa de preeclampsia, sin
embargo la eficacia de esta indicacin ha sido cuestionada.

Referencia Bibliogrfica:

1. Myers JE, Baker PN. Hupertensive diseases and eclampsia. Curr Opin Obstet Gynecol
2002; 14: 119-125.
2. Tierney, McPhee, Papadakis. Diagnstico clnico y tratamiento 2003. 38 ed, Mxico,
Manual Moderno, 2003: 770-773.
3. Wilson MI, Goodwin TM, Pan VI, Ingles SA. Molecular epidemiology of preeclampsia.
Obstet and Gynecol Survey 2003; 58(1):39-66.
4. Burrow GM. Complicaciones mdicas durante el embarazo. 4 ed, Mxico, McGraw-Hill
panamericana: 1996: 1-25.
5. Guyton AC, Hall JE. Embarazo y lactancia en: Tratado de fisiologa mdica, 10 ed,
Mxico, McGraw-Hill Interamericana 2001: 1135-45.
6. Vaticon D. Fisiologa de la fecundacin, embarazo, parto y lactancia, en: Tresguerres
JAF. Fisiologa Humana. Mxico, Interamericana McGraw-Hill, 1992: 1086-1109.
7. Pridjian G, Puschett JB. Preeclampisa. Part 1: Clinical and Pathophysiologic
Considerations. Obstet and Gynecol Survey 2002; 57 (9): 598-618.
8. Pridjian G, Puschett JB. Preeclampisa. Part I1: Experimental and Genetic Considerations.
Obstet and Gynecol Survey 2002; 57 (9): 619-40.
9. IMSS. Embarazo de alto riesgo. Gua diagnstica teraputica. Rev Med IMSS 1998;
36(1):45-60.

70.- Se trata de paciente de 34 aos que cursa con 39 SDG; a la exploracin fsica
reflejos patelares hiperactivos, inquieta, se reportan cifras de TA 145/95, se realiza
laboratorio que reporta proteinuria 2+,. El diagnstico ms probable es:

a) Glomerulonefritis aguda.
b) Hipertensin esencial.
c) Feocromocitoma.
d) Preeclampsia.

La hipertensin es la complicacin mdica ms comn del embarazo , aunque para algunos


autores es la segunda complicacin mdica del embarazo slo despus de la anemia; es ms
frecuente en jvenes durante el primer embarazo y en nulparas de mayor edad,
hipertensas previas y diabticas.
En Mxico, tambin es la complicacin ms frecuente del embarazo, la incidencia es de 47.3
por cada 1 000 nacimientos y es adems, la primera causa de ingreso de pacientes
embarazadas a las unidades de terapia intensiva (debido a hemorragia masiva, para recibir
soporte hemodinmico), segn la secretara de salud (2001) la mortalidad por
complicaciones del embarazo ocupa el 15 lugar en la mortalidad hospitalaria en general.
Adems, la tasa de preeclampsia se ha incrementado 40% en el periodo entre 1990 y 1999 y
constituye hasta 40% de los partos prematuros iatrognicos.
.

Preeclampsia
La preeclampsia es un sndrome clnico caracterizado por hipertensin con disfuncin
orgnica mltiple, proteinuria, edemas.
Es definida como un incremento de al menos 140/90 mmHg despus de la semana 20 de
gestacin, un incremento en la presin sangunea diastlica de al menos 15 mmHg respecto a
un nivel previo a la semana 20 combinado con proteinuria (> 300 mg en 24 horas). Las
mediciones de la presin arterial citadas deben ser medidas al menos 2 ocasiones con por lo
menos 6 horas de separacin. La proteinuria puede ser una toma simple de orina al azar que
indique al menos 30 mg/dL 3 ++ en dos muestras de orina1 segn el tipo de prueba. El
criterio del incremento de 30 mmHg en la presin sistlica y/o 15 mmHg en la presin
diastlica respecto a valores previos a la semana 20 de gestacin ha sido eliminado por ser
poco especfico15

1.Myers JE, Baker PN. Hupertensive diseases and eclampsia. Curr Opin Obstet Gynecol
2002; 14: 119-125.
2. Tierney, McPhee, Papadakis. Diagnstico clnico y tratamiento 2003. 38 ed, Mxico,
Manual Moderno, 2003: 770-773.
3. Wilson MI, Goodwin TM, Pan VI, Ingles SA. Molecular epidemiology of preeclampsia.
Obstet and Gynecol Survey 2003; 58(1):39-66.
4. Burrow GM. Complicaciones mdicas durante el embarazo. 4 ed, Mxico, McGraw-Hill

panamericana: 1996: 1-25.


5. Guyton AC, Hall JE. Embarazo y lactancia en: Tratado de fisiologa mdica, 10 ed,
Mxico, McGraw-Hill Interamericana 2001: 1135-45.
6. Vaticon D. Fisiologa de la fecundacin, embarazo, parto y lactancia, en: Tresguerres
JAF. Fisiologa Humana. Mxico, Interamericana McGraw-Hill, 1992: 1086-1109.
7. Pridjian G, Puschett JB. Preeclampisa. Part 1: Clinical and Pathophysiologic
Considerations. Obstet and Gynecol Survey 2002; 57 (9): 598-618.
8. Pridjian G, Puschett JB. Preeclampisa. Part I1: Experimental and Genetic Considerations.
Obstet and Gynecol Survey 2002; 57 (9): 619-40.
9. IMSS. Embarazo de alto riesgo. Gua diagnstica teraputica. Rev Med IMSS 1998;
36(1):45-60.

71.- Las hiper y polimenorreas o prdidas de sangre continuas sin conservacin del
ciclo, son ms frecuentes en los miomas de localizacin:

a) Submucoso.
b) Intramural.
c) Intraligamentaria.
d) Cervical.

Descripcin
Los miomas son tumores monoclonales benignos de las clulas del msculo liso del miometrio.
Estn compuestos por grandes cantidades de matriz extracelular que contiene colgeno,
fibronectina y proteoglicanos. El colgeno tipo I y tipo II es abundante, pero las fibrillas
de colgeno son anormales y estn desorganizadas, de modo similar a lo que se observa en
la formacin de queloides.
Los miomas submucosos son los menos frecuentes, constituyendo nicamente el 5% de la
totalidad de los miomas, a menudo producen un aumento del sangrado menstrual en forma
de hiper y polimenorreas, e incluso hemorragias importantes que exigen tratamiento de
urgencia. Las metrorragias tambin son habituales en este tipo de miomas. Segn Novak, el
peligro de degeneracin sarcomatosa es mucho mayor en los miomas submucosos, e
igualmente es causa de dismenorreas ms intensas y frecuentes.

BIBLIOGRAFA:
1. De la Fuente U. Tratado de Obstetricia y Ginecologa. Mc Graw-Hill. (Madrid). 1998.
Volumen II.
2. DI SAIA S. Tratado de Obstetricia y Ginecologa de Danforth. Sexta. Mc Graw-Hill.
(Nueva York). 1990. Sexta Edicin.
3. Gonzlez-Merlo J. Tratado de Obstetricia y Ginecologa. Salvat Editores S.A.
(Barcelona). 1990. Quinta Edicin.
4. Aller J., Pages G. Obstetricia Moderna. Tercera Edicin. Mc Graw-Hill. (Caracas). 1999.
5. Formacin Mdica Continuada en Atencin Primaria. Marzo 1995. Vo. 5, N (3).

72.-Femenino de 26 aos la cual presenta abortos espontneos recurrentes, Cul es el


factor etiolgico ms comn de aborto espontneo del primer trimestre?
a)
b)
c)
d)

Anormalidades cromosmicas del embrin.


Anormalidades hormonales de la madre.
Malformaciones uterinas.
Enfermedades sistmicas de la madre.

El aborto se presenta entre un 15-20% de todos los embarazos, el riesgo disminuye


conforme el embarazo progrese, es posible que tanto como el 50% de los abortos ocurren
antes de la implantacin en el tero, despus de la implantacin la tasa de prdida es de
30% (esto es antes de que sea reconocido clnicamente), despus de ser reconocido es de
25% y disminuye dramticamente despus de las 8 semanas.

Muchas son las causas de abortos las ms comunes son las anomalas cromosmicas, estas
cuentan con el 50% de los casos que se producen en el primer trimestre, si el aborto se
produce ms tarde hay tambin una tasa alta de anomalas cromosmicas, 30% en el
segundo trimestre y 5% en el tercer trimestre. La mayora de las anomalas cromosmicas
encontradas en los abortos espontneos son las trisomas autosmicas (50% de los
especmenes), en el cariotipo la que se ve con ms frecuencia en el abortos es 45 XO, la
gran mayora de esas anormalidades genticas son fenmenos espontneos aislados.
Se conoce bien la relacin entre la edad materna y la anormalidad cromosmica, hay un
aumento claro en la frecuencia de trisomas principalmente 13, 14, 15, 21 y 22 debido a la
edad materna, muchos de esos embarazos terminan en abortos temprano y contribuyen, en
forma importante al ndice elevado de interrupciones espontneas en esa poblacin de
pacientes.

Bibliografa:
1. Laing FC, Frapes MC. Evaluacin ecogrfica durante el primer trimestre del embarazo.
En: Callen PW. Ecografa en Obstetricia y Ginecologa. 4a. edicin. Buenos Aires: Editorial
Mdica Panamericana; 2002. p. 126-34.
2. Lomax B, Tang S, Separovic E, Phillips D, Hillard E, Thomson T, et al. Comparative
genomic hybridization in combination with flow cytometry improves results of cytogenetic
analysis of spontaneous abortions. Am J Hum Genet 2000; 66:1516-21.

73.- Mujer de 54 aos que presenta prolapso uterino e incontinencia urinaria de ezfuerzo
el procedimiento de eleccin en esta paciente es:

a)
b)
c)
d)

Histerectomia total abdominal.


Histerectoma vaginal reconstructiva.
Colpoperineoplasta.
Uterosuspensin.

El prolapso genital y su tratamiento ha sido siempre un importante captulo de la


ginecologa. No existe consenso clnico del concepto que define al prolapso genital como
patolgico. Cierto grado de descenso y relajacin de la pared vaginal es considerado normal
en la gran mayora de las mujeres, siendo ms frecuente en mujeres de mayor edad. En la
poblacin general solo un 3% presenta prolapso genital severo, entendiendo como tal al de
III y IV grado. Es probable que alrededor de 3 a 6% de la poblacin femenina desarrolle
un prolapso de esas caractersticas en algn momento de su vida
La intervencin quirrgica por excelencia para el tratamiento quirrgico del prolapso
uterino es la histerectoma vaginal. Esta intervencin se clasifica como "limpia-contaminada

INDICACIONES PARA LA HISTERECTOMA VAGINAL


Prolapso uterino
Hemorragia uterina disfuncional
Carcinoma cervico-uterino "in situ"
Miomatosis uterina poco voluminosa
Hiperplasia endometrial
Piometra

VENTAJAS DE LA HISTERECTOMA VAGINAL


No deja cicatriz abdominal
Mnimo trauma abdominal
Escasa hemorragia transoperatoria
Mnima manipulacion intestinal
Menos dolor postoperatorio
CUADRO
CONTRAINDICACIONES PARA LA HISTERECTOMA VAGINAL
Impericia
tero muy voluminoso
Tumores ovricos
Endometriosis
Enfermedad plvica inflamatoria
Ciruga previa en tero, trompas y ovarios
Cncer de endometrio
Histerectoma obsttrica

74.- Una mujer de 21 aos, nuligrvida, acude a consulta para hablar sobre anticoncepcin.
Es sexualmente activa desde hace 2 semanas y actualmente utiliza condn. Tiene
antecedente de asma, la cual se ha mantenido inactiva por 2 aos. No toma medicamentos y
niega alergias. No hay AHF de cncer. Su EF es normal. Despus de una pltica con su
mdico, escoge tomar anticonceptivos orales combinados, y contina tomndola por 6 aos.
Ahora ha disminuido su riesgo de desarrollar:
a)
b)
c)
d)

Cncer de mama.
Cncer cervical.
Cncer heptico.
Cncer ovrico.

PROTECCION CONTRA EL CANCER EPITELIAL OVARICO.


Debido a la falta de estrategias eficaces para el diagnostico y tratamiento temprano del
cncer de ovario, es de capital importancia la prevencin (76). Esta patologa es una
importante causa de morbilidad y mortalidad. Se estim que para 1980 ocurrieron 137.600
casos nuevos en el mundo (86). Estudios a gran escala realizados por el Centro para el
Control de las Enfermedades de EE.UU. y el Royal Collage of General Practitioners del
Reino Unido (RCGP) indican que la supresin de la ovulacin causada por los anticonceptivos
orales protege contra el desarrollo del cncer epitelial ovrico (20,76). Este efecto
benfico es directamente proporcional al tiempo de uso y persiste muchos aos despus de
suspendida la planificacin con este mtodo (87). La evaluacin norteamericana denominada:
Estudio sobre cncer y hormonas (CASH) demostr que el uso de uno a cinco aos de
anovulatorios orales disminuye en un 50 a 70% el riesgo de cncer ovrico (88). Este efecto
protector aumenta entre ms sea el tiempo de uso y se extiende por lo menos hasta diez
aos despus de interrumpido (70,89). La Organizacin Mundial de la Salud tambin realiz
un estudio multicntrico confirmando el efecto protector de los anticonceptivos orales
contra el cncer epitelial ovrico (90). Dos estudios de Cohortes realizados en Gran
Bretaa, confirmaron el efecto protector de la pldora al encontrar riesgos relativos de 0.3
y o.6 en mujeres que haban usado el mtodo en algn momento (83,91). El efecto protector
es tanto para tumores malignos como para Bordenline (92) y cada uno de los principales
subtipos histolgicos de cncer epitelial (70,93).

TABLA N 3
BENEFICIOS NO CONTRACEPTIVOS DE LOS ANTICONCEPTIVOS
COMBINADOS:
MEJORIA DE LA DISMENORREA.
CORRECCION DE LOS CICLOS MENSTRUALES IRREGULARES.
PREVENCION DE QUISTES OVARICOS FUNCIONALES.
PROTECCION CONTRA EL CANCER EPITELIAL OVARICO.
MEJORIA DEL MITTELSCHMERZ.
PROTECCION CONTRA EL CANCER ENDOMETRIAL.
PROTECCION CONTRA TUMORES BENIGNOS MAMARIOS.
DISMINUCION DE LA ENFERMEDAD PELVICA INFLAMATORIA.
DISMINUCION EN LA INCIDENCIA DE EMBARAZO ECTOPICO.
PREVENCION DE ANEMIA FERROPENICA.
MENOR INCIDENCIA DE ARTRITIS REUMATOIDEA.
MENOR INCIDENCIA DE OSTEOPOROSIS POST-MENOPAUSICA.
MEJORIA DEL SINDROME PREMENSTRUAL.
PREVENCION DE LA MIOMATOSIS UTERINA.
MEJORIA DEL ACNE.

Bibliografa:

1.
Bagshaw S. the combined oral contraceptives. Risk and adverse effects in
perspective. Drug-Saf 1995; 12 (2): 91 - 96.

ORALES

2.
American Collage of Obstetricians and Gynecologist. Hormonal contraception.
ACOG technical bulletin N. 198 - October de 1994 Int J Gynaecol Obstet 1995; 48 (1):
115 - 126.
3.
Mishell Jr DR. Oral contraception: past, present and future perspectives. Int J
Fertil 1992; 37 (1) Suppl: 7 - 18.
4.
Melo NR, Pinotti J.Advances in hormonal contraception. Adv. in contraception 1994;
10 (suppl 1): 33 - 39.
5.
Winkler UH, Schindler AE, Endrikat J, et al. A comparative study of the effects
of the hemostatic system of two monophasic Gestodene oral contraceptive containing 20
ug and 30 ug Etinil-Estradiol. Contraception 1996; 53: 75 - 84.
6.
Coenen CMH, Thomas CMG, Borm GF, et al. Changes in androgens during treatment
with four low-dose contraceptives. Contraception 1996; 53: 171 - 176
7.
Wilde MI, Balfour JA. Gestodeno. A review of its pharmacology, efficacy and
tolerability in combined contraceptive preparation. Drug 1995; 50 (2): 364 - 395.
8.
Monterrosa A. Anticoncepcin hormonal. EN : Caraballo J, Parra E, Taylor H.
Memorias del 1 Curso de actualizacin en Ginecologa y Pediatra. Imprenta U. de
Cartagena. Cartagena.1994; 241 - 250.
9.
Hannaford PC, Combined oral contraceptives : do we know all of their effects.
Contraception 1995; 51: 325-327.
10.Rosenberg MJ, Waugh MS, Meehan T. Use and misuse of oral contraceptives : risk
indicators for poor pill taking and discontinuation. Contraception 1995; 51: 283- 288.

75.- Femenino de 32 aos segunda gesta a trmino sin anormalidades en el transcurso de


ste. Inicia trabajo de parto de forma espontnea, con evolucin normal hasta que se
rompe la bolsa, con una dilatacin de 4 cm. A partir de entonces, comienza con hemorragia
de sangre roja, en moderada cantidad y aparecen signos de sufrimiento fetal agudo. El
estado general de la mujer es bueno y la dinmica uterina es normal. Este cuadro
corresponde a:
a) Abruptio placentae.
b) Rotura uterina.
c) Placenta previa central
d) Rotura de vasa previa.

La vasa previa es una condicin de alto riesgo obsttrico en la cual vasos fetales o
placentarios cruzan el segmento uterino por debajo de presentacin.
Estos vasos estn desprotegidos de gelatina de Wharton o tejido placentario, lo que los
hace altamente vulnerables y susceptibles de ruptura o laceraciones en cualquier perodo
del embarazo, principalmente en el momento del parto. Tambin es frecuente la compresin
de estos vasos, especialmente durante el tercer trimestre de la gestacin, lo que puede
condicionar asfixia y muerte fetal.

Esta condicin ocurre como resultado de que vasos velamentosos cruzan por el segmento
uterino debido a una insercin velamentosa del cordn, situacin en la cual el cordn
umbilical se inserta en las membranas ovulares en vez del tejido placentario (vasa previa
tipo I), o por el cruce de vasos fetalesentre uno o ms lbulos accesorios de la placenta
(vasa previa tipo II) (Figura 1).

Figura 1. A, Vasa previa tipo I, debida a insercin velamentosa.


B, Vasa previa tipo II, debida a cotiledones aberrrantes. Reproducido de Daly-Jones y cols.
Ultrasound 2008.

Vasa previa se puede presentar si existe alguna (o ninguna) de las siguientes condiciones:
placenta baja (que puede ser causa de abortos previos seguidos por legrado o por
operaciones uterinas, que provocan cicatrices en el tero), placenta bilobada o de lbulo
succensuriado, embarazos resultado de fertilizacin in vitro, o embarazos mltiples (5-6).
El sangrado por vasa previa no es doloroso. Otros sangrados por complicaciones o por
nacimiento no necesariamente son sin dolor.

Referencias
1. Oyalese Y, Smulian JC. Placenta previa, placenta acreta, and vasa previa. Obstet Gynecol
2006; 107:
927-941.
2. Oyalesse KO, Turner M, Less C, Campbell S. Vasa previa: an avoidable obstetric tragedy.
Obstet Gynecol Surv 1999; 54: 138-145.
3. Seplveda W, Sebire NJ, Harris R, Nyberg DA. The placenta, umbilical cord, and
membranas. In Diagnostic Imaging of Fetal Anomalies, Nyberg DA, MaGahan JP, Pretorius
DH, Pilu G (eds). Philadelphia, PA: Lippicont Williams & Wilkins 2003, 85-132.
4. Daly-Jones E, John A, Leahy A, McKenna C, Sepulveda W. Vasa praevia; a preventable
tragedy. Ultrasound 2006; 16: 8-14.
5. Derbala Y, Grochal F, Jeanty P. Vasa previa. J Prenat Med 2007; 1: 2-13.
6. Fung TY, Lau TK. Poor perinatal outcome associated with vasa previa. It is preventable?
A report of three cases and review of the literature. Ultrasound Obstet Gynecol 1998; 12:
430-433.

7. Robert JA, Sepulveda W. Fetal exsanguination from ruptured vasa previa: still a
catastrophic event in modern obstetrics. J Obstet Gynaecol 2003; 23: 574.
8. Cordero DR, Helfgott AW, Landy HJ, et al. A non-hemorrhagic manifestation of vasa
previa: a clinicopathologic case report. Obstet Gynecol 1993; 82: 698-700.
9. Schachter M, Tovbin Y, Arieli S, et al. In vitro fertilization as a risk factor for vasa
previa. Fertil Steril 2002; 78: 642-643.

76. - The anemic condition in a chronic renal failure patient is caused by defficence:
a) Glucose-6-phosphate dehydrogenase.
b) Erytropoyetin.
c) 1- antitripsine.
d) B12 vitamin.

La anemia en la IRC se caracteriza por ser normoctica y normocroma.


Puede detectarse con FG <60 ml/min, hacindose ms severa a medida que empeora la
funcin renal.
El dficit en la secrecin de EPO es el principal mecanismo patognico.
Otros factores mltiples contribuyen al desarrollo de la anemia renal. La vida media del
hemate est acortada. Algunas molculas del grupo de poliaminas, como la espermina y
espermidina, se comportan como toxinas urmicas; inhibiendo la eritropoyesis. Por otra
parte, en la IRC puede presentarse dficit de hierro y vitaminas, prdidas hemticas,
intoxicacin por aluminio y fibrosis de la mdula sea secundaria a hiperparatiroidismo.
La EPO es una glicoprotena sintetizada por las clulas intersticiales peritubulares renales
en el individuo adulto. La hipoxia estimula su secrecin, con el fin de conservar la masa de
hemates para satisfacer la demanda tisular de oxgeno. En la IRC se observa una respuesta
inapropiada. Los niveles plasmticos son anormalmente normales en relacin a los niveles de
bajos de hematocrito o hemoglobina.
La anemia, adems de la sintomatologa propia de cualquier anemia crnica, tiene
repercusiones sobre las funciones cognitivas, el sistema cardiovascular, la trombopata
urmica, la nutricin, la inmunidad y la disfuncin sexual. Tiene una significacin especial la
relacin de la anemia con la miocardiopata urmica. La anemia contribuye de forma
importante al desarrollo de hipertrofia ventricular izquierda (HVI).
En los ltimos aos, se conocen otros efectos de la EPO, ms all de los relacionados con la
eritropoyesis. La EPO es un potente regulador de la proliferacin y diferenciacin de las
clulas progenitoras endoteliales (EPCs). Las clulas madre CD34+ de la mdula sea pueden
diferenciarse en dos vas, la hematopoytica y la endotelial. La EPO, adems de estimular la
produccin de hemates, tendra la funcin de movilizar la circulacin de EPCs y actuar
sobre las EPCs maduras que expresan receptores de EPO. Estas clulas tendran una
particular relevancia en los fenmenos reparadores de neoangiognesis.

77. - A 26-year-old woman presents with malodorus gray-wellow discharge. You take a wet
mount preparation and observe Clue cells. The agent of this infection and its treatment
is:

a)
b)
c)
d)

Gardnerella vaginalis / clindamycin.


Gardnerella vaginalis / Ketoconazole.
Trichomona vaginalis / metronidazole / treat the partner.
Candida albicans / nistatin.

Gardnerella vaginalis fue clasificada como una sola especie y fue establecida como agente
causal de la vaginosis (antes conocida como vaginitis inespecfica). El cuadro clnico que
presenta es caracterizado por una secrecin blanca o blanco-griscea que se percibe
generalmente despus de la relacin sexual con olor ftido aminado (pescado). El
diagnstico certero es la base para evitar posibles complicaciones como la enfermedad
inflamatoria pelviana y las complicaciones del embarazo. El tratamiento se basa
principalmente en los frmacos como: metronidazol y clindamicina, debido a su efectividad
y espectro, pero como todos se deben emplear con adecuada prudencia debido a su
toxicidad. Adems de que se deben corregir o modificar los factores predisponentes, ya
que esta patologa va en aumento convirtindose por su frecuencia en un problema de salud
pblica.

Bibliografa:
1. Hernndez F. Gardnerella vaginalis mobiluncus en la etiologa de la vaginosis bacteriana.
Rev Costarricense Ciencias Mdicas 1998; 19: 57-61.
2. Hansen EA. Gardnerella. Rev Ginecol 2005; 25: 99.
3. Espinosa I, Lorenzo M, Bentancourt A, Rivern Y, Romero M. Caracterizacin bioqumica

y antignica de diferentes aislamientos de Gardnerella vaginalis.

Rev Cubana Invest Biomed 2005; 24: 22-7.


4. Taylor F. Vaginal flora morphotypic profiles and assessment of bacterial vaginosis in
women at risk for HIV infection.Infect Dis Obstet Gynecol
2004; 12: 121-6.

78.- Los sntomas de visin de cuerpos volantes (miodesopsias), destellos luminosos


(fotopsias) y disminucin de la visin perifrica en alguna zona del campo visual, estn
relacionados con la siguiente patologa:

a) Descompensacin de la retinopata diabtica.


b) Retinopata hipertensiva.
c) Degeneracin macular senil.
d) Desprendimiento de retina.

Desprendimiento de retina:

Es una separacin del epitelio pigmentario del resto de las capas de la retina.
FORMAS CLNICAS.
Exudativo. Tal vez el menos frecuente La retina se desprende por procesos exudativos
de los vasos coroideos.
Puede ocurrir en procesos inflamatorios, vasculares, como la hipertensin arterial o
neoplsicos.

Traccional. Se forman tractos fibrosos en vtreo que al contraerse traccionan la retina y


la desprenden. Ocurre en casos de hemorragia vtrea, como en los estadios finales de la
retinopata diabtica proliferativa.
Regmatgeno. El ms frecuente. Aparece un agujero o desgarro en la retina a travs del
cual pasa lquido al espacio subretiniano que despega la retina. La mayora de los desgarros
se producen entre el ecuador y la ora serrata. Los agujeros retinianos pueden producirse
por degeneraciones de la retina perifrica y por alteraciones del vtreo, como el
desprendimiento posterior del vtreo. Entre los factores de riesgo de los desgarros
retinianos estn la miopa, vejez, traumatismos, afaquia y degeneraciones perifricas de la
retina.
CLNICA.
Aparecen miodesopsias o moscas volantes si hay rotura de pequeos capilares o en el
momento del DVP. Cuando la retina se va desprendiendo se producen fosfenos, o visin de
luces, por estmulos mecnicos. Cuando el rea desprendida va aumentando de tamao,
aparece una sombra continua en el campo visual perifrico que va progresando hacia el
centro. Puede pasar desapercibida durante bastante tiempo, si no afecta al rea macular
En el fondo de ojo se aprecia una bolsa mvil, blanquecina, con pliegues y a veces algn
desgarro o solucin de continuidad. Si no se trata, acaba por desprenderse toda la retina.
Tienen peor pronstico aquellos desprendimientos de retina en los que la mcula est
afectada, los de afectacin temporal superior (tienen tendencia a despegar la mcula) y
aquellos que reciben tratamiento tardo.
TRATAMIENTO.
Hay que cerrar la solucin de continuidad quirrgicamente, ya que, de no hacerlo, se acaba
desprendiendo toda la retina. Se realiza mediante criocoagulacin o lser, que provocan una
inflamacin con fibrosis posterior que pega la retina.
Si la retina est muy separada de la coroides, es preciso acercar la esclera y coroides
cosiendo un material inerte sobre la esclera (esponja de silicona), o bien lo contrario,
acercar la retina a la pared del ojo inyectando gases expansibles.
Cuando existe traccin del vtreo se disminuye colocando un cerclaje con banda de silicona
alrededor del ojo en el ecuador, o en casos muy graves, se realiza una vitrectoma. En
sujetos expuestos a padecer desprendimiento por presentar reas de degeneracin
perifricas en la retina, es preciso hacer profilaxis mediante fotocoagulacin de las mismas
con lser. En un 70-90% se consigue una curacin anatmica. La recuperacin funcional
depender de si la mcula se ha desprendido o no y de la precocidad del tratamiento.
En un 10% de los ojos operados con xito se producen nuevos agujeros y recidiva del
desprendimiento. En otro 10% se produce un desprendimiento en el otro ojo.

Bibliografa:
1. American Academy of Ophtalmology. Basic and Science Course. 1995-96. Section 8-11.
2. American Academy of Ophtalmology. Basic and Science Course. Section 7: External
diseases and Cornea. 1990. pp. 136-45.
3. American Academy of Ophtalmology. Diabetic Retinopathy and Treatment. San
Francisco. California 1992.
4. American Academy of Ophtalmology. Intraocular Inflamation, Uveitis and Ocular
Tumors. 1992-1993.
5. American Academy of Ophtalmology. Manual de Neurooftal-mologa. 1995-1996.
6. Amig, et al. Acute Rheumatic Fever. Rheum. Dis. Clin. North Am. 1993; 19: 333-50.

79.- Cul de los siguientes datos es el ms relevante en el dengue hemorrgico?

a)
b)
c)
d)

Trombocitopenia ( <100,000)
Fiebre >39 C.
Poliserositis.
Hipoproteinemia.

El dengue es actualmente la ms importante arbovirosis que afecta al hombre. Su agente


etiolgico son los 4 serotipos del virus del dengue (D1-4). Se transmite entre humanos
mediante la picadura del mosquito Aedes aegypti. Se estima que el 40% de la poblacin
mundial vive en reas de riesgo de esta entidad (1,2).
Cuadro clnico
El curso de la enfermedad, desde el punto de vista clnico, se puede dividir en cuatro fases:
Inicial, crtica, de recuperacin y de convalecencia.
Fase inicial: En ella el enfermo tiene un sndrome febril sin localizacin. Es muy
sintomtica y adems de la fiebre predominan las manifestaciones generales como cefalea,
artromialgias,
dolor
retroocular
y
malestar,
puede
aparecer
rash.
Suele durar alrededor de 3 das, tras lo cual la fiebre cede y algunos casos de dengue
clsico comienzan a presentar manifestaciones hemorrgicas leves, pero la mayora tienden
a mejorar. Otro grupo menor de pacientes desarrollar la FHD/SCD. Los llamados signos de
alarma
que
preceden
al
choque
pueden
comenzar
en
esta
fase.
Fase crtica: Transcurre entre el 4to y 7mo da de la enfermedad, en ella se presentan los

sntomas que definen al DH, como son la extravasacin de plasma, la trombocitopenia, y las
manifestaciones hemorrgicas. Algunos pacientes desarrollan el sndrome de choque por
dengue que es la forma ms severa de la enfermedad. Las manifestaciones hemorrgicas
pueden ser tan leves que solo se hacen evidentes a travs de la prueba del torniquete, o tan
severas como sangramientos digestivos graves con compromiso hemodinmico.

Los signos de extravasacin de lquidos (derrame en serosas, hemoconcentracin) son


indispensables para hablar de FHD/SCD, ya que es la fuga de lquido la que casi siempre
lleva el paciente al choque y no las hemorragias. Este hecho es de gran importancia pues
influenciado por el nombre de fiebre hemorrgica de dengue, en no pocas ocasiones los
mdicos asistentes esperan las grandes hemorragias que nunca llegan y el paciente cae en
choque por la extravasacin de plasma.
Fase de recuperacin: Se inicia cuando cesa el escape de lquido y las manifestaciones de
sangrado comienzan a disminuir. Pueden aparecer edemas o agravarse los derrames serosos
producto de la sobre hidratacin. Tambin se puede apreciar un rash tardo asociado a
prurito intenso. Se recuperan el apetito y el nmero de plaquetas.
Fase de convalecencia: Puede prolongarse hasta ms de 6 meses, se caracteriza por
cefalea discreta, cansancio fcil y artromialgias. No se presenta en la totalidad de los
casos.
Alarma:
La tendencia progresiva de aumento del hematocrito y la disminucin progresiva del nmero
de plaquetas es un signo de laboratorio a tener en cuenta, an antes que llegue al clsico
20%. Y en opinin de los autores esta tendencia es suficiente para iniciar la rehidratacin
endovenosa y prevenir la hemoconcentracin.
Referencias:
1. Fowler VG Jr, Scheld WM, Bayer AS. Endocarditis and Intravascular Infections. In:
Mandell GL, Bennett JE, Dolin R, eds. Principles and Practice of Infectious Diseases. 7th
ed. Philadelphia, Pa: Elsevier Churchill Livingstone; 2009; chapt 77.
2. Karchmer AW. Infective Endocarditis. In: Libby P, Bonow RO, Mann DL, Zipes DP, eds.
Braunwald's Heart Disease: A Textbook of Cardiovascular Medicine. 8th ed. St. Louis, Mo:
WB
Saunders;
2007:chap
63.The
Lancet
Inf
Dis
2002;2:33-42.
3. Gubler DJ, Clark CG. Dengue/dengue hemorrhagic fever: the emergence of a global
health problem. Emerging Infectious Diseases. Atlanta USA. 1995;1:55-57.
4. Kour GP, Guzmn MG, Bravo JR, Triana C. Dengue haemorrhagic fever/ dengue shock
syndrome: lessons from the Cuban epidemic. 1981. Bull World Health Organ 1989;87:37580.
5. Organizacin Panamericana de la Salud. 2003: Number of reported cases of dengue and
dengue hemorrhagic fever (DHF), region of the Americas (by country and subregion).
[citado
del
25
de
agosto
de
2003].
Disponible
en
URL:
http://www.paho.org/English/AD/DPC/CD/dengue-cases-2003.htm.

6. Valdes L, Guzman MG, Kouri G, Delgado J, Carbonell I, Cabrera MV, Rosario D, Vazquez
S. La Epidemiologa del Dengue en Cuba en 1997. Rev Panam Salud Publica/Pan American
Journal of Public Health 1999;6:16-25.
7. Pelaez O et al. Havana dengue 3 epidemic, 2001. Enviada a Emerging Infection Diseases.
8. George R, Lum LCS. Clinical spectrum of dengue infection. In: Gubler DJ, Kuno G, eds.
Dengue and dengue hemorrhagic fever. London: CAB International, UK, 1997; 89-113.

80.- La deficiencia de factor intrnseco produce el siguiente tipo de anemia:


a) Micorctica.
b) Drepanoctica.
c) Talasmica.
d) Perniciosa.

Carencia del factor intrnseco


El factor intrnseco es una sustancia natural que normalmente se encuentra en el estmago
y es necesaria para absorber la vitamina B12 de los alimentos.
Una carencia del factor intrnseco ocasiona anemia perniciosa y deficiencia de vitamina
B12, lo cual puede causar anemia y problemas del sistema nervioso y del cerebro
(neurolgicos).
Las causas ms comunes de anemia perniciosa abarcan:

Debilitamiento del revestimiento del estmago (atrofia de la mucosa gstrica)

El sistema inmunitario ataca las clulas que producen el factor intrnseco


(autoinmunidad contra las clulas parietales gstricas)

Autoinmunidad contra el factor intrnseco en s.

El comienzo de la enfermedad es lento y puede tomar dcadas para establecerse por


completo. Aunque la forma congnita ocurre en nios, la anemia perniciosa por lo general no
aparece antes de los 30 aos en adultos y la edad promedio del diagnstico es a los 60 aos.

Referencias:
Antony AC. Megaloblastic anemias. In: Goldman L, Ausiello D, eds. Cecil Medicine. 23rd ed.
Philadelphia, Pa: Saunders Elsevier; 2007: chap 170.

Anda mungkin juga menyukai